Tạp chí Epsilon số 11

171 55 1
Tạp chí Epsilon số 11

Đang tải... (xem toàn văn)

Tài liệu hạn chế xem trước, để xem đầy đủ mời bạn chọn Tải xuống

Thông tin tài liệu

Ngược lại, một đề bài quá mẹo, được thiết kế để bẫy những học sinh áp dụng quy tắc một cách cẩu thả, không kiểm tra xem nó có áp dụng được không, thường sẽ được cảm nhận (khá đúng) như l[r]

(1)

tháng 10 - 2016 NO

Người ta thường hay nói “Mọi đường dẫn đến Roma” Nhưng đó đường lát gạch trang trí tuần hồn, chúng dẫn đến Lisbon!

NGUYỄN TIẾN DŨNG (Đối xứng nghệ thuật)

giới phụ nữ toán học. MICHAEL SIPSER

(Nước Mỹ chọn luyện đội tuyển thi toán quốc tế nào?)

Giải toán bạn Hà Huy Khoái

Nước Mỹ chọn luyện đội tuyển thi toán quốc tế nào? Lê Tự Quốc Thắng Đường thẳng Steiner Điểm Anti-Steiner Ngô Quang Dương

(2)

BIÊN TẬP VIÊN: Võ Quốc Bá Cẩn Ngô Quang Dương Trần Quang Hùng Nguyễn Văn Huyện Dương Đức Lâm Lê Phúc Lữ Nguyễn Tất Thu

Đặng Nguyễn Đức Tiến

No 11

(3)

Những ngày năm trước ý tưởng Epsilon cịn chưa hình thành Lúc đó, với gợi ý GS Ngơ Bảo Châu, Hội tốn học Việt Nam Viện nghiên cứu cao cấp tốn với số nhân tích cực cố gắng xin phép đời tạp chí Pi, tạp chí phổ biến tốn học dành cho học sinh sinh viên Nhưng thủ tục không đơn giản người tưởng ban đầu dự án bị chựng lại Epsilon đời tổng diễn tập trước vào trận đánh thức Ngày ý tưởng đời Epsilon cơng bố, TS Lê Thống Nhất, người nhắm làm Phó tổng biên tập Pi làm thơ chúc mừng

Chỉ cánh én nhỏ Không làm nên Mùa xuân Không nhỏ Chẳng có thứ ta cần Từ cánh én nhỏ Sẽ sinh sôi Ra trời én nhỏ Rõ ràng Mùa Xuân

Epsilon số 11 lần xuất xưởng bối cảnh thủ tục thành lập Tạp chí Pi có bước tiến triển lạc quan có giấy phép thức tháng 10 Có nghĩa khả số báo Pi đời vào tháng 1/2017 cao

Trong chờ đợi số báo chuyên nghiệp đó, Epsilon làm nhiệm vụ mình, chắt chiu điều nho nhỏ đem đến cho bạn đọc

(4)

Hà Huy Khoái

Giải toán bạn 6

Nguyễn Tiến Dũng

Đối xứng nghệ thuật 9

Nguyễn Ái Việt

Tô Pô học ứng dụng Vật lý 34

Terrence Tao (Phùng Hồ Hải dịch)

Về câu hỏi trắc nghiệm toán học 39

Lê Tự Quốc Thắng

Nước Mỹ chọn luyện đội tuyển thi toán quốc tế (IMO) nào? 45

Trần Thanh Hải

Luận lý với 51

Henry Trần

Các phương pháp sai phân hữu hạn cho phương trình đạo hàm riêng 55

Kiều Đình Minh

Phương pháp giải tích tốn Olympic 79

Trần Quang Hùng

Tổng quát hoá đường thẳng Droz Farny 93

Vandanjav Adiyasuren

Note on Hermite - Hadamard Inequalities 100

Slava Gerovitch (Hoàng Mai dịch)

Andrei Kolmogorov - Người mở đường ngành xác suất đại 103

Đào Thanh Oai

Mở rộng bổ đề Sawayama định lý Sawayama-Thebault 109

Ngô Quang Dương

Đường thẳng Steiner Điểm Anti-Steiner 113

Lê Phúc Lữ

Về toán tam giác 80-80-20 (tiếp theo) 125

Lê Phúc Lữ

(5)

Nguyễn Quốc Khánh

Những câu đố Mát-Xcơ-Va 149

Ban Biên tập Epsilon

Bài toán hay - Lời giải đẹp 153

Ban Biên tập Epsilon

(6)

GIẢI TỐN CÙNG BẠN

Hà Huy Khối (Hà Nội)

LỜI TỰA

Trình bày lời giải tốn ta biết lời giải khơng phải khó Nhưng trình bày để người đọc hiểu lối suy nghĩ dẫn dắt đến lời giải ln khó Và thực điều mà ta cần học Vì suy cho cùng, khơng thể học thuộc hết tất lời giải Cái mà ta học, suy luận có lý dẫn dắt ta đến với lời giải Số11của Epsilon xin giới thiệu với độc giả toán với dẫn dắt thầy Hà Huy Khối

Cái khó người đứng trước tốn tìm phương pháp để giải quyết? Khơng “mách” cho bạn với đó, cần dùng phương pháp (trừ tập “minh hoạ” cuối chương sách) Những sách tập (với đề ra, lời giải hồn chỉnh) nhiều khơng cho ta biết làm mà tác giả tìm cách giải Dù hiểu lời giải, chí nhớ lời giải, chưa thể nói hiểu toán chưa trả lời câu hỏi Và gặp lại tốn đó, với cách phát biểu khác, bạn tưởng gặp lần đầu

Những điều nói gợi cho ý định viết sách tập, khơng có sẵn lời giải đẹp đẽ, mà bạn đọc với tác giả lần mị để tìm cách giải Để làm ví dụ cho việc đó, mà tơi nghĩ cần thiết giảng dạy, chọn (chưa thể gọi “chọn lọc”, khơng có đủ thời gian) số toán thuộc loại khác nhau, thuộc phần mà theo chưa giảng dạy nhiều THPT (chuyên)

Tôi cố gắng bổ sung để đến hồn thành sách tập theo cách Ta toán sau đây, mà theo kinh nghiệm cá nhân, “độ khó” tương đương với kỳ thi học sinh giỏi tồn quốc mơn tốn (có thể khơng khó nhất, khơng dễ nhất)

Ví dụ Cho plà số nguyên tố lẻ Hãy xây dựng dãy {an} ∈ N sao cho∀n, an là số nguyên

không âm nhỏ khác với số trước dãy, vàa0, a1, , ankhơng chứa cấp

số cộng khác cópsố hạng.

(7)

Từ ra, rõ ràng ta có       

a0 =

a1 =

ap−2 =p−2 Dễ thấyap−1 6=p−1,và dãy tiếp tục sau:

      

ap−1 =p

ap =p+ · · ·

a2p−3 = 2p−2

Tiếp theo phải làa2p−2 = 2p.Như vậy, ta “tuần tự” cộng thêm đơn vị, làm với đọanp−1số hạng

Thử nghĩ lại, ta gặp điều tương tự? “Saup−1thì phải thay đổi?” Điều gợi ý cho ta

để giải tốn, cần sử dụngcơ sốp−1 Tất nhiên, đoán

hướng Cần phải kiểm nghiệm

Xét số hạng cho viết số p−1.Từa0 đếnap−2 thìak = k.Tất nhiên, viết số≥p−1thìk =k,vớik= 0,1, , p−2.Nhưng viếtp−1trong sốp−1, ta đượcp−1 = 10,trong khiap−1 =p.Số10chỉ bằngpnếu xem số sốp Tiếp tục với số viết đây, ta dự đoán quy luật:annhận cách viếtntrong

cơ sốp−1và đọc sốp

Xét dãyB ={bn}, n = 0,1, , màbnnhận cách viếtntrong sốp−1,đọc sốp.Ta hy vọng rằng, dãy cần tìm

Nhận xét Sốb ∈B khi viếtbtrong sốpthìbkhơng chứa chữ sốp−1

Điều rõ ràng từ định nghĩa dãy{bn}

Nhận xét TrongB khơng có cấp só cộng gồmpphần tử.

Thật vậy, giả sử∃a, d ∈Nsao cho

a, a+d, , a+ (p−1)d∈B

Cần suy mâu thuẫn, tức cần chứng minh số có số khơng thuộcB,tức số chứa chữ số≡(p−1) (mod p)

Tất nhiên điều dẫn đến việc cần chứng minh tồn tạii mà chữ số thứ icủa số lập thành hệ thặng dư đầy đủ modulop−1

Giả sửa=a1a2· · ·amvàd=d1d2· · ·dm.Giả sửilà chữ số khác0đầu tiên củadtính từ phía bên phải

d=d1d2· · ·di00| {z }· · ·0 ksố

(8)

Khi nếua+kd=c1c2· · ·ci· · ·cn,thìci ≡ai+k·di (mod p).Doplà số nguyên tố,kvà di nhỏ hơnpnênai +kdi, k = 0,1, , p−1lập thành hệ thặng dư đầy đủ modulop,tức tồn tạikđểa+kdcó chữ số (thứitừ phải sang) bằngp−1

Để kết thúc, ta chứng minh an = bn với n Ta có a0 = b0 Giả sử ak = bk với k =

0,1,2, , n−1.Theo định nghĩa dãyanta cóan ≤bn

Nếuan ∈Bthìankhơng thể nhỏ hơnbn(vì ngược lại, theo giả thiết quy nạp,anphải ainào đứng trước Như vậy, cịn phải chứng minhan∈B

Giả sử ngược lại, an 6∈ B Ta suy mâu thuẫn tìm cấp số cộngpsố hạng dãy{an}.Thực ra, “trong tay” có phần tử dãyB, nên phải dựa vào chúng Cần tìm cấp số cộng số thuộcB mà ta biết, tức số nhỏ hơnanvà không chứa chữ sốp−1khi viết sốp.Để ý rằngancó số chữ số(p−1)khi viết sốp.Như vậy, cần trừ số dương không vượt qp−1tại vị trí để

được số thuộcBvà nhỏ hơnan.Cách làm rõ ràng

Giả sửan=α1α2· · ·αm.Xét sốdmà viết sốpcó dạngd=d1d2· · ·dm

d=

1 nếuαi =p−1

0 nếuα1 6=p−1

Do tồn chữ số củaanbằngp−1nênd≥1

Xét dãyan−d, , an−(p−1)d.Các số khơng có chữ sốp−1khi viết sốp,tức thuộcB Mặt khác, số hơnannên theo giả thiết quy nạp, chúng thuộc dãy {an}

(9)

ĐỐI XỨNG TRONG NGHỆ THUẬT

Nguyễn Tiến Dũng (Đại học Toulouse, Pháp)

Hình 1: Mái nhà thờ Sagrada Familia Barcelona (Tây Ban Nha), nghệ sĩ kiến trúc sư Antonio Gaudí (1852 1926) thiết kế, nhìn từ bên gian Nguồn: Wikipedia

Các hình đối xứng hình có giống phần, tức chúng tuân thủ nguyên lý

(10)

1 Các phép đối xứng

Hình 2: Mặt nước phản chiếu tạo hình ảnh với đối xứng gương

Trong tốn học có định lý sau: Mọi phép biến đổi bảo tồn khoảng cách khơng gian bình thường (tức không gian Euclid3chiều mặt phẳng2chiều) thuộc

một bốn loại sau:

1)Phép đối xứng gương(mirror symmetry), hay gọi làphép phản chiếu(reflection): Trong không gian3chiều phản chiếu qua mặt phẳng đó, cịn mặt phẳng phản chiếu

qua đường thẳng

2)Phép quay(rotation): Trong không gian3chiều quay quanh trục đó, cịn mặt

phẳng quay quanh điểm đó, theo góc

3)Phép tịnh tiến(translation): Dịch chuyển tất điểm khoảng cách theo hướng Như kiểu ánh xạ W.x; y/ 7! x CT; y/trên mặt phẳng, dịch chuyển

điểm theo hướng trụcxmột đoạn có độ dài bằngT

4)Phép lượn(glide), kết hợp phép đối xứng gương phép tịnh tiến theo hướng song song với trục hay mặt đối xứng gương Như kiểu ánh xạ g W x; y/ 7!

xC T2; y kết hợp phép đối xứng gương biến y thành y phép tịnh tiến biến x

thànhxCT2 Chú ý thực liên tiếp phép lượn hai lần lại

phép tịnh tiến

Định lý khơng q khó, dùng làm tập thú vị cho học sinh THCS (trường hợp

2chiều) THPT (trường hợp3chiều)

(11)

Hình 3: Con biển có đối xứng gương lẫn đối xứng quay phần năm vịng trịn Có loại biển có n chân vớin > (thậm chí vớin D 18), đối xứng quay

theo góc n

Hình 4: Đường viền sư tử thành cổ Persepolis (Iran)

thường, tức phép giữ nguyên tất điểm Nhưng nói đến đối xứng, người ta thường hiểu phép đối xứng khơng tầm thường Nếu hình có phép đối xứng khơng tầm thường, gọi hình đối xứng Hình mà có nhiều phép đối xứng, hình đối xứng

(12)

Hình 5: Một dải gỗ trang trí Nguồn: invitinghome.com

khơng cần thực tồn hình mà phần hình, ta hình dung hình trải dài nối tiếp đến vô cùng, phép tịnh tiến phép lượn trở thành phép đối xứng, theo nghĩa mở rộng

Hình khắc họa sư tử tường thành phố cổ Persepolis Iran ví dụ phép đối xứng tịnh tiến theo nghĩa mở rộng: vector tịnh tiến vector nối từ mũi sư tử đến mũi sư tử Cịn hình có phép đối xứng lượn theo nghĩa mở rộng

Hình 6: Các cơng trình kiến trúc hay có đối xứng gương hai bên Trong ảnh Mosque (nhà thờ Hồi giáo) Abu Dhabi

Trong toán học, tập hợp phép đối xứng vật hay hình gọi nhóm

(13)

“hợp thành” chúng: ta thực biến đổi theo phép thứ nhất, biến đổi phép thứ hai Tất nhiên, hai phép biến đổi bảo tồn hình, hình bảo tồn ta thực liên tiếp hai phép biến đổi

Hình 7: Tháp Phước Duyên chùa Thiên Mụ (Huế) có đối xứng theo hình bát giác, kiến trúc xung quanh có đối xứng gương

Các cơng trình kiến trúc, đồ vật, hình họa trang trí nghệ thuật phân loại theo nhóm đối xứng chúng Ví dụ, tháp Phước Duyên chùa Thiên Mụ (Hình 7) có tám mặt, với đáy giống hình bát giác đều, nhóm đối xứng giống nhóm đối xứng hình bát giác (nếu ta bỏ qua chi tiết khơng đối xứng tháp, ví dụ khơng phải mặt có cửa) Tháp Eiffel Paris (Hình 8) có bốn mặt giống nhau, đáy hình vng, nên nhóm đối xứng giống nhóm đối xứng hình vng

Ở đây, tìm hiểu phân loại theo nhóm đối xứng cho hình đa giác, cho trang trí đường viền (frieze) cho kiểu lát gạch tuần hoàn (tessellation)

2 Phân loại đa giác theo nhóm đối xứng

Vào khoảng năm2013;tơi có dành buổi để tìm hiểu với gái, lúc học năm

(14)

Hình 8: Tháp Eiffel Paris với mặt nhau, có nhóm đối xứngD4giống hình vuông

Đầu tiên xét tam giác Chúng có đối xứng (trong trường hợp tam giác khơng cân, có phép “để n” bảo toàn tam giác),2đối xứng (nếu tam giác cân, phép để n

cịn có phép đối xứng gương), đối xứng tam giác đều? Có người trả lời là3;và có người trả lời là4:Câu trả lời xác là6;trong có3phép đối xứng

gương, và3phép quay theo góc0ı; 120ıvà240ı(quay theo góc0ıcó nghĩa để yên)

Đến lượt tứ giác: Nhiều đối xứng hình vng, với 8đối xứng (4 đối xứng gương

phép quay), đến hình chữ nhật hình thoi có4đối xứng Tiếp theo hình

có2đối xứng: Hình bình hành (với đối xứng quay180ı), hình thang cân, hình mũi tên hình

cánh diều (với đối xứng gương) Cịn lấy hình tứ giác tùy ý, khơng có cạnh cạnh nào, nhóm đối xứng nhóm tầm thường, có phần tử, phép để yên

Đến lượt ngũ giác: lại có trường hợp, tương tự với tam giác, khơng có nhiều trường hợp tứ giác Khi ngũ giác có52D10đối xứng, khơng

là nhóm đối xứng có phần tử (phép để yên) có hai phần tử (đối xứng gương phép để yên) Con biển Hình có hình năm cánh đều, nhóm đối xứng nhóm đối xứng ngũ giác

Đến lượt lục giác lại có nhiều trường hợp khác nhau, đến thất giác lại có trường hợp, Từ thí nghiệm này, ta rút số kết luận tốn học sau:

Hìnhn-giác có nhiều là2nđối xứng, ứng với trường hợpn-giác Nhóm

đối xứng trường hợp gồmnđối xứng gương vànphép quay, gọi lànhóm nhị

diện(dihedral group)Dn Nếun-giác khơng đều, nhóm đối xứng nhóm

(15)

Hình 9: Các đa giác số đối xứng chúng

Nếunlà số ngun tố có khả xảy ra: nhóm đối xứng làDn, nhóm

đó có hai phần tử phần tử không tầm thường đối xứng gương, nhóm tầm thường (chỉ có phép để yên)

Khi số cạnh đa giác tiến tới vơ ta hình trịn, hình có nhiều đối xứng hình phẳng: vơ hạn đối xứng (quay quanh tâm theo góc tùy ý, đối xứng gương theo đường kính tùy ý)

3 Bảy kiểu trang trí đường viền

Các trang trí dải mép tường, mép bàn, mép váy, hay đường dài hẹp gọi chung trang tríđường viền(“frieze” tiếng Anh, “frise” tiếng Pháp) Có thể hình dung đường viền dải băngDhẹp dài (coi dài vô tận cho đơn giản) nằm ngang

mặt phẳng:

D DRŒ a; aD f.x; y/2 R2j a6y 6ag:

Theo nguyên lý lặp lặp lại đẹp, người ta thường trang trí đường viền cách tuần hồn, tức hình trang trí dải băng D có tính chất bất biến theo phép tịnh tiến (dịch

(16)

Hình 10: Trang trí mái nhà Toulouse

Hình 11: Gạch đá hoa trang trí theo kiểu phương Đơng

Ví dụ Hình 4, sư tử xếp cách đường viền, dịch sư tử sang bên phải đoạn khoảng cách hai mũi hai sư tử liên tiếp sư tử

Các phép tịnh tiến bảo tồn trang trí đường viền tuần hồn tạo thành nhóm tương đương vớiZ, tức tập số nguyên: với số nguyênk 2Zthì ta có phép “tịnh tiếnkbước”

bảo tồn hình trang trí:k W.x; y/ 7!.x CkT; y/

Ngồi phép tịnh tiến ra, hình trang trí đường viền cịn bất biến theo phép biến đổi khác Người ta phân loại kiểu trang trí đường viền tuần hồn qua nhóm nhóm đối xứng chúng Tổng cộng có bảy kiểu khác nhau:

Kiểu thứ gọi làhop (nhảy lò cò) Trong kiểu này, có phép tịnh tiến bảo tồn hình trang trí Hình dung vết chân bàn chân nhảy lị cị lên phía trước Các sư tử Hình trang trí theo kiểu hop

(17)

Hình 12: Trang trí hàng rào đá Ấn Độ, kỷ XVI-XVII

Kiểu thứ ba gọi là sidle (đi ngang) Trong kiểu này, ngồi phép tịnh tiến, cịn phép đối xứng gương theo trục dọc Hình dung hai chân xếp theo hướng dọc ngang cua, đối xứng gương đối xứng hai chân Hình 10 ví dụ

Kiểu thứ tư gọi làspinning hop (nhảy xoay lị cị) Trong kiểu này, có phép quay180ıcũng bảo tồn hình trang trí Hình 11 ví dụ.

Hình 13: Kiểu trang trí “Ngaru” thổ dân Maori (New Zealand)

Kiểu thứ năm gọi là spinning sidle (đi xoay ngang) Trong kiểu này, phép tịnh tiến theo chiều ngang, cịn có phép đối xứng gương theo trục dọc (đối xứng hai chân) phép quay180ı:Chú ý tâm phép quay18ınằm

ngoài trục đối xứng, kết hợp phép quay180ıvới phép đối xứng gương phép

lượn (glide) Hình 12 coi ví dụ kiểu đường viền thứ năm bỏ qua vài chi tiết

(18)

Hình 14: Một góc balcon Paris

Kiểu thứ bảy gọi làspinning jump (nhảy xoay hai chân), kiểu cuối Trong kiểu này, ngồi phép tịnh tiến, cịn có phép đối xứng gương theo trục ngang lẫn trục dọc, phép quay180ı Hình 14 ví dụ

4 Mọi đường dẫn tới Lisbon

Người ta thường hay nói “Mọi đường dẫn tới Roma” Nhưng đường lát gạch trang trí tuần hồn, chúng dẫn tới Lisbon!

Thành phố Lisbon xinh đẹp nằm bên bờ biển Đại Tây Dương có nhiều khu lát gạch đá vôi (limestone) nhỏ màu trắng đen, theo phương pháp truyền thống gọi “lát gạch Portugal” (Portuguese pavements), tạo thành hình trang trí nghệ thuật

Người bạn đồng nghiệp Rui Loja Fernandes tôi, cựu chủ tịch Hội Toán học Portugal cựu giáo sư Đại học Bách khoa Lisbon (Instituto Superior Técnico de Lisboa) có kể rằng, sau nghe nói nhóm đối xứng việc lát gạch, đích thân ơng thị trưởng thành phố mời nhà toán học trường làm cố vấn để đảm bảo tất kiểu nhóm lát gạch khác xuất khu Lisbon

Khi trang trí mặt phẳng, quảng trường Rossio (Hình 15) hay tường nhà, sàn nhà, vải, thảm, v.v người ta chọn cách trang trí tuần hồn hai chiều (tức có hai hướng tịnh tiến khác bảo tồn hình) Những kiểu trang trí gọi làlát gạch(tiếng Anh làtessellation, tiếng Pháp làpavage) tuần hồn Bởi ta hình dung lấy viên gạch trông giống (hoặc vài kiểu gạch) xếp chúng lại cạnh hình trang trí ý muốn

(19)

Hình 15: Quảng trường Rossio Lisbon với hình sóng tuần hồn

Hình 16: Ảnh quảng trường Restauradores Lisbon Jee Wee, với lát đá theo nhóm đối xứng p4

trên Hình 15 có đối xứng quay theo góc (180ı), cịn đá hoa Hình 16 Hình 19 có

đối xứng quay theo góc (90ı)

Nếu kiểu lát gạch tuần hồn có đối xứng quay, tính chất tuần hồn nên góc quay nhỏ phải số;23 ;2;6 (ứng với chuyện lát kín mặt phẳng

(20)

Hình 17: Đường đỏ trục đối xứng gương, điểm xanh tâm đối xứng xoay180ı Nguồn:

kleinproject.org

Tương tự nhóm đường viền, ta phân loại nhóm lát gạch theo chuyện có đối xứng quay hay khơng góc quay có, có đối xứng gương hay khơng, có đối xứng lượn hay khơng, tâm đối xứng quay có nằm trục đối xứng gương hay không

Người đưa phân loại đầy đủ cho nhóm nhà toán học khoáng vật học người Nga Evgraf Fedorov (1853-1919) vào cuối kỷ XIX Có tổng cộng 17 nhóm lát gạch khác nhau, ứng với 17 kiểu lát gạch tuần hồn khác Hình 18 sơ đồ minh họa tồn 17 kiểu

Mỗi hình Hình 18 ứng với kiểu lát gạch Miền tô xanh miền mà làm viên gạch có vậy, dịch chuyển theo phép biến đổi đối xứng nhóm tương tứng, ta lát kín vừa khít tồn mặt phẳng

Trong số ký hiệu 17 kiểu nhóm đối xứng Hình 18, có ký hiệu bắt đầu chữ c, có nghĩa “centred” (ở giữa) Mỗi kiểu “c” có hai vector tịnh tiến có độ dài (tạo thành hình thoi), trục đối xứng trục glide hình khơng song song với hai vector mà lại “nằm giữa” hai vector (tức song song với tổng chúng) Tất kiểu lại bắt đầu chữ p, có nghĩa “primitive” (nguyên thủy): kiểu này, trục đối xứng hay glide song song với vector tịnh tiến “nguyên thủy” hình Chữ số ký hiệu kiểu cho biết có phép quay theo góc bao nhiêu: chữ số làkthì

góc quay nhỏ

k Ví dụ có chữ số4thì có phép quay theo góc D

4 Chữ

m ký hiệu dùng để đối xứng gương (mirror), chữ g dùng để đối xứng lượn (glide)

Danh sách chi tiết17kiểu sau:

Kiểu thứ nhất, ký hiệu làp1, kiểu có đối xứng tịnh tiến, ngồi khơng cịn thêm đối xứng khác Hình 20 phía bên trái ví dụ

Kiểu thứ hai, ký hiệu làpg, có thêm glide, khơng có đối xứng quay hay đối xứng gương Trong kiểu có hai hướng tịnh tiến vng góc với Tranh lát gạch Kỵ sĩ Maurits Cornelis Escher Hình 21 ví dụ tiêu biểu (nếu ta bỏ qua màu ngựa): phép glide chuyển ngựa màu nhạt thành ngựa màu thẫm

(21)

Hình 18: Sơ đồ 17 nhóm lát gạch Nguồn: http://black.mitplw.com/

(22)

Hình 20: Một trang trí giấy dán tường có nhóm đối xứng p1, trang trí kiểu Ai Cập có nhóm đối xứng pm

Hình 21: Tranh lát gạch “Kỵ sĩ” “Đầu Escher” Escher

là ví dụ: Các trục đối xứng gương trục đối xứng bơng hoa lys, cịn trục glide song song nằm hai trục đối xứng gương liên tiếp

Kiểu thứ tư, ký hiệu là pm, khơng có đối xứng quay có đối xứng gương, khơng có glide với trục nằm trục đối xứng gương kiểu thứ ba Một ví dụ trang trí kiểu Ai Cập Hình 20 phía bên phải Chú ý kiểu có vector tịnh tiến song song với trục đối xứng vector tịnh tiến vng góc với trục đối xứng

Kiểu thứ năm, ký hiệu làp2, ngồi đối xứng tịnh tiến cịn có thêm đối xứng quay theo góc

, ngồi khơng có thêm đối xứng khác Hình lát gạch đầu ông Escher (với đầu

chổng ngược qua phép quay180ı) Hình 21 ví dụ

Kiểu thứ sáu, ký hiệu làpgg, khơng có đối xứng gương, có hai họ đối xứng glide với trục glide vng góc với Kiểu có đối xứng quay 180ı, lấy tích hai

(23)

Hình 22: Vải trang trí hoa lys có nhóm đối xứng kiểu cm thảm phương Đơng có nhóm đối xứng kiểu pmm

Hình 23: Sàn lát gỗ có nhóm đối xứng kiểu pgg, cịn hình trang trí bình cổ từ Kerma (Sudan) đối xứng kiểu pmg

Kiểu thứ bảy, ký hiệu làpmg, vừa có đối xứng gương, vừa có đối xứng quay180ıvới tâm khơng

nằm đối xứng gương Tích hai phép đối xứng phép glide, nên ký hiệu kiểu có m (mirror) g (glide) Chiếc bình cổ đại Hình 23 có kiểu trang trí thành bình

Kiểu thứ tám, ký hiệu làpmm Thay có đối xứng gương theo hướng đối xứng glide theo hướng vng góc với nó, kiểu pmm có hai đối xứng gương theo hai hướng vng góc với nhau, tích chúng phép quay180ı Tấm thảm bên phải Hình 22 ví dụ

Kiểu thứ chín, ký hiệu là cmm có đối xứng giống kiểu pmm, ngồi cịn có phép quay180ıvới tâm không nằm trục đối xứng gương Hình xây gạch thành

tường Hình 24 ví dụ nhóm lát gạch kiểu cmm Các điểm tô đỏ tô xanh hình tâm đối xứng quay180ıcủa hình Các trục đối xứng gương qua

điểm đỏ không qua điểm xanh

Kiểu thứ mười, ký hiệu làp3, có đối xứng quay với góc nhỏ

(24)

Hình 24: Mặt tường gạch có nhóm đối xứng kiểu cmm xứng gương Hình 25 ví dụ

Kiểu thứ mười một, ký hiệu làp3m1, có đối xứng quay với góc

3 vịng trịn, có đối xứng gương,

và tâm đối xứng quay nằm trục đối xứng gương

Kiểu thứ mười hai, ký hiệu làp31m, có đối xứng gương, có đối xứng quay với góc

4 vịng trịn

và tâm khơng nằm trục đối xứng quay

Kiểu thứ mười ba, ký hiệu là p4, có đối xứng quay với góc

4 vịng trịn (tức

2) khơng có

đối xứng gương Hình 16 ví dụ

Kiểu thứ mười bốn, ký hiệu là p4g, có đối xứng quay với góc

4 vịng trịn, có đối xứng gương,

và có đối xứng glide với trục tạo thành góc45ıvới trục đối xứng gương

Kiểu thứ mười lăm, ký hiệu làp4m, có đối xứng quay với góc

4 vịng trịn, có hai đối xứng

gương với trục tạo với góc45ı Hình 19 ví dụ

(25)

Hình 26: Một cửa sổ lăng Salim Chishti, Ấn Độ, có nhiều kiểu nhóm đối xứng lát gạch

Kiểu thứ mười sáu, ký hiệu là p6, có đối xứng quay với góc 1/6 vịng trịn (tức

3) khơng

có có đối xứng gương Hình 27 bên phải ví dụ

Hình 27: Sàn đá hoa Duomo di Siena (Toscana, Italia) có nhóm đối xứng p6m, cịn tranh “Con bướm” Escher có nhóm đối xứng p6

(26)

bên trái ví dụ

Ngồi Lisbon, có nơi khác coi có đủ 17 kiểu nhóm lát gạch khu cung điện Alhambra (tiếng Ả Rập có nghĩa “Đỏ”) người Hồi giáo xây Granada, Tây Ban Nha, từ kỷ XIII Đây cung điện nguy nga, với nhiều trang trí tuần hồn (và khơng tuần hồn) đẹp tường Tuy nhiên, chưa thấy cơng bố kiểm chứng có đủ 17 kiểu lát gạch

Hình 28: Trần gian phịng Abencerrajes cung điện Alhambra, với nhiều trang trí kiểu lát gạch khác tường

Người ta nói họa sĩ Escher thăm Alhambra có ý tưởng cảm hứng vẽ tranh lát gạch tiếng ơng từ hình trang trí tường cung điện này, tranh Escher có chứa đủ 17 kiểu nhóm lát gạch Trong sáchCác giảng tốn cho Mirella có chương tạo hình trang trí bắt chước Escher cách sử dụng phép đối xứng Vào thập kỷ 1980, nhà toán học William Thurston nghĩ phương pháp hình học mới, dựa lý thuyết vềorbifold(có thể hiểu orbifold tập hợp quỹ đạo (orbit) nhóm hữu hạn tác động lên đa tạp), để phân loại nhóm lát gạch Phương pháp Thurston cho giải thích gọn ghẽ có 17 nhóm, để mơ tả tác động nhóm mặt phẳng phải làm

Nếu mặt phẳng “chỉ có”17cách lát gạch tuần hồn, khơng gian ba chiều số

(27)

Hình 29: Hai ví dụ lát gạch chiều Andrew Kepert Nguồn: wikipedia Các viên gạch

“truncated octahedra” (“bát diện cụt”) “rhombic dodecahedra” (“thập nhị diện thoi”)

kê chúng tất nhiên cần sách, hình dung chúng cịn khó hiều so với hình dung nhóm lát gạch hai chiều

5 Đối xứng không gian phi Euclid

Ngồi mặt phẳng ra, cịn có hai loại mặt khác mà có phép tịnh tiến, phép phản chiếu phép quay bảo toàn khoảng cách, mặt cầumặt hyperbolic (hay gọi mặt Lobachevsky) Chúng không gianphi Euclid Các không gian phi Euclid lát gạch tương tự mặt phẳng, hình lát gạch cho tác phẩm đẹp mắt

Hình 30 ví dụ lát gạch hình cầu Vấn đề lát gạch phủ hình cầu liên qua đến vấn đề phân loại đa diện gần đều, mà bàn tới Chương??

Có thể hình dung mặt hyperbolic dạng đĩa (khơng có biên), gọi đĩa Poincaré Khoảng cách đĩa khơng giống khoảng cách mặt phẳng bình thường, mà tăng lên nhanh điểm tiến tới gần biên đĩa

(28)

Hình 30: Một cầu trang trí “Thiên thần quỷ sứ” dựa theo tranh Escher có bán amazon, hai mơ hình lát gạch hình cầu giấy đất sét Makoto Nakamura

Hình 31: Lát mặt hyperbolic ảnh hoa hồng, sử dụng phần mềm online từ trang mạng http://www.malinc.se/ Malin Christersson

6 Lát gạch khơng tuần hồn

Vào năm 1982, nhà vật lý Dan Shechtman phát tồn vật rắn mà cấu trúc phân tử khơng tuần hồn Người ta gọi cấu trúc làgiả tinh thể (quasicrystal) Nhờ phát mà ơng giải Nobel vào năm 2011

(29)

Hình 32: Bộ kiểu viên gạch Raphael Robinson, hai gạch Penrose viên

Người ta xây dựng lý thuyết kiểu gạch có tính chất ép cho việc lát gạch khơng thể tuần hồn Raphael Robinson có lẽ người chứng minh được, vào năm 1971, tồn kiểu viên gạch lát kín mặt phẳng cho khơng thể lát chúng cách tuần hồn Ông nghĩ hình viên gạch Hình 32 bên trái Dùng viên gạch lát kín mặt phẳng, minh họa Hình 33 Chỉ có điều, hình vng màu da cam gạch lát tạo nên bắt buộc nằm góc hình vng màu da cam to Từ suy hình lát gạch khơng thể tuần hồn

Hình 33: Lát mặt phẳng viên gạch Robinson

(30)

đều hình thoi, Hình 32 Các góc hình thoi 5;

4 ;

2

5 (tương tự góc ngũ giác hình cánh đều), chúng có

thể cộng với thành2 để lát khớp đỉnh Một gạch viên khác Penrose, với

một viên hình cánh diều viên hình mũi tên, Hình 32 bên phải, có tính chất tương tự Các viên gạch kiểu Penrose có sản xuất dùng để lát sàn nhà nhiều nơi giới

Hình 34: Tranh sơn dầu họa sĩ Urs Schmid (1995) vẽ kiểu lát gạch Penrose dùng viên gạch hình thoi

Penrose khơng phải người nghĩ viên gạch có góc bội số

5 Ơng lấy ý

tng ú t cỏc tỏc phm ca Albrecht Dăurer Johannes Kepler từ thời kỷ XVI-XVII Từ trước nữa, nghệ sĩ Hồi giáo (ắt hẳn đồng thời nhà toán học) nghĩ việc dùng “viên gạch” Hình 35, gọi girih, có góc bội

5, để lát trang trí

Viên girih to có hình thập giác Tiếp đến viên hình lục giác với góc nhọn

2

5 góc tù

5 Tiếp hình nơ bướm với góc nhọn

5 ,

hình thoi với góc nhọn

5 , sau hình ngũ giác Girih theo tiếng

(31)

Hình 35: Các viên girih

kế Trên tường trang trí girih, nói chung khơng nhìn thấy biên “viên gạch girih” trên, thực khơng có viên gạch

Hình 36: Bìa kinh Quoran từ kỷ XIV, thiết kế girih Nguồn: David James, Qur’ans of the Mamluks (Thames & Hudson) & aramcowworld.com

Với kiểu thiết kế girih, người Hồi giáo khơng tạo hình nghệ thuật lát tường tuần hồn, mà hình khơng tuần hồn có đối xứng khác, ví dụ đối xứng kiểu cánh hay 10 cánh (đối xứng quay theo góc

5;khơng thể tuần hồn có đối xứng quay

(32)

Hình 37: Khu lăng tẩm “Shah-i Zinda” (“Vua Sống”) Samarquand, Uzbekistan (ảnh Fulvio Spada), trang trí girih bên

7 Các trang web tham khảo

https://en.wikipedia.org/(rất nhiều thông tin tra từ Wikipedia)

http://bridgesmathart.org(trang web hội nghị quốc tế thường niên toán học nghệ thuật Bridges: Mathematical Connections in Art, Music, and Science, với nhiều triển lãm hay)

http://www.mathaware.org/mam/03/(trang web AMS với nhiều tài liệu toán nghệ thuật)

http://www.math.nus.edu.sg/aslaksen/teaching/math-art-arch.html

(một cua giảng toán học nghệ thuật NUS, Singapore) http://www.malinc.se/(làm hyperbolic tilings)

https://plus.maths.org/content/teacher-package-maths-and-art

(toán học nghệ thuật cho giáo viên)

http://www.maths2art.co.uk/

https://www.artofmathematics.org/

http://tiasang.com.vn/Default.aspx?tabid=113&CategoryID=6&News= 9429(bài báo “Ích gì, tốn học?” GS Hà Huy Khối)

http://www.ams.org/samplings/math-and-music(trang nhạc AMS) http://im-possible.info/english/index.html(trang web với tranh không

(33)

http://thomay.vn(trang thơ máy)

https://imaginary.org(Open mathematics)

http://3d-xplormath.org/

http://virtualmathmuseum.org/

http://people.eecs.berkeley.edu/~sequin/SCULPTS/ (tượng toán học Carlo Séquin)

http://www.math.nus.edu.sg/aslaksen/teaching/math-art-arch.html

http://peinture-mathematique.fr/index.html(các tranh nghệ thuật chủ đề toán học đẹp Silvie Donmoyer)

http://www.apprendre-en-ligne.net/blog/index.php/Art-et-maths

http://www.maths-et-tiques.fr

http://dfgm.math.msu.su/myths.php(trang web có tranh Fomenko)

http://mathbun.com

http://thirddime.com/

https://plus.maths.org

http://mandelwerk.deviantart.com/

http://talesofcuriosity.com(Xem thơ limerick có minh họa Edward Lear) http://www.bl.uk/works/alices-adventures-in-wonderland(British

Library)

https://www.fulltable.com(Xem Alice in the Wonderland) http://blog.kleinproject.org/?p=1381(Lisbon)

journal.eahn.org/articles/10.5334/ah.bv/(Matthew A Cohen: Two kinds of proportions)

(34)

TÔ PÔ HỌC VÀ ỨNG DỤNG TRONG VẬT LÝ

Nguyễn Ái Việt

(Viện Công Nghệ Thông Tin, Đại Học Quốc gia Hà Nội)

TÓM TẮT

Giải thưởng Nobel Vật lý năm 2016 trao cho ba nhà vật lý lý thuyết lĩnh vực vật chất đông đặc David Thouless, Michael Kosterlitz Duncan Haldane pha tô pô vật chất chuyển pha chúng Hiện tượng chuyển pha tô pô gắn liền với việc phát vật liệu với tính chất kỳ lạ siêu dẫn khơng có điện trở siêu lỏng khơng có độ nhớt vật liệu Hall lượng tử phân số Chính vật liệu sở để chế tạo máy tính lượng tử vượt giới hạn tính tốn hệ máy tính

Ứng dụng tô pô học nào?

Tô pô học đời không gắn liền với ứng dụng thực tế Các tư tưởng ban đầu tô pô manh nha Leinitz Euler tên gọi "giải tích vị trí" "hình học vị trí" Các tốn ban đầu tơ pơ "tụ mu bn " hoc "by chic cu Kăonigsberg" mang tính giải trí nhiều mở lĩnh vực ứng dụng thực tiễn Theo khía cạnh đó, đối tượng hình học nhúng khơng gian có tơ pơ khác có tính chất khác Chẳng hạn đường cong đóng mặt cầu mặt xuyến có tính chất khác Tuy nhiên trước cơng trình tiếng "Giải tích vị trí" Henri Poincaré tính chất tơ pơ cịn mù mờ "Giải tích vị trí" có vai trị định hướng nghiên cứu lĩnh vực có ảnh hưởng ngày

(35)

Hình 1: Bánh vịng tách cà phê giống tơ pơ

Đối với nhà tốn học, ứng dụng ý tưởng trừu tượng vào thực tế việc viển vông Trong thực tế việc đưa ý tưởng tô pô vào vật lý q trình khó khăn trắc trở Tuy nhiên, điều bất ngờ loạt vật liệu có nhiều tính chất kỳ lạ đặc nhờ tính chất tơ pơ khơng gian vật lý

Các vật liệu có tính chất kỳ lạ

Trong chương trình vật lý phổ thơng, biết vật liệu dẫn điện với điện trở R Khi R = ∞vật liệu gọi chất cách điện Người ta phát loại vật liệu gọi bán dẫn, có điện trở thay đổi số điều kiện khác Vật liệu bán dẫn sử dụng để chế tạo máy tính ngày Năm 1911, nhà vật lý người Hà Lan H.Kamerlingh Onnes (Giải thưởng Nobel 1913), phát tính chất siêu dẫn thủy ngân bị làm lạnh xuống nhiệt độT = 4.2K, có điện trởR = Do đó, dịng điện

chạy vịng siêu dẫn tạo từ trường mà không lượng Đó ngun tắc cơng nghệ để tạo từ trường lớn máy gia tốc đại

Loại vật liệu có tính chất kỳ lạ thứ hai chất siêu lỏng Năm 1937, nhà vật lý Xô viết Pyotr Kapitsa (Giải thưởng Nobel 1978) phát chất helium hóa lỏng nhiệt độ T = 2.17K có tính siêu chảy, với độ nhớt khơng Nói cách trực giác tính siêu chảy sau: Nếu đổ chất siêu chảy vào ống nghiệm, chất siêu chảy tự "bò" qua thành ống hết Hình

(36)

Hình 2: Chất siêu chảy, tự động bò qua thành ống nghiệm

trạng thái tự do, tương tác với mạng tinh thể vật chất xung quanh, trở nên hút tạo thành cặp Cooper, gây nên tượng siêu dẫn Ở nhiệt độ thấp mức đó, chuyển động nhiệt không đủ lượng để phá hủy cặp Cooper, trạng thái siêu dẫn trở nên bền vững

Năm 1986, nhà vật lý công ty IBM l G.Bednorz v K.Măuller (Gii thng Nobel 1987) phát vật liệu gốm từ có tính siêu dẫn nhiệt độ caoT = 138K, tức nhiệt độ nitrogen lỏng Năm 2015, người ta tìm vật liệu có tính siêu dẫn nhiệt độT = 203K Lý thuyết chuyển pha Landau, giải thích tượng siêu dẫn nhiệt độ cao Mặc dù có số mơ hình giải thích mặt định lượng tượng siêu dẫn nhiệt độ cao Nhưng chưa có lý thuyết giải thích tượng siêu dẫn nhiệt độ cao cách thuyết phục.Có điều chắn mơ hình cho gốm từ siêu dẫn, tính chất tơ pơ khơng gian vật lý đóng vai trị quan trọng

Từ năm 1970, nhà vật lý quan tâm đến vật liệu chiều màng mỏng, vật liệu graphene màng carbon có cấu trúc tổ ong Nhờ cơng nghệ phát triển, màng đạt tới độ mỏng quy mô nguyên tử Khi phần tử mang điện electron chuyển động khơng gian hai chiều Các vật liệu có pha có tính chất kỳ lạ Chẳng hạn, năm 1980 nhà vật lý người Đức K.Von Klizing (Giải thưởng Nobel 1985) tìm thấy số vật liệu chiều nhiệt độ đủ thấp có hiệu ứng Hall lượng tử Độ dẫn điện vật liệu thay đổi theo bội số ngun lượng khơng đổi tăng cường độ từ trường ngồi đặt vng góc với vật liệu Điều kỳ lạ trạng thái với độ dẫn định tồn ổn định trọng phạm vi vào từ trường Tính ổn định liên quan tới đặc trưng tơ pô không gian vật lý, phụ thuộc vào không gian có lỗ Một số vật liệu chiều khác lại có quy luật thay đổi độ dẫn điện Hall phân số với mẫu số lẻ thí nghiệm tương tự R.Laughlin (Giải thưởng Nobel 1998) giải thích tượng cho trường hợp tử số Trong thực tế, trường hợp tử số khác quan sát Cho đến chưa có giải thích thuyết phục cho trạng thái

(37)

So với ngành tốn học khác, tơ pơ tìm thấy ứng dụng thực tế muộn màng sau đời phát triển Chúng ta tìm lý

Tô pô học vật lý

Năm 1834, Ngài John Scott Russel cưỡi ngựa dọc theo kênh đào Union (Scottland) nhìn thấy xốy nước trơi Ơng đuổi theo xốy nước tới vài dặm đặt câu hỏi: xoáy nước gì, chúng lại có hình dạng kích thước ổn định Sau xoáy nước gọi soliton lời giải ổn định phương trình Navier-Stokes, phương trình vi phân phi tuyến Các lời giải ổn định chúng có lượng cực tiểu

Sau Albert Einstein xây dựng thành công lý thuyết tương đối rộng làm tảng cho vũ trụ, ông đặt kế hoạch xây dựng lý thuyết trường thống Trong lý thuyết này, tương tác có chất hình học mơ tả phương trình vi phân phi tuyến Khi đó, người ta biết có hai tương tác hấp dẫn mơ tả phương trình Einstein tương tác điện từ mơ tả phương trình Maxwell Einstein hy vọng hạt vật chất (khi người ta biết có electron proton) mơ tả lời giải soliton phương trình phi tuyến Einstein không thực ý tưởng Cho đến ngày nay, hệ nhà vật lý toán học tiếp tục theo ý tưởng ơng để tìm "Lý thuyết Vạn vật" Trong việc sử dụng cơng cụ tô pô học quan trọng Vào thời Einstein, nhà toán học chưa hiểu mối liên quan tô pô ổn định lời giải soliton Einstein cơng cụ tơ pơ, sau hệ nhà toán học xuất sắc Chern, Atyiah, Grothendieck, Pontrijagin, phát triển vào năm 1950-1960 Một ví dụ khác nhà vật lý Tony Skyrme, cuối năm 1950 thực thành công việc đưa ý tưởng soliton vào vật lý sử dụng cách xác chất tơ pơ, mà sau người ta hiểu Ơng mơ tả lực hạt nhân cách đẹp đẽ xác, có ảnh hưởng ngày Rất tiếc cơng trình đương thời Skyrme khơng có nhiểu người hiểu chia sẻ Vào năm 1960-1970, người ta bắt đầu hiểu mối quan hệ soliton đặc trưng tô pơ đặc biệt lời giải soliton có spin bán ngun Cơng trình năm 1985 G.Adkin, C.Nappi E.Witten mơ hình Skyrme tạo nên sốt thực nhằm khai thác ý nghĩa toán học khả ứng dụng soliton vật lý Ngày nay, mơ hình Skyrme trở nên phổ biến tất lĩnh vực vật lý

(38)

Vật liệu tơ pơ máy tính lượng tử

Máy tính ngày xây dựng chủ yếu dựa tính chất vật liệu bán dẫn vật liệu từ Các vật liệu có nhiều tính chất kỳ lạ phong phú nhiều, giúp xây dựng thiết bị thông minh

Từ nghiên cứu đến công nghệ đoạn đường dài Tuy nhiên vào năm 1970 đưa ý tưởng truyền thơng tin sợi quang học, khơng hình dung ngày nay, cáp quang đến nhà với tốc độ truyền tin hàng triệu lần so với cách 20 năm Thế hệ máy tính dựa khái niệm bit lấy giá trị logic Tất thông tin xử lý máy tính đại quy phép toán với Để xử lý số lượng tính tốn khổng lồ, người ta cần phải dùng số lượng khổng lồ mạch logic vơ nhỏ Năm 2016 số mạch logic có chip điều khiển trung tâm (CPU) Intel tới số trên7.2tỷ

Rõ ràng, phải có giới hạn cho việc thiết kế nhiều mạch logic chip điều khiển Máy tính lượng tử chờ đợi bước phát triển có tính chất cách mạng dựa khái niệm qbit (bít lượng tử) lấy giá trị với xác suất khác tương ứng với trạng thái lượng tử ngun tử Do lực tính tốn, xử lý thơng tin máy tính lượng tử gần vô tận

Một vấn đề quan trọng máy tính lượng tử làm trạng thái ổn định bền vững Các trạng thái lượng tử nói chung không bền vững, hệ thức bất định Heisenberg hiệu ứng lượng tử Điều có phần giống ổn định xoáy nước mặt nước

Chính ổn định nhờ bất biến tô pô làm trạng thái lượng tử trở nên bền vững Các hệ Hall lượng tử phân số có trạng thái lượng tử bền vững ổn định nhờ bất biến tơ pơ Chính chìa khóa để giải ổn định qbit máy tính lượng tử

Gần có bước tiến đáng kể mặt này, để người ta hy vọng có đột phá việc chế tạo máy tính lượng tử Chính mà cơng trình Thouless, Kosterlitz Haldane có ảnh hưởng lớn đến phát triển tương lai nhân loại

Lời kết

Khác với ngành toán học khác, ứng dụng theo quy luật tô pô vật liệu không hình thành ngẫu nhiên tự nhiên Vật liệu sở gốm từ, hệ điện tử hai chiều, chuyển pha tơ pơ máy tính lượng tử sáng tạo người, thay Chúa điều khiển biến đổi tự nhiên

(39)

VỀ CÂU HỎI TRẮC NGHIỆM TRONG TOÁN HỌC

Terence Tao

(University of California, Los Angeles) Người dịch Phùng Hồ Hải

Trong việc nâng cấp ứng dụng câu hỏi trắc nghiệm lên định dạng đại tương tác thực hiện, nghĩ thời điểm tốt để thu thập ý kiến suy nghĩ việc câu hỏi trắc nghiệm sử dụng giảng dạy tốn học, hình thức tiềm chúng sử dụng tương lai Ý kiến câu hỏi trắc nghiệm có hạn chế đáng kể sử dụng lớp học với mơ hình truyền thống, có nhiều tiềm thú vị chưa khai thác sử dụng công cụ tự đánh giá

1 Câu hỏi trắc nghiệm lớp học

Về nguyên tắc, chất rõ ràng xác mệnh đề tốn học có thiên hướng cho phương thức trắc nghiệm, trái ngược với số lĩnh vực tri thức khác, nhiều câu hỏi tốn học có câu trả lời xác nhất, khách quan, với tất câu trả lời khác đồng thuận coi khơng xác Với kiểm tra trắc nghiệm, học sinh thử nghiệm câu hỏi cách khách quan Thực vậy, chấm điểm cho câu đố chí tự động thực máy tính quét máy Miễn câu hỏi phát biểu cách rõ ràng (và đáp án xác), việc chấm điểm đơn giản phương tiện kiểm tra khác Điểm mạnh cuối là, hình thức thi trắc nghiệm quen thuộc với tất sinh viên đại học (những người phải vượt qua kỳ thi trắc nghiệm để nhập học) quy tắc kiểm tra địi hỏi lời giải thích

(40)

trong lựa chọn.) Một vấn đề câu hỏi trắc nghiệm dễ bị số loại gian lận tiêu cực hình thức kiểm tra khác, đáp án dễ dàng chép sử dụng, chí học sinh không thực hiểu tài liệu (Vấn đề cụ thể phần bảo vệ cách xáo trộn câu hỏi riêng cho học sinh, điều tất nhiên làm việc chấm việc cung cấp đáp án khó khăn hơn.) Một vấn đề thứ ba học sinh thu câu trả lời không nằm số lựa chọn liệt kê có khuynh hướng làm bừa, nhiều lý luận phi logic để đến câu trả lời liệt kê, khơng phải thói quen tốt để thấm nhuần vào nhà toán học

Tuy nhiên, vấn đề sâu sắc hơn, câu trắc nghiệm cho ấn tượng sai lệch việc giải toán, làm để thực điều Trong nghiên cứu tốn học, câu hỏi không thường kèm với danh sách năm phương án, số xác Thơng thường, hình dung câu trả lời tiềm năng, có lý, nhiều khả xảy ra, chí kiểu câu trả lời mong đợi hay đặt vấn đề liệu có nên hỏi câu hỏi đó, quan trọng khơng việc xác định câu trả lời Câu hỏi trắc nghiệm có xu hướng khuyến khích cho cách tiếp cận nhanh-chóng-và-khơng-lành-mạnh cẩu thả để giải vấn đề, trái ngược với cách tiếp cận thận trọng, cân nhắc, tinh tế Đặc biệt, câu hỏi có xu hướng khuyến khích việc áp dụng ngun si quy tắc hình thức để đến câu trả lời, mà không dành nhiều suy nghĩ việc liệu quy định thực áp dụng vấn đề xét hay không (Thực ra, đào sâu mức đề trắc nghiệm, tìm kiếm mẹo mực, chỗ thiếu chặt chẽ, đặc biệt cách diễn đạt câu hỏi, cố gắng để chơi số loại “luật chơi”, cố gắng thần thánh hóa mục đích người đề [xem cảnh từ “The Princess Bride” cho ví dụ cực đoan này], làm cho sinh viên hơn, hiểu nội dung kiến thức, lại có kết tồi tệ so với người đơn giản việc áp dụng quy tắc mà họ dạy mà khơng có hiểu nội dung kiến thức Ngược lại, đề mẹo, thiết kế để bẫy học sinh áp dụng quy tắc cách cẩu thả, khơng kiểm tra xem có áp dụng không, thường cảm nhận (khá đúng) không công với học sinh.) Trong việc luyện tập quy tắc (ví dụ quy tắc thuật tốn mơn Giải tích) chắn cần thiết, đặc biệt cấp trung học giai đoạn đầu đại học mơn tốn, thời điểm chuyển lên giai đoạn cao bậc đại học sinh viên cần bắt đầu hiểu sở lý thuyết giải thích cho quy tắc đó, phần việc phát triển tư mơn học (Ngồi ra, học mơn nâng cao, có nhiều ngoại lệ điểm yếu quy tắc kiến việc áp dụng cách khơng suy nghĩ trở nên nguy hiểm Ví dụ, tính tốn tích phân đường cách tịnh tiến đường dễ cho câu trả lời sai khơng có cảm giác tốt tích phân hội tụ tới không chuyển qua giới hạn, thi không hội tụ Cách học thuộc số quy tắc dễ nhớ tích phân an tồn khơng, thất bại có nhiều biến thể khác nhau, đặc biệt ứng dụng thực tế, cách đáng tin cậy để tiến hành để thực hiểu cách ước lượng tích phân tính toán giới hạn)

(41)

pháp cho ta nhiều thông tin vấn đề Việc xác định điểm mạnh điểm yếu cụ thể lý luận học sinh cho nhiều phản hồi có giá trị trình chấm điểm so với đơn giản biết xem câu hỏi đưa trả lời cách xác khơng xác

2 Trắc nghiệm hình thức tự kiểm tra

Tôi thảo luận hạn chế việc sử dụng câu hỏi trắc nghiệm kỳ thi lớp học, đặc biệt khóa học tốn học năm cuối đại học Mặt khác, cảm thấy câu hỏi đóng vai trị hỗ trợ hữu ích việc tự kiểm tra cho khóa học này, đặc biệt liên quan đến nội dung kiến thức (ví dụ định nghĩa quy tắc tính tốn) Tơi chứng minh điều với khóa học giả định đại số trung học phổ thông, điểm chắn áp dụng cho nhiều khóa học tốn học câp cao

Giả sử khóa đại số nhằm dạy cho học sinh cách giải phương trình đại số Tất nhiên có nhiều cạm bẫy thông thường mà học sinh gặp phải thực cố gắng để giải phương trình đó, ví dụ phổ biến là, từ phương trình x2 D y kết luận sai rằng

x Dpy, thay nói xD pyhoặcx D py Bây giờ, ta cảnh báo lỗi

các lớp học, học sinh chí viết cảnh báo ghi chú, lặp lại thường xuyên giải toán đại số phức tạp thi (hoặc tệ hơn, ứng dụng thực tế đại số) Khi đó, sinh viên nhận nguyên nhân lỗi - phản hồi đến sau nhiều ngày nhiều tuần từ lần đầu tiên, không nhắc nhắc lại, lỗi tương tự tái phát sau khóa học, khóa học Lặp lặp lại tiếp xúc với đại số cuối loại bỏ lỗi, trình khơng hiệu Đây nơi mà lựa chọn nhiều trắc nghiệm tự làm (đặc biệt, kiểm tra trực tuyến) giúp đỡ, với câu hỏi như:

Câu hỏi Nếuxylà số thực thỏax2 Dy, điều nói vềx

a) x Dpy:

b) x Dy2:

c) x Dy 2:

d) x DpyhoặcxD py:

e) x Dy hoặcx D y 2:

hoặc pha trộn với với biến thể

Câu hỏi Cho x y là số thực Khẳng định sau không đủ để ngụ ý rằng x2 Dy?

(42)

b) x D py:

c) x D CpyhoặcxD py.

d) x Dy2:

e) y Dx2:

Câu hỏi Nếuxylà số thực chox3 Dy;khi tốt nói vềx

a) x Dpy:

b) x Dy13:

c) x D Cy13 hoặcx D y 3:

d) x Dy 3:

e) x Dy hoặcx D y 3:

Những câu hỏi có cho biết trực tiếp (và có thơng tin phản hồi lập tức) học sinh có mắc sai lầm vấn đề cụ thể không, mà không cần can thiệp trực tiếp giảng viên giảng dạy trợ (Một cách lý tưởng, kiểm tra tự động câu trả lời lựa chọn hay sai, mà cịn để giải thích lỗi trường hợp này)

Lưu ý số khác biệt loại câu hỏi trắc nghiệm câu hỏi kiểm tra lớp Trong khung cảnh kỳ thi, người ta thường muốn có câu hỏi phức tạp mà kiểm tra số khía cạnh kiến thức lúc (ví dụ phân tích nhân tử, rút gọn, thế, ) thay tập trung cách hẹp đơn giản lên khía cạnh (Đặc biệt, sinh viên thực nắm kiến thức trả lời câu hỏi dễ dàng, mà khơng cần phải tính tốn đáng kể.) Ngoài ra, kiểm tra lớp học cố gắng làm cho câu trả lời xác khác biệt so với lựa chọn thay khơng xác (để phân biệt người hiểu kiến thức với người thực khơng biết gì), việc tự kiểm tra cho phép khác biệt tinh tế câu trả lời câu trả lời khác, để khuyến khích học sinh suy nghĩ cẩn thận để giải quan niệm sai lầm đầu vào, loại “câu hỏi lừa” không công môi trường căng thẳng kỳ thi đánh giá lớp, thực cách an toàn đề tự kiểm tra Câu hỏi trắc nghiệm dường có hiệu dùng để giải thích định nghĩa xác khái niệm quan trọng (“với mỗi"tồn mộtı” hay “cho mỗiıtồn một"?”), việc xây dựng xác số quy

tắc (đạo hàm f g

fg0 gf0 g2 hay

f0g gf0 g2 hay

f0g gf0

f2 ; ?), kiểm tra trực tiếp

lỗi cụ thể thường thực (nếux < y, x < y, hay x > y?) Xem thêm danh

(43)

Câu hỏi Chof W R ! Clà hàm số Trong số tất giả thiết đây, đâu giả

thiết yếu mà cho phép biến đổi FourierfOWR!Ctồn liên tục?

a) f trơn giảm nhanh chóng.

b) f là hồn tồn khả tích.

c) f là bình phương khả tích.

d) f là liên tục.

e) f là liên tục có giá compact.

f) f là phân phối tăng chậm.

Các loại kiến thức giải tích Fourier mà câu hỏi kiểm tra khó kiểm tra kiểu câu hỏi khác (trừ thi vấn đáp) Một khả thú vị khác sử dụng trắc nghiệm để khám phá chiến lược giải toán, vấn đề gián tiếp giải hầu hết phương pháp kiểm tra Ví dụ, khóa học giải tích biến, người ta tập trung vào chiến thuật tích hợp, sử dụng câu hỏi này:

Câu hỏi Kỹ thuật sau bạn cảm thấy bước để tìm nguyên hàm

R

x2log.1Cx/dxcủa hàmx2log.1Cx/?

a) Tích phân phần, đạo hàmx2và tích phânlog.1Cx/.

b) Tích phân phàn, đạo hàmlog.1Cx/và tích phânx2.

c) Thếy Dx2:

d) Thếy D1Cx:

e) Thếy Dlog.1Cx/:

f) Thử đạo hàm hàm sốx3log.1Cx/:

g) Phác thảo đồ thị củax2log.1Cx/:

h) Khai triển Taylorlog.1Cx/:

i) Khởi động Maple, Mathematica, SAGE.

(44)

câu hỏi câu truyền thống “Tính nguyên hàm củax2log.1Cx/.” Sự nhấn mạnh

đây chiến thuật tính tốn

Tóm lại, tơi tin có số cách thú vị nhiều số chưa khai thác -trong câu tự trắc nghiệm thiết kế tốt trực tuyến có hiệu để đánh giá điểm mạnh điểm yếu người mơn tốn Tất nhiên, có tương tác một-một với giảng viên trợ giảng một-một cách thích hợp để đạt điều loại thông tin phản hồi lập tức, điều không thực tế cho lớp học lớn Ngoài mức độ trưởng thành kỷ luật định cần thiết học sinh để thực hưởng lợi từ phương thức tự đánh giá này, đặc biệt chúng khơng có ảnh hưởng trực tiếp tới điểm số lớp học sinh, triết lý sinh viên hưởng lợi từ nghi, cảm thấy khả thực mức tối thiểu để thi đỗ phần khóa học năm cuối đại học cần hướng tới

(45)

NƯỚC MỸ CHỌN VÀ LUYỆN ĐỘI TUYỂN THI TOÁN

QUỐC TẾ (IMO) THẾ NÀO?

Lê Tự Quốc Thắng

(Học viện Công Nghệ Georgia, Mỹ)

GIỚI THIỆU

Bài viết giới thiệu cách Mỹ chọn luyện đội tuyển IMO Để viết này, tham khảo tài liệu Internet, vấn huấn luyện viên đội tuyển Mỹ, thành viên đội tuyển IMO nhiều nước giới

Mỹ nước khơng có hệ thống trường chuyên lớp chọn Nga, Trung Quốc, hay Việt Nam, Mỹ ln chiếm vị trí cao kỳ thi vơ địch tốn quốc tế (IMO), hai năm2015; 2016đã đứng đồng đội

Trước hết xin mạnh điểm:Độc giả đừng nhầm lẫn việc tuyển chọn huấn luyện cho kỳ thi (Toán, Lý, Tin học, ) với việc đào tạo học sinh có khiếu Mỹ, Mỹ hai vấn đề này, dù có liên quan với nhau, khác xa, khơng Việt Nam Mục đích đào tạo học sinh giỏi Mỹ để phát triển hết khả cho học sinh có khiếu chứ không nhằm vào việc thi kỳ thi Olympiad

Tôi viết việc đào tạo học sinh khiếu Mỹ khác Bài viết tập trung vào nội dung nêu tiêu đề

Ở Mỹ học sinh giỏi toán biết/muốn thi toán quốc tế Văn hóa Mỹ khơng đánh giá q cao tài toán học Các nghiên cứu nhiều học sinh đội tuyển Mỹ kỳ thi học sinh nhập cư người nhập cư từ nước mà giáo dục tốn học coi trọng tài tốn học ni dưỡng thơng qua quy trình khó khăn kiên trì “Có thể nói văn hóa Mỹ ngày dường như khơng khuyến khích nam giới phụ nữ tốn học” - Michael Sipser, trưởng khoa Tốn Học viện cơng nghệ MIT nói

Đúng Mỹ khơng có trường chuyên với mục đích luyện thi học sinh giỏi cho kỳ thi Olympiad Nhưng Mỹ có số trường chuyên, câu lạc toán để đào tạo học sinh khiếu/giỏi Các chương trình đào tạo học sinh giỏi thường trường đại học hỗ trợ, học sinh IMO đến từ khắp miền nước Mỹ Có học sinh học nhà (home schooling) cha mẹ dạy, khơng đến trường Thí dụ Reid Barton, người thi4lần huy chương vàng cả4, home schooling

(46)

của Mỹ tổ chức kỹ, có chiến lược, ngày phức tạp kỹ Tơi có hỏi trưởng đồn thành viên đội tuyển số nước có thành tích cao thi tốn quốc tế, tất cho để có thành tích cao, cần có chương trình huấn luyện kỹ, thời kỳ3 tuần trước thi IMO

Và Mỹ từ đầu huấn luyện kỹ đội tuyển Một số nước khơng có chương trình chọn lựa kỹ (qua vịng thi tuyển) khơng có luyện tập, thường khơng có kết cao Điển hình đội Italia, năm1984;cả6thành viên đội tuyển Italia tổng cộng 0điểm Ngoại lệ có lẽ nước Anh Trong10 15năm đầu tiên, nước Anh không huấn luyện đội tuyển trước thi, đội tuyển Anh thành công N Boston, thành viên đội tuyển Anh năm 1979và giáo sư toán đại học Wisconsin, nói với tơi rằng, Anh có truyền thống coi trọng việc giải tốn nhanh trường phổ thơng, có nhiều câu lạc tốn trường

Việc chọn luyện đội IMO Mỹ thực MAA, tổ chức phi phủ Nhà nước Mỹ hay Bộ giáo dục Mỹ không nhúng tay vào việc này, lúc phát giải thưởng kỳ thi USAMO (vô địch tốn Mỹ) trợ lý tổng thống Mỹ khoa học cơng nghệ thường có mặt đọc phát biểu Chi phí cho đội tuyển, tập trung luyện, thi, chủ yếu từ MAA nhà tài trợ NSF (quỹ nghiên cứu khoa học Mỹ) có đóng góp

IMO khởi xướng nước Đông Âu (khối XHCN cũ) từ năm1959:Nước tư tham gia Phần Lan, năm 1965;và xếp chót bảng (Phần Lan nghỉ ln8năm, đến năm 1973 tham gia lại) Pháp tham gia lần đầu vào năm 1967; Hà Lan lần đầu năm 1969;và xếp chót bảng M Klamkin, huấn luyện đội tuyển IMO Mỹ năm 1977 1984 nói nhiều người Mỹ “khơng muốn Mỹ tham gia IMO, sợ đội Mỹ bị nước cộng sản đè bẹp.” Năm 1971N Turner viết báo tạp chí “Amer Math Monthly” nói Mỹ có số kỳ thi tốn cấp tiểu bang, nên gửi đội tuyển tham gia IMO, lúc đầu bị “nhục” thứ hạng thấp, tình hình cải thiện Năm 1972Mỹ lần đầu tổ chức thi vơ địch tốn nước Mỹ (USAMO) năm1974MAA đồng ý gửi đội tuyển Mỹ tham gia IMO Trong nước tư Mỹ nước thành cơng thi IMO, có thứ hạng trung bình 1995 2008là3(Việt Nam thứ hạng trung bình là6) Đặc biệt năm1994cả6thành viên đội tuyển Mỹ điểm tuyệt đối, kỷ lục mà chưa nước làm (Điều tương phản với thành tích trung bình đánh giá trình độ tốn học sinh Mỹ nói chung) Để có thành cơng nhờ việc chọn lựa chuẩn bị cho đội tuyển IMO Mỹ kỹ

(47)

Khoảng

4 số thành viên đội tuyển IMO Mỹ sau trở thành nhà toán học Khoảng số thành viên đội tuyển IMO Mỹ dân nhập cư dân nhập cư vào Mỹ từ nước có thành tích cao IMO (theo hội tốn học Mỹ, có người gốc Việt) Để huấn luyện cho đội tuyển, MAA lập MOP (Mathemetical Olympiad Program, sau MOSP), khóa huấn luyện khơng cho thành viên đội tuyển IMO Mỹ mà tạo nguồn cho đội tuyển tương lai Thường MOP kéo dài2 4tuần, trước thi, đội tuyển đến địa điểm thi (ở nước đó) trực tiếp từ MOP

Quy trình chọn đội tuyển IMO Mỹ thay đổi nhiều lần Vào thời điểm phức tạp bạn nghĩ Tuy nhiên việc “luyện” đội tuyển Mỹ không nhiều số nước Trung Quốc, Nga hay Việt Nam Những trường chuyên nước cho học sinh làm loại tốn Olympiad quanh năm Làm nhiều q có lẽ có hại cho sáng tạo (một số) học sinh sau

Ở Mỹ có3kỳ thi tốn Kỳ AMC12, học sinh trung học muốn tham gia được, phải đóng khoản lệ phí Những học sinh đạt điểm cao mời tham dự vòng AIME Rồi khoảng gần300 học sinh điểm cao AIMECAMC 12được tham gia kỳ thi vơ địch tốn nước Mỹ (USAMO) Cả2vịng sau miễn phí

Trước đậy,6 8người điểm cao USAMO thành viên đội tuyển IMO Mỹ Sau USAMO, thành viên đội tuyển Mỹ, khoảng 25 35học sinh điểm cao USAMO học lớp12, mời đến MOP (hoặc sau MOSP) Mục đích ngồi huấn luyện đội tuyển cịn đào tạo hệ tương lai chuẩn bị cho IMO năm sau Rất nhiều thành viên đội tuyển tham dự MOSP năm trước, chí khơng lần MOP tổ chức năm1974;năm Mỹ bắt đầu tham gia IMO Ngay lần đầu MOP “đào tạo” cho số thành viên đội tuyển năm sau

Về sau, đội tuyển IMO không chọn dựa vào kết USAMO Sau khoảng 40 người điểm cao USAMO tập trung MOSP, có2 3bài thi chọn đội tuyển Kết thi chọn đội tuyển điểm USAMO dùng để chọn đội tuyển Việc tuyển chọn xác đội tuyển chọn khoảng tuần trước IMO, gây số phiền phức cho đội tuyển cơng việc hành làm giấy tờ, visa Đến năm2011;công thức chọn đội tuyển IMO lại thay đổi Lần việc chọn đội tuyển năm trước thi IMO, gồm bước sau Để vào đội tuyển năm sau, bạn phải chọn vào MOSP năm trước

1 Đầu tiên là2 3bài sơ tuyển diễn suốt Chương trình Tốn học mùa hè (MOSP) tháng6năm trước Khoảng40người qua vòng tham gia thi tuyển mục2và mục3dưới

2 Hai thi chọn đội tuyển vào mùa đông (tháng12, tháng1)

(48)

Kỳ thi định tham gia kỳ 3: AMC12C AIME định tham gia USAMO, sau đó2C3CUSAMO định đội tuyển Vì thế, đội tuyển định sau USAMO diễn khoảng cuối tháng 4:Đến tháng 6các thành viên đội tuyển khoảng 50 ứng cử viên cho đội tuyển năm sau tập trung luyện chuẩn bị thi toán quốc tế (MOSP)

Ban tổ chức MOSP MAA (Hiệp hội Toán học Mỹ) Các thành viên tham gia MOSP tài trợ hồn tồn chi phí lại ăn trong3 4tuần Phần lớn chi phí chi trả MAA nhà tài trợ Thí dụ AKAMAI nhà tài trợ lớn – công ty giúp giúp MOSP tăng gấp đôi số học sinh Số tiền nhận từ phủ Mỹ khiêm tốn

Được nhận vào MOSP khó, vi dự học sinh phổ thơng Mỹ Một chương trình khác có uy tín cao Research Science Institute (RSI), tổ chức6tuần hàng năm MIT RSI thiên nghiên cứu, không thi tuyển chọn mà xét hồ sơ tuyển chọn Khoảng 80 học sinh cấp đến từ khắp nơi giới (50 từ Mỹ và30 học sinh nước ngoài) chọn tham gia RSI (miễn phí hồn tồn) Rất nhiều cơng trình nghiên cứu thành viên RSI giải thưởng cao thi Intel Sience Talent Search MOSP RSI có cạnh tranh với nhau, tranh giành học sinh giỏi Mỹ toán

Các giảng viên MOSP Có 2loại chức danh: Giảng viên (hay cịn gọi huấn luyện viên) trợ giảng Hầu hết giảng viên trợ giảng, trừ số ngoại lệ Titu Andreescu, Zuming Feng Razvan Gelca Những người Huấn luyện viên trưởng (hiện giáo sư Po-Shen Loh tới từ ĐH Carnegie-Mellon) mời đến làm việc Họ thường cựu thành viên đội tuyển IMO Mỹ quốc gia khác, thí sinh tham gia MOSP, hầu hết trẻ

Các giảng viên phần lớn giáo sư đại học, nghiên cứu viên hậu tiến sĩ nghiên cứu sinh Trợ giảng thường sinh viên đại học Họ đến từ trường MIT, Carnegie Mellon, Harvard, Berkeley Có số giảng viên từ công ty tư nhân (như Jane Street Capital, Microsoft) Rất giáo viên phổ thơng Tổng cộng có khoảng15giảng viên và7trợ giảng Nhưng khơng phải đủ3 4tuần MOSP

Cuối cùng, để có góc nhìn đầy đủ “bếp núc” MOSP, chương trình huấn luyện đội tuyển Mỹ, tơi xin giới thiệu phần trả lời Razvan Gelca, huấn luyện viên kỳ cựu MOSP cho câu hỏi mà đặt cho anh Một vài câu câu hỏi đề xuất Trần Nam Dũng, giảng viên trường Đại học Khoa Học Tự Nhiên thành phố Hồ Chí Minh, tên quen thuộc giới Olympiad toán Việt Nam Trong câu trả lời đây, đại từ “tôi” nghĩa Razvan Gelca

1:Một ngày bình thường MOSP nào? Có3kiểu ngày MOSP:

1 Có giảng vào buổi sáng, 90 phút giảng 90 phút vào buổi chiều Sau đó, học sinh hoạt động tự do, nghỉ ngơi, làm tốn có hoạt động chung Cũng có báo cáo khoa học vào buổi tối dành cho giảng viên trợ giảng Học sinh tham gia không tùy ý

(49)

3 Các ngày thứ Bảy có số kiểm tra vào buổi sáng, thi sơ tuyển thi thử IMO, buổi chiều thứ bảy Chủ nhật nghỉ ngơi

Kiểu ngày số1và số2xen kẽ tuần, có nhiều kiểm tra

2:Hình thức giảng?

Tơi nói với bạn cách mà điều hành học Tôi tập trung nhiều vào toán, tập trung vào lý thuyết Tơi bắt đầu việc đưa cho em danh sách tốn, tơi cho họ làm việc khoảng45phút, sau chúng tơi viết thảo luận lên bảng Những giảng viên khác tập trung nhiều vào lý thuyết

3:Các cấp độ giảng dạy?

Học sinh chia thành nhóm tùy theo khả mình:

A: Nhóm dai đen: Đây đội tuyển thức Mỹ, thêm người giỏi số gái tham dự EGMO (Olympiad Tốn học dành cho nữ sinh châu Âu), thành viên đội tuyển Canada có2quốc tịch Đây nơi mà tất người phải tập trung vào tốn, khơng nhiều lý thuyết Trình độ khả cao Chỉ cần đưa cho họ toán, họ lên bảng cho lời giải tuyệt vời

B: Nhóm đai xanh: Nhóm gần với nhóm đai đen, tập trung chủ yếu vào tốn Nhưng có dạy cho họ số lý thuyết Với số toán dành cho nhóm đai đen, nhóm cần gợi ý cách giải

C: Nhóm đai đỏ: Đây tân binh Ở nhóm này, lý thuyết dạy trước, sau tốn Dù tơi thường chọn tốn mà học sinh giải được, ln có nhóm nhỏ khơng theo kịp

4:Có dạng tốn cao cấp dạy MOSP khơng?

Có số giảng tốn cao cấp vào cuối ngày Tôi dạy lý thuyết Chern - Simons: Topo học, hàm theta, học lượng tử, cố gắng sử dụng từ đơn giản để truyền tải ý Nhưng làm thứ cao cấp lớp học

Bài giảng tài sản riêng giảng viên Các giảng viên làm việc độc lập với Tôi người khác làm gì, ngoại trừ tơi có ghé thăm lớp họ, cho vui thơi Trình độ học sinh ngày cao, tơi ln phải tìm tốn mới, khó

5:Thành viên đội tuyển IMO Mỹ có nhận giải thưởng từ Chính phủ

hay tổ chức khơng? Các thí sinh sau thường theo học trường nào?

(50)

thích Carnegie - Mellon đối thủ lớn kỳ thi Putnam Ngay UCLA trao học bổng để thu hút sinh viên, họ thu hút số sinh viên nước Trước đây, Duke trao học bổng, họ khơng cịn thu hút học sinh Princeton đứng sau Harvard, MIT, CMU việc thu hút người Olympiad

Tơi muốn nói có nhiều người trở thành nhà tốn học, tất Đáng tiếc nhiều người cuối lại làm việc cho quỹ đầu (tiền nhiều tương lai mù mịt) Tôi không rõ số cụ thể

6:Có trường chuyên biệt Mỹ giống trường Nga dạy tốn mức độ cao khơng?

Hay có chương trình đào tạo tập trung vào vấn đề tốn học phục vụ riêng Olympiad khơng? Có trường Mỹ (hay địa phương nào) có truyền thống lâu đời việc giành giải cao USAMO không?

Có trường Phillips Exeter, Thomas Jefferson số trường khác chuyên cung cấp nhiều học sinh giỏi, Mỹ khác nước Nga Có nhiều sở giáo dục, chương trình đào tạo tư nhân phi lợi nhuận chí cịn đóng góp nhiều Tơi ủng hộ tham gia vào số chương trình Và tơi tin chúng định hình lại mặt giáo dục đất nước Đây số chương trình: Nghệ thuật giải vấn đề (chương trình đào tạo trực tuyến), Toán học tuyệt vời (trường hè), Toán học lý tưởng (trường hè)

Có nhiều câu lạc tốn học, bật câu lạc toán học Bay Area chương trình dành cho học sinh tài Johns Hopkins Và cịn có nhiều chương trình khác New England, East Coast (New York, New Jersey, North Carolina), Bay Area, San Diego, Dallas Tuy nhiên có học sinh khắp nước Mỹ tự học vào MOP

7: IMO mà Mỹ tham gia vào năm 1974; cũng năm mà đội tuyển Mỹ thành

cơng Vậy việc ơn luyện cho MOSP thời điểm có nặng nề khơng? Bạn so sánh MOSP và việc tuyển chọn lúc đó?

(51)

LUẬN LÝ VỚI THÌ Trần Thanh Hải (Vienna)

TĨM TẮT

Luận lý với (temporal logics đơi cịn gọi tense logics) phát triển nhà triết học để nghiên cứu yếu tố thời gian lập luận Hiện tại, nhiều loại luận lý với phát triển chúng thường phân biệt dựa việc mô tả cấu trúc yếu tố thời gian: nhánh tuyến tính Trong phần này, chúng tơi giới thiệu LTL - loại loại luận lý với thuộc nhóm tuyến tính LTL thường dùng để mơ tả kiểm chứng chương trình máy tính Giá trị công thức LTL xác định dựa cấu trúcKripke

1 Cấu trúc Kripke

Ở đây, giả sử người đọc quen thuộc vớiluận lý bậc nhất(gồm phép toán∧,∨, , hai vị từ

với mọi∀và tồn tại∃) GọiAP tập hợp mệnh đề luận lý bậc Cấu trúcKripkecũng xem đồ thị chuyển trạng thái có gán nhẵn mô tả bộK = (S, S0, R, L)[2], với

• Slà tập hợp nốt haytrạng thái,

• S0⊆Slà tập hợp cáctrạng thái bắt u,

ã Rl mtquan hton phn bờn trỏi trờnSìS, (nói cách khác, với trạng tháis∈S, tồn trạng

tháis0∈Ssao choR(s, s0)),

• Llà mộthàm gán nhãnL:S→2AP gắn trạng tháis∈Smột tậpL(s)các mệnh đề trongAP

và mệnh đề xem ln ln trongs1

VìRlà quan hệ tồn phần bên trái, ln ln xây dựng đường đivô hạnπdựa cấu trúc Kripke cho sẵn Mộtđường đitrongK chuỗi có thứ tự trạng tháiπ=s0, s1, cho

với mọii≥0, R(si, si+1)luôn tồn Chúng ta dùng ký hiệuπ(i)để mô tả trạng thái thứitrên đường điπvà

πiđể đường trạng thái thứi, nói cách khácπi= (si, si

+1, )

2 Biểu diễn chương trình với luận lý

GọiV ={v1, , vn}là tập hợp cácbiếncủa hệ thống Chúng ta giả sử biến trongV gán

một giá trị từ tập hữu hạnD2 Mộtphép lượng giátrênV hàm gán giá trị trongDcho biến

trongV

Mộttrạng tháicủa hệ thống mơ tả giá trị gán cho biến trongV, nói cách khác,

một trạng thái hiểu làm hàms:V →D Giả sử hệ thống có hai đồng hồhr1và

hr2, hayV = {hr1, hr2}, phép lượng sauhhr1= 5, hr2= 10i, biểu diện trạng

thái tương ứng cơng thức(hr1= 5)∧(hr2= 10) Ngồi ra, hình dung cơng thức luận lý

bậc nhấtfnhư tập hợp phép lượng giá chofcó giá trị đúng, biểu diễn tập trạng thái công thức luận lý bậc

1Trong số trường hợpAPcó thể tập hợp mệnh đề loại luận lý khác

(52)

Algorithm 1Hai đồng hồ

1: procedureHOẠT ĐỘNG 2: hr1∈Int

3: hr2∈Int

4: khởi tạo:

5: hr1←1

6: hr2←6

7: lặp:

8: whiletruedo

9: hr1= (hr1+ 1) mod 12

10: hr2= (hr2+ 1) mod 12

Trong ví dụ “Hai đồng hồ”, thấy ban đầu hai đồng hồhr1, hr2lần lượt có giá trị Sau đó,

chúng có giá trịh2,7i,h3,8i Với cách biểu diễn mô tả trên, trạng thái khởi tạo hệ thống

sẽ biểu diễn công thức(hr1= 1)∧(hr2= 6) Và để biểu diễn bước bước chuyển trạng thái, ví

dụ từh1,6isangh2,7i, cần thêm số khái niệm khác Chúng ta tạo tập biến, ký hiệu làV0 Chúng ta hiệuV là tập biến trạng tháihiện tạivàV0là tập hợp biến trạng

thái Mỗi biếnvtrongV có biến tương ứngv0 trongV0 Một phép lương giá (có thứ tự) trênV

vàV0 được xem cặp (có thứ tự) hai trạng thái hay gọi mộtbước chuyển trạng thái Trọng ví đụ

dụ “Hai đồng hồ”, biểu diễn bước chuyển trạng thái công thức(hr1= 1)∧(hr2= 6)∧

(hr0

1= 2)∧(hr20 = 7) Tập hợp cặp trạng thái gọi mộtquan hệ trạng thái NếuRlà quan hệ

trạng thái, đùng ký hiệuR(V, V0)để công thức luận lý bậc tương ứng Trong ví dụ trên, ta có

R(V, V0) = (hr0

1= (hr1+ 1) mod 12)∧(hr02= (hr2+ 1) mod 12)

Để mơ tả tính chất hệ thống, cần định nghĩa tập hợp mệnh đề bảnAP Ở đây, mệnh đề có dạngv=dvớivlà biến trongV vàdlà giá trị trongD

3 Định nghĩa LTL

Chúng ta thấy cách biểu diễn chương trình máy tính dùng luận lý bậc Tuy nhiên bất tiện dùng luận lý bậc để mơ tả số tính chất gắn với yếu tố thời gian hệ thống [2,4] Ví dụhr1khơng nhận giá trị 13 đèn giao thông từ màu xanh chuyển sang màu đỏ lập

tức Để việc mô tả số tính chất hệ thống thuận tiện, LTL phát triển Sau đây, tìm hiểu định nghĩa LTL [4]

3.1 Ngữ pháp

Các biểu thức LTL xây dựng dựa luật sau:

ϕ::=> |p|ϕ∨ϕ| ¬ϕ|Xϕ|ϕUϕ

với

• plà mệnh đề trongAP

XandUlà nhữngliên kết với thìvà gọitiếp theotới khi

Các liên kết khác định nghĩa tương tự luận lý cổ điển Ngoài ra, định nghĩa thêm hai liên kếtsau lúcFluôn luônGnhư sau

Fϕ=>Uϕ

(53)

3.2 Ngữ nghĩa

Theo ngữ nghĩa tự nhiên, liên kếtX,U,FGcó thể hiểu sau:

Xϕ:ϕsẽ trạng thái kế tiếp,

• ϕ1Uϕ2: nếuϕ2đúng thời điểm thìϕ1sẽ từ trạng thái trạng thái thời

điểm đó,

Fϕ:ϕsẽ thời điểm tương lai,

Gϕ:ϕln lng

Chính xác hơn, giá trị mệnh đềϕtrong LTL xác định dựa cấu trúc Kripke đường K, π(πlà đường trênKnhư sau:

• K, π>ln ln đúng,

• K, π⊥ln ln sai,

• K, πpkhi khip∈L(π0), nói cách khác, mệnh đề bảnpchỉ trênK, πkhi

hàm gán nhãnLgắnpvới phần tử u tiờn(0)ca ng i,

ã K, ơkhi v ch ifM, π2ϕ

• K, πϕ∨ψkhi khiK, πϕ∨K, πψ

• K, πXϕkhi khiK, π(1)Xϕ

• K, πϕUψkhi khi∃i.(K, π(i)ψ)∧(∀j < i.(K, π(j)ϕ))

• K, πFϕkhi khi∃i.K, π(i)ϕ

• K, πGϕkhi khi∀i.K, π(i)ϕ

4 Ứng dụng LTL

4.1 Mơ tả tính chất chương trình máy tính

Dễ thấy LTL khơng thể mơ tả tính chất hệ thống Tuy nhiên, may mắn LTL đủ hữu dụng để mơ tả hai lớp tính chất quan trọng chương trình máy tính [2,4]:

Tính an tồn: điều xấu không xảy suốt trình chạy chương trình,

Tính sống sót: điều tốt sau lúc xảy

Những trạng thái vi phạm tính chất mong đợi gọi nhữngtrạng thái xấu

Ví dụ 4.1, 4.3 4.2 mô tả cách dùng LTL để biểu diễn số tính chất chương trình Hai ví dụ tính an tồn ví dụ thứ ba tính sống sót

Ví dụ 4.1 Hai chương trìnhAandBkhơng thể trạng thái quan trọng (critical) lúc.

G(¬inCSA∨ ¬inCSB)

Ví dụ 4.2 Đèn giao thơng khơng thể từ màu xanh chuyển sang màu đỏ lập tức.

(54)

4.2 Kiểm lỗi hệ thống

Dựa phương pháp biểu diễn hệ thống công thức luận lý với thì, nhà khoa học máy tính phát triển phương pháp tự động kiểm lỗi hệ thống dựa mơ hình (model checking) Hiện tại, tảng hỗ trợ viết đặc tả kiểm lỗi tự động TLA+, Spin, nuSMV .đều cho phép người dùng sử dụng LTL [3,4,1].

Chúng ta bàn tới phương pháp tảng phần sau

Tài liệu

[1] Alessandro Cimatti, Edmund Clarke, Enrico Giunchiglia, Fausto Giunchiglia, Marco Pistore, Marco Roveri, Roberto Sebastiani, and Armando Tacchella Nusmv 2: An opensource tool for symbolic model checking In

International Conference on Computer Aided Verification, pages 359–364 Springer, 2002 [2] Edmund M Clarke, Orna Grumberg, and Doron Peled.Model checking MIT press, 1999

[3] Gerard J Holzmann The model checker spin IEEE Transactions on software engineering, 23(5):279, 1997 [4] Leslie Lamport Specifying systems: the TLA+ language and tools for hardware and software engineers

(55)

CÁC PHƯƠNG PHÁP SAI PHÂN HỮU HẠN CHO PHƯƠNG TRÌNH ĐẠO HÀM RIÊNG

Henry Tran

Wayne State University, Michigan, USA

LỜI BAN BIÊN TẬP

Bài viết tác giả Henry Tran có nguyên tiếng Anh, có ba phần gồm: • Lý thuyết ứng dụng lập trình MATLAB phương trình Parabolic • Lý thuyết ứng dụng lập trình MATLAB phương trình Hyperbolic • Lý thuyết ứng dụng lập trình MATLAB phương trình Elliptic

Chúng tơi tiến hành dịch, biên tập trích đăng số báo Epsilon, mở đầu Parabolic

1 Giới thiệu

Chúng ta sử dụng phương trình đạo hàm riêng (Partial Differential Equations, ký hiệu PDEs) để biểu diễn cho nhiều tượng vật lý thực tế, toán học vật lý lý thuyết Trong viết này, tập trung vào phương trình đạo hàm riêng phương trình truyền nhiệt, phương trình Laplace, phương trình sóng, phương trình Poisson, phương trình Helmholtz, v.v

Thơng thường, với dạng phương trình đạo hàm riêng, áp dụng phương pháp số để tìm nghiệm gần với điều kiện biên cho trước, sai số, độ hội tụ nghiệm gần tìm nghiệm xác

(56)

Thông qua việc biểu diễn đồ họa phương trình nêu tính tốn phần mềm lập trình cho tốn ứng dụng MATLAB khơng gian hai chiều ba chiều, so sánh phương pháp khía cạnh định Hơn nữa, dựa vào giá trị khác bước không gian hvà bước thời gian k, điều kiện biên, xác định tính ổn định, tính hội tụ nghiệm qua dạng toán PDEs

Chúng ta lập trình với phần mềm MATLAB để hỗ trợ việc tính tốn biểu diễn nghiệm toán PDEs đồ thị MATLAB phiên 14 cơng cụ mạnh cho tốn ứng dụng, cịn chứa cơng thức, bảng cơng cụ dành cho phương trình đạo hàm riêng Trong tính tốn lập trình cho tốn PDEs câu lệnh MATLAB qua việc sử dụng hàm tính tốn

Cuối cùng, thơng qua so sánh, biểu diễn tính ứng dụng phương pháp sai phân hữu hạn vấn đề, nêu kết tuỳ thuộc vào tính ổn định tính hội tụ

2 Các toán dạng Parabolic

2.1 Phương trình đạo hàm riêng dạng Parabolic

Một phương trình đạo hàm riêng bậc hai có dạng a∂

2u

∂x2 + 2b

∂2u

∂xy +c ∂2u

∂y2 +d

∂u ∂x +e

∂u

∂y +f = gọi toán Parabolic ma trận

M =

a b b c

thỏa mãn điều kiệndet (M) =ac−b2 = 0.

Từ phương trình trên, ta viết dạng đơn giản PDEs tốn Parabolic khơng gian chiều, cịn gọi phương trình truyền nhiệt:

∂u ∂t =a

∂2u

∂x2

0≤x≤L,

0≤t≤T,

(57)

• L: Độ dài cho trước • T: Thời gian cho trước • ∆x: Bước độ dài • ∆t: Bước thời gian

Chúng ta có hai điều kiện biên làu(0, t) = bvàu(L, t) =c

2.2 Phương pháp tường minh (Explicit method)

2.2.1 Phương pháp tường minh sử dụng cơng thức

Xét phương trình

∂u ∂t =a

∂2u

∂x2

Chúng ta bắt đầu với định lý Taylor theo biếnhtrong công thức tổng quát

f(x+h, y) =

n X k=0

f(k)(x, y)

k! h

k+f(n+1)(ξ, y)

(n+ 1)! h

n+1

vớiξ ∈(x, x+h).Do đó, hàm sốf có dạng

f(x+h, y) =f(x, y) +hfx(x, y) +

1

2h

2f00(ξ, y) đưa dạng

fx(x, y) =

1

h[f(x+h, y)−f(x, y)]−

1

2hf

(58)

hoặc xấp xỉ thành

fx(x, y)≈

1

h[f(x+h, y)−f(x, y)] Tương tự, ta có kết sau

f(x−h, y) =f(x, y)−hfx(x, y) +

1

2h

2f

xx(x, y)−

1 3!h

3f000(ξ)

f(x+h, y) =f(x, y) +hfx(x, y) +

1

2h

2f

xx(x, y) +

1 3!h

3f000(ξ). Cộng vế đẳng thức, ta có biểu thức củafxx(x, y)là

fxx(x, y) =

1

h2 [f(x+h·y)−2f(x, y) +f(x−h, y)]

Tiếp theo, ta có phương trình đạo hàm riêng với số bước độ dàihtrong không gian giá trịk theo thời gian

uxx(x, t) =

∂2u(x, t)

∂x2 =

1

h2 [u(x+h, t)−2u(x, t) +u(x−h, t)]

ut =

∂u(x, t)

∂t =

1

k [u(x, t+k)−u(x, t)] Suy

1

h2 [u(x+h, t)−2u(x, t) +u(x−h, t)] =

1

k[u(x, t+k)−u(x, t)] (1) với điều kiện

• a=

• hlà độ dài bước khơng gian (space steps) • klà giá trị bước thời gian (time steps)

(59)

Phương trình (1) cịn viết thành dạng

u(x, t+k) =σ u(x+h, t) + (1−2σ)u(x, t) +σ u(x−h, t) (2)

trong đóσ = hk2

Chúng ta có điều kiện ổn định cho toán dạng Parabolic là:σ= k h2 ≤ 12

2.2.2 Công thức tường minh theo dạng ma trận

Chúng ta sử dụng phương trình (1) với bốn điểm sau:

u(x, t+k) =σ u(x+h, t) + (1−2σ)u(x, t) +σ u(x−h, t) Bằng cách viếtuj = [u

0j, u1j, u2j, , unj]T phương trình (2) đưa dạng với giá trịi, j cho hàmunhư sau:

ui,j+1 =σ ui+1,j+ (1−2σ)ui,j +σ ui−1,j đóu(xi, tj) =uij =u(idx, jdt)vài= 1, , n;j = 1, , m Bây viết phương trình (2) thành dạng ma trận là:

uj+1 =Auj A=               

1−2σ σ · · ·

σ 1−2σ σ

0 σ 1−2σ

σ

σ

1−2σ σ

σ 1−2σ σ

0 · · · · · · σ 1−2σ

(60)

2.2.3 Phương trình truyền nhiệt: tốn mẫu lập trình tính tốn

Xét hệ sau 

 

∂2

∂x2u(x, t) = ∂t∂u(x, t) u(0, t) = u(1, t) =

u(x,0) = sinπx

trong đó0≤x≤1,0≤t≤1.Chúng ta phương trình vi phân với tham sốh, klà

1

h2 [u(x+h, t)−2u(x, t) +u(x+h, t)] =

1

k[u(x, t+k)−u(x, t)] ⇔u(x, t+k) =σ u(x+h, t) + (1−2σ)u(x, t) +σ u(x−h, t)

Điều kiện ổn định làσ = k h2 ≤ 12

2.2.4 Ví dụ lập trình tính tốn

Xét ví dụ sau 

 

∂2

∂x2u(x, t) = ∂

∂tu(x, t) u(0, t) = u(1, t) =

u(x,0) = sinπx

ChọnL= 1, T = 0.25,h=dx= L nx =

1

24 ; k=dt = nTt = 02.1025

• ntlà số bước thời gian • nxlà số bước độ dài

(61)

Chúng ta áp dụng công thức tường minh để giải phương trình truyền nhiệt

σ= k

h2 =

dt dx2 Nghiệm gần tính cơng thức

u(x, t+k) =σ u(x+h, t) + (1−2σ)u(x, t) +σ u(x−h, t) Trong đó, nghiệm xác

uexact(x, t) = e−π 2t

sinπx

Tiếp theo lập trình tính tốn (xem thêm phần Phụ lục bên dưới)

Chúng ta có kết nghiệm gần phương pháp tường minh nghiệm xác tạiT = 0.25trong đồ thị sau

(62)(63)

Đồ thị nghiệm gần phương pháp tường minh nghiệm xác vớih = n1x với

(64)

Minh họa nghiệm gần phương pháp tường minh không gian chiều

Minh họa nghiệm xác khơng gian chiều

2.3 Phương pháp Crank-Nicolson

Dựa vào phương trình 1, ta biến đổi lại sau

1

h2 [u(x+h, t)−2u(x, t) +u(x−h, t)] =

1

k[u(x, t)−u(x, t−k)], đór= +s, s = h2

k Tiếp theo, ta viết dạng

−u(x−h, t) +ru(x, t)−u(x+h, t) =su(x, t−k)

(65)

Đặtbi =su(x, t−k), từ phương trình (2), viết lại dạng ma trận thành         

r −1

−1 r −1 0

0 −1 r

−1

0 −1 r −1

0 −1 r

                  u1 u2 u3 un−2

un−1

         =          b1 b2 b3 bn−2

bn−1

        

Phương trình viết thành dạng đơn giản

bj =M uj M =          r −1 −1 r −1 −1 r

−1

−1 r −1

−1 r

        

2.3.1 Ví dụ

Xét ví dụ sau   

∂2

∂x2u(x, t) = ∂t∂u(x, t) u(0, t) = u(1, t) =

u(x,0) = sinπx

(66)

L= 1, T = 0.25;

h=dx= L

nx =

1 24;

k =dt= T

nT =

1 210;

nx = 24 nt= 210

• ntlà số bước thời gian • nxlà số bước khơng gian

• dtlà giá trị bước thời gian • dxlà độ dài bước không gian

Ta áp dụng phương pháp sai phân hữu hạn để giải phương trình truyền nhiệt

r= +s, s = h

2

k ,

bi =su(x, t−k),−u(x−h, t) +ru(x, t)−u(x+h, t) =su(x, t−k)

2.3.2 Lập trình tính tốn.

Chúng ta có kết nghiệm gần phương pháp Crank-Nicolson nghiệm xác vớiT = 0.25trong đồ thị minh họa

Nghiệm xác

uexact(x, t) = e−π 2t

sinπx

(67)(68)(69)

Minh họa nghiệm gần phương pháp Crank-Nicolson không gian chiều

(70)

3 Quá trình hội tụ nghiệm

Ta so sánh nghiệm xác, nghiệm cho phương pháp tường minh phương pháp Crank-Nicolson

3.1 Ví dụ

Xét ví dụ sau 

 

∂2

∂x2u(x, t) = ∂t∂u(x, t) u(0, t) = u(1, t) =

u(x,0) = sinπx

Nghiệm xác

uexact(x, t) = e−π 2t

sinπx, tạiT = 0.25

Các điều kiện biên ban đầu là:

u(x,0) = sinπx

u(0, t) =

u(1, t) =

Chúng ta dùng chuẩn vô hạn (infinity norm) để tìm sai số nghiệm:

kuexact−uhk∞ ≤C·hα kuexact−uhk∞= max

0≤j≤n |(u−uh) (xj)|

• uexact :là nghiệm xác • uh :là nghiệm gần • α:là độ hội tụ nghiệm

Sai số nghiệm gần nghiệm xác phương pháp sai phân hữu hạn e=kuexact−uhk∞= max

(71)

3.2 Độ hội tụ

Từ việc tìm sai sốt nghiệm, ta tìm độ hội tụ nghiệm (α2i)bởi cơng thức

Tổng quát, ta có

e2i

e2(i+1)

=

h2i

h2(i+1) α2i

=

2i

1 2i+1

α2i

= 2α2i

α2i = log2

e2i

e2(i+1)

Vớii= 2, ta có cơng thức sau:

e4 e8 = h4 h8 α4 =

1/4 1/8

α4

= 2α4 ⇒α

4 = log2

e4 e8 ,

• e2i, i= 1, n: sai số nghiệm gần nghiệm xác

• α2i :là độ hội tụ nghiệm

3.3 Lập trình tính tốn (xem thêm phần Phụ lục bên dưới)

Chúng ta có bảng giá trị hội tụ nghiệm gần nghiệm xác từ hai phương pháp là:

trong đó:

(72)

• The order of the explicit and exact solution: Độ hội tụ nghiệm theo phương pháp tường minh nghiệm xác

• The errors of the CN method and the exact solution: sai số của nghiệm theo phương pháp Crank-Nicolson nghiệm xác

• The order of the CN method and the exact solution: Độ hội tụ nghiệm theo phương pháp Crank-Nicolson nghiệm xác

4 Kết luận

Trong phần này, trình bày phương pháp sai phân hữu hạn cho PDEs Parabolic với dạng đơn giản nhất, phương trình truyền nhiệt

Bằng cách tính tốn biểu diễn đồ thị theo giá trị củahvàk, thấy giá trị khác đồ thị khác hàmu.Điều chứng tỏ độ ổn định độ hội tụ nghiệm Hơn nữa, áp dụng dạng khác phương pháp sai phân hữu hạn, phương pháp Crank- Nicolson để biểu diễn tính ổn định tính hội tụ nghiệm (do tính ổn định phương pháp), sử dụng giá trị lớn củak

Phần biểu diễn sai số nghiệm gần nghiệm xác hai phương pháp (tường minh Crank-Nicolson) giá trị khác hvà với giá trị cố định củak.Điều thể tính xấp xỉ nghiệm gần

Xét mặt giải tích, khơng thể biết nghiệm xác hai phương pháp sai phân hữu hạn trên, trình hội tụ bảng cho biết sai số độ hội tụ nghiệm hai phương pháp, hai phương pháp cho kết khác chúng ln hội tụ nghiệm xác

Xét mặt hình học, biểu diễn kết ba nghiệm bao gồm nghiệm gần phương pháp tường minh, phương pháp Crank-Nicolson nghiệm xác đồ thị với giá trị khác h.trong không gian chiều Trong không gian ba chiều, biểu diễn hai đồ thị phương pháp chúng cho ta thấy rõ dáng điệu hình ảnh đồ thị nghiệm

(73)

5 Phụ lục

5.1 Chương trình MATLAB 1: Phương pháp tường minh nghiệm chính xác

function fdm_para(L,T,nT,nx)

% Matlab Program : The explicit method and the exact solution %L = 1.; % Length of space

%T = 0.25; % Length of time

%Parameters needed to solve the equation within the explicit method %Choose nT= 2^10 Number of time steps

dt = T/nT; %dt = 2^(-10);

%nx = 2^4; %Choose nx= 2^4 Number of space steps

dx = L/nx; %Change with dx = h = 1/4,1/8,

s = (dx^2)/dt; r = + s;

sigma = dt/(dx^2) %Stability parameter (sigma = <1/2)

%Initial temperature of the wire: %initilalize u

u = zeros(nx+1,nT+1); u2 = zeros(nx+1,nT+1); x = zeros(nx+1,1); b = zeros(nx+1,1);

time = zeros(nT+1,1);

for i = 1:nx+1

x(i) = (i-1)*dx; u(i,1) = u0(x(i));

end

% Temperature at the boundary (t=0)

for k = 1:nT+1

u(1,k) = 0; u(nx+1,k) = 0.; time(k) = (k-1)*dt;

end

% Implementation of Finite Difference method by matrix form

A = zeros(nx-1,nx-1); A(1,1) = 1-2*sigma; A(1,2) = sigma;

A(nx-1,nx-2) = sigma; A(nx-1,nx-1) = 1-2*sigma;

for i = 2:nx-2 % Matrix form

A(i,i) = 1-2*sigma; A(i,i+1) = sigma; A(i,i-1) = sigma;

(74)

for k = 1:nT %Time Loop

u(2:nx,k+1) = A*u(2:nx,k);

end

%Implementation of the exact equation

for i = 1:nx+1

for k=1:nT+1

u2(i,k) = exact_u(x(i),time(k));

end end

%Graphical representation of the temperature at different selected times

% Plot explicit method in 2D

plot(x,u(:,nT),’-k’,’MarkerFaceColor’,’k’); hold on

%Plot the exact solution in 2D

plot(x,u2(:,nT),’-b’,’MarkerFaceColor’,’b’); hold on

xlabel(’x’)

ylabel(’temperature’)

legend({’Explicit method’ ’Exact solution’ },’location’,’NE’);

title(’The explicitmethod & the exact solution for h=1/32, at T=0.25’)

hold off

% Plot explicit method in 3D

figure

mesh(x,time,u’)

title(’Temperature within the explicit method for h=1/32,at T=0.25’)

xlabel(’x’)

ylabel(’time’)

zlabel(’Temperature’)

%Plot the exact solution in 3D

figure

mesh(x,time,u2’)

title(’Temperature within the exact solutions for h=1/32,at T=0.25’)

xlabel(’x’)

ylabel(’time’)

zlabel(’Temperature’)

Nghiệm xác

function u2 = exact_u(x,t)

u2 = exp(-(pi^2)*t)*sin(pi*x);

(75)

Trong đó, sử dụng chương trình để tính : function u = u0(x)

% Initial condition

u = sin(pi*x);

5.2 Chương trình MATLAB 2: Phương pháp Crank-Nicolson nghiệm chính xác

function cn_para(L,T,nT,nx)

%Matlab Program : The Crank-Nicolson method and the exact solutions %L = 1.; % Length of the wire

%T = 0.25; % Final time

% Parameters needed to solve the equation

%Choose nT= 2^10 Number of time steps

dt = T/nT; %dt = 2^(-10); nT is fixed 2^10

%nx = 2^4; %Choose nx= 2^4 Number of space steps

dx = L/nx; %Change with dx =h = 1/4,1/8,

s = (dx^2)/dt; r = + s;

% Initial temperature of the wire % Initilalize u

u = zeros(nx+1,nT+1); u2 = zeros(nx+1,nT+1); x = zeros(nx+1,1); b = zeros(nx+1,1);

time = zeros(nT+1,1);

for i = 1:nx+1

x(i) = (i-1)*dx; u(i,1) = u0(x(i));

end

%Temperature at the boundary (t=0)

for k = 1:nT+1

u(1,k) = 0; u(nx+1,k) = 0.; time(k) = (k-1)*dt;

end

%Implementation of the exact equation

for i = 1:nx+1

for k = 1:nT+1

u2(i,k) = exact_u(x(i),time(k));

(76)

end

%Implementation of the Crank - Nicolson method by matrix

M = zeros(nx-1,nx-1); M(1,1) = r;

M(1,2) = -1;

M(nx-1,nx-2) = -1; M(nx-1,nx-1) = r;

for i = 2:nx-2

M(i,i) = r; M(i,i+1) = -1; M(i,i-1) = -1;

end

for k = 2:nT+1 %Time Loop

u(2:nx,k) = M\(s*u(2:nx,k-1));

error2 = max(abs(u2(2:nx,k) - u(2:nx,k)));

end

disp(error2);

%Graphical representation of the temperature at different selected times

figure

%Plot the exact solution in 2D

plot(x,u2(:,nT),’-k’,’MarkerFaceColor’,’k’); hold on

%Plot Crank -Nicolson method in 2D

plot(x,u(:,nT),’-r’,’MarkerFaceColor’,’r’); hold off

%Plot Crank -Nicolson method in 3D

figure

mesh(x,time,u’)

title(’Temperature within the Crank-Nicolson method for h=1/32,at T=0.25’)

xlabel(’x’)

ylabel(’time’)

zlabel(’Temperature’)

%Plot the exact solution in 3D

figure

mesh(x,time,u2’)

title(’Temperature within the exact solutions for h=1/32,at T=0.25’)

xlabel(’x’)

ylabel(’time’)

zlabel(’Temperature’)

Nghiệm xác

(77)

u2 = exp(-(pi^2)*t)*sin(pi*x);

end

Trong đó, sử dụng chương trình để tính function u = u0(x)

% Initial condition

u = sin(pi*x);

Tài liệu

[1] W Cheney., D Kincaid 2008 Numerical Mathematics and Computing, Sixth edition Belmont CA: Thomson, 582-624

[2] E.K.P Chong, S.H Zak., 2001 An Introduction to Optimization, Second edition New York: John Wiley&Sons, Inc

[3] R J Lopez 2001 Advanced Engineering Mathematics Addison Wesley

[4] G.E Forsythe., W.R.Wasow.1960 Finite Difference Methods for Partial Differential Equa-tions New York: John Wiley&Sons, Inc

[5] G.F.D Duff., D Naylor 1966 Differential Equations of Applied Mathematics New York: John Wiley&Sons, Inc

[6] G.F.D Duff., D Naylor 1966 Differential Equations of Applied Mathematics New York: John Wiley&Sons, Inc

[7] R J LeVeque.2007 Finite Difference Methods for Ordinary and Partial Differential Equa-tions Philadelphia: SIAM

[8] E.D Rainville., P.E Bedient., R.E Bedient 1997 Elementary Differential Equations, Eight edition New Jersey: Prentice Hall Inc

[9] S.C Chapra., 2012 Applied Numerical Methods with Matlab for Engineers and Scientists, Third edition New York: McGraw-Hill

[10] P.D.Lax., Hyperbolic Systems of Conservation Laws and the Mathematical Theoryof Shock Waves Society for Industrial and Applied Mathematics, Philadelphia, 1973 ,Octo-ber

(78)

[12] W Boyce and R.D Prima., Elementary Differential Equations and Boundary Value Prob-lems John Wiley&Sons Ltd., Hoboken, N.J., Eight edition, 2005

[13] L.C Evans , Partial Differential Equations, volume 19 of Graduate Studies in Mathemat-ics American Mathematical Society, Providence, RI, Second edition, 2010

[14] R Haberman., Applied Partial Differential Equations Prentice Hall Inc., Upper Saddle River, N.J., Fourth edition, 2004

[15] T Myint-U, L Debnath., Linear Partial Differential Equations for Scientists and Engi-neers Birkhauser Boston Inc., Boston, MA, Fourth edition, 2007

[16] F John., Partial Differential Equations, volume of Applied Mathematica Sciences Springer-Verlag, New York, Fourth edition, 1982

[17] M A Pinsky Partial Differential Equations and Boundary Value Problems with Applica-tions Waveland Press Inc., Prospect Heights, Illinois, Third edition, 2003

[18] I P Stavroulakis, S A Tersian Partial Differential Equations World Scientific Publishing Co Inc., River Edge, NJ, second edition, 2004 An introduction with Mathematica and MAPLE

[19] W A Strauss Partial Differential Equations John Wiley&Sons Ltd., Chichester, Second

edition, 2008

[20] S.Attaway., MATLAB: A Practical Introduction to Programming and Problem Solving, Elsevier Science, Burlington, MA, 2009

[21] D Hanselma , B Littlefield., Mastering MATLAB 7, Prentice Hall, Upper Saddle River, NJ, 2005

(79)

PHƯƠNG PHÁP GIẢI TÍCH

TRONG CÁC BÀI TỐN OLYMPIC

Kiều Đình Minh

(Trường THPT chuyên Hùng Vương, Phú Thọ)

GIỚI THIỆU

Giải tích ngành tốn học rộng lớn có nhiều ứng dụng mạnh mẽ Trong chương trình trung học phổ thơng làm quen với phần mở đầu giải tích thực Tuy nhiên việc vận dụng số kiến thức ỏi vào giải tốn thi Olympic thật thú vị hiệu Trong viết chủ yếu tập trung khai thác định nghĩa giới hạn dãy, giới hạn hàm việc giải toán dãy số số học Các toán ứng dụng đạo hàm quen thuộc, tốn phương trình hàm, đa thức, bất đẳng thức nêu nhiều tài liệu mà khơng nhắc lại

1 Các tốn đại số

Ví dụ Cho dãy số.an/ ; bn/xác định bởi

an Dln 2n2C1

ln n2CnC1; nD1; 2; : : : bnDln 2n2C1

Cln n2CnC1; nD1; 2; : : :

a) Chứng minh có hữu hạn sốnsao chofang< 12:

b) Chứng minh tồn vô hạn sốnsao chofbng< 20161 :

Lời giải a) Dễ thấy n2n2C2nCC11 < 2;8n D 1; 2; : : : Từ suy an < ln2 < ŒanD0:Với kết này, ta cófang Danvà

limfang DlimanDlim ln

2n2C1

n2CnC1 Dln2:

Do tồn tạin0 2Nđểfang>ln2 19921 ;8n>n0:Bây có vơ hạn sốnđểfang < 12;

ta chọnn1 > n0là số Khi đó, theo lý luận trên, ta có

1

2 >fan1g >ln2 1992 )

1

(80)

Mâu thuẫn cho ta kết cần chứng minh

b) Dễ thấy.bn/tăng limbn D C1:Ngồi ta có

lim.bn bn 1/Dlim ln

2n2C1 n2CnC1

.2n2 4nC3/ n2 nC1/ D0:

Trở lại toán, giả sử tồn hữu hạn nđể fbng < 20161 :Khi đó, ta thấy tồn n0 N để

fbng> 20161 ;với mọin>n0:

Do lim.bn bn 1/D 0nên tồn tạin1 Nđủ lớn đểbn bn < 20161 ;với mọin > n1:Vì

.bn/tăng dần vơ hạn nên ta thấy tồn vô số sốn >maxfn0; n1gđểŒbn Œbn 1D1:

Xét sốnnhư thế, từ bất đẳng thức trên, ta suy Œbn Œbn 1C fbng fbn 1g<

1 2016;

hayfbn 1g >fbng C 20152016:Dofbng > 20161 ;nênfbn 1g > 1:Mâu thuẫn nhận cho ta điều

phải chứng minh

Nhận xét Bài toán thể vận dụng sâu sắc định nghĩa giới hạn dãy Mấu chốt

ý b) phát giới hạn dạng lim

n!C1 ˛bn bn 1/ ;giới hạn gặp nhiều

bài tốn khác

Ví dụ Giả sử0 < ˛62a1; a2; : : : ;là dãy số thực dương thỏa mãn

an˛ >an 1Can 2C Ca1;8n>2:

Chứng minh có số thựccsao choan>nc;8n>1:

Lời giải Ta cóan > a1=˛1 ;8n >2:Do đóa˛n > n 2/ a1=˛1 Ca1;8n > 2:Vậyan ! C1:

Do đó, tồn tạin0 choan>1;8n > n0:

Lấyc Dminn14; a1; a2

2; : : : ; an0

n0

o

:Xét bất đẳng thức

an>cn; 8n>1: (1)

Rõ ràng (1) với mọin 6n0 Xétn > n0:Giả sử (1) với mọik 6n:Ta chứng minh

(1) chok DnC1:Thật 2.nnC1/ > 14 >c;cho nên

a2nC1 >an˛C1 >anC Ca1>

n nC1/

2 c >.nC1/

2

c2: Suy raanC1 >.nC1/ c:

Ví dụ Choa0 < a1 < a2 < là dãy vô hạn số nguyên dương Chứng minh tồn

tại nhấtn>1sao cho an<

a0Ca1Ca2C Can

n 6anC1:

(81)

Lời giải Xét dãyxnDnan P iD0

ai:Ta có

x0 Dx1 D a0;

xnC1 xnDn anC1 an/ > 0;

xnD a0C n

X

iD1

.an ai/> a0C n

X

iD1

.n i /D n n 1/

2 a0 ! C1:

Do tồn nhấtnthỏa mãn xn < xnC1; suy điều phải chứng minh Nếu bỏ giả

thiết nguyên dương kết khơng cịn nữa, chẳng hạnanD2 21n:

Nhận xét Ta ý dãy số hàm số đặc biệt Trong toán ta sử dụng định lý

giá trị trung gian để có khẳng định điều cần chứng minh

Ví dụ Choa2 R;jaj< 1:Tìm tất dãy số thực.xn/ ; nD1; 2; : : :thỏa mãn điều

kiện sau

a) jxnj < 1:

b) xnD 12 xn2C1Ca2

:

Lời giải VìjxnC1j < 1nên theo b) ta cójxnj a

2C1

2 :Đặtˇ D a2C1

2 :Dojaj< 1nênˇ < 1;

suy rajxnj6ˇ:Mặt khác với mọin>1;ta có

jxnC1 xnj D

ˇ ˇ ˇ ˇ

xn2C2Ca2

2

x2nC1Ca2

2 ˇ ˇ ˇ ˇD j

xnC2 xnC1j

ˇ ˇ ˇ ˇ

xnC2CxnC1

2

ˇ ˇ ˇ ˇ

6jxnC2 xnC1j

jxnC2j

2 C

jxnC1j

2

6ˇjxnC2 xnC1j:

Bằng quy nạp theok, ta có với mọik2 Nthì

jxnC1 xnj6ˇkjxnCkC1 xnCkj 6ˇk.jxnCkC1j C jxnCkj/6ˇk 2ˇ D2ˇkC1:

Vì0 < ˇ < 1nên chok! C1thì2ˇkC1 !0:Vậy ta cójxnC1 xnj D0suy raxnC1 Dxn:

Thay vào điều kiện b) ta có

xnD

xn2Ca2

2 ;8n>1; ,xn2 2xnCa2 D0:

Dojxnj< 1;nênxn D1

p

1 a2:

Nhận xét Bài toán tinh tế chỗ chok ! C1thì với ncố định ta khẳng định jxnC1 xnj D0suy raxnC1 Dxn;8n>1:

Ví dụ Xét dãy số thực vơ hạnx1; x2; : : : ; xn; : : :thỏa mãn điều kiện

jxmCn xm xnj<

1

mCn;8m; n2N :

(82)

Lời giải Theo giả thiết ta có

j.xnC1 xn/ xkC1 xk/j D j.xnCkC1 xn xkC1/ xnCkC1 xnC1 xk/j

6jxnCkC1 xn xkC1j C jxnCkC1 xnC1 xkj

<

nCkC1;8k; n2N :

Suy

j.xnC1 xn/ xkC1 xk/j6j.xnC1 xn/ xmC1 xm/j C j.xmC1 xm/ xkC1 xk/j

<

mCnC1C

mCkC1 <

m;8m 2N :

Do lim

m!C1

4

m D0nên

j.xnC1 xn/ xkC1 xk/j D0;8k; n2N;

suy

xnC1 xn DxkC1 xk;8k; n2N:

Vậyx1; x2; : : : ; xn; : : :là cấp số cộng

Nhận xét Từ bất đẳng thức

j.xnC1 xn/ xkC1 xk/j6j.xnC1 xn/ xmC1 xm/j C j.xmC1 xm/ xkC1 xk/j

<

mCnC1C

mCkC1 <

m;8m 2N :

Cố địnhk chon! C1ta dãy số.xnC1 xn/có giới hạn làxkC1 xk Nhưngklà

số bất kỳ, mà giới hạn dãy số tồn nên hiệuxkC1 xk khơng phụ thuộc

vàok:Nói cách khác dãy cho cấp số cộng

Ví dụ Chứng minh không tồn dãy vô hạn số thực.xn/ ; n D1; 2; : : :thỏa mãn

đồng thời điều kiện sau: a) jxnj 60; 666:

b) jxn xmj> n.n1C1/ C m.m1C1/;8m; n2N; mÔn:

Li gii Giả sử tồn dãy.xn/thỏa mãn đề Với mọin2Nta xếpx1; x2; : : : ; xnthành

dãy khơng giảmxi1 6xi2 6xinở đó.i1; i2; : : : ; in/là hoán vị của.1; 2; : : : ; n/ Đặt AD0; 666;ta có

2A>xin xi1 D.xin xin 1/C.xin xin 2/C C.xi2 xi1/ >i

n.inC1/

C

in 1.in 1C1/

C C

1

i2.i2C1/

C

i1.i1C1/

D2

n

X

iD1

1 i iC1/

1 in.inC1/

1 i1.i1C1/

>2

n

X

iD1

1 i iC1/

1 D2 1

(83)

Chon ! C1thì 43 nC21 ! 43 mà 43 > 1; 332 D 2A:Điều mâu thuẫn chứng tỏ không tồn dãy vô hạn số thực.xn/thỏa mãn điều kiện đề

Ví dụ Cho dãy số.xn/thỏa mãnjxn xmj > 1n;8m; n2 N; n < m:Chứng minh dãy

.xn/không bị chặn.

Lời giải Ta chứng minh phản chứng Giả sử tồn số dươngM chojxnj < M với

mọin2N:Xét khoảng

./D

M

2; M C

; chứa tất số hạng củaxn:

Với mỗixn;ta xétn D xn 2n1 ; xnC 2n1

2n - lân cận củaxn:Cácnđôi không giao

nhau tập của./nên tổng độ dài chúng không lớn hơn2MC1là độ dài của./:Mặt khác tổng độ dài chúng là1C 12 C

1

3 C C

nC ! C1:Điều vô lý

này chứng tỏ dãy.xn/là khơng bị chặn

2 Các tốn số học

Trước hết, ta nhắc lại tính chất quen thuộc quan trọng sau:Nếu dãy số nguyên.an/hội tụ về

athì tồn tạin0sao cho với mọin>n0 thìan Da:

Thật vậy, hiển nhiêna2Z:Theo giả thiết lim

n!C1anDa;ta suy

9n0 W jan aj<

1

2;8n > n0 ) jan aj D0;8n > n0)anDa;8n > n0:

Ví dụ Cho dãy số nguyên.an/ ; nD0; 1; 2; : : :thỏa mãn

06anC7anC1C10anC2 69:

Chứng minh tồn số tự nhiênn0 sao cho với mọin>n0thìan D0:

Lời giải Đặtxk D minfak; akC1; : : :g; yk Dmaxfak; akC1; : : :gthì.xk/là dãy tăng,.yk/là

dãy giảm vàxk 6yk;8k:Dãy.an/bị chặn nên hai dãy.xk/ ; yk/cũng bị chặn Do hai

dãy hội tụ

Giả sử limxn D x; limyn D y: Do xk; yk Z nên tồn n0 cho với mọin > n0

xnDx; ynDy:Tồn tạin>n0sao cho

anC2 Dy; an; anC1 >x)8xC10y 69: (2)

Tương tự, tồn tạim>n0sao cho

amC2 Dx; am; amC1 6y )10xC8y >0: (3)

Từ (2), (3) vàx; y2 Zsuy raxDy D0:Do lim

(84)

Ví dụ Choa b là số nguyên choa2nCb là số phương với số

nguyên dươngn Chứng minh rằngaD0:

Lời giải Giả s rnga Ô 0:Th thỡa > 0;vỡ ngc lia < 0thì với nlấy giá trị lớn số a2nCblà âm

Từ giả thiết, tồn dãy số nguyên dương.xn/n>1sao choxn D

p

a2nCb;8n:Tính tốn

trực tiếp

lim

n!C1 2xn xnC2/D0:

Suy tồn số nguyên dươngn0sao cho2xnDxnC2;8n>n0:Nhưng2xnDxnC2là tương

đương vớibD0:Khi đóavà2ađều số phương, điều ny l khụng th viaÔ0: Vy gi s ca ta sai, đóaD0:

Nhận xét Đây tốn hay khó Một lần ta thấy mấu chốt phát giới hạn

dạng lim

n!C1 ˛bn bn 1/ :

Ví dụ 10 ChoA; B; C 3số nguyên cho với số nguyênn;thìf n/DAn2CBnCC

là bình phương số nguyên Chứng tỏ tồn hai số nguyêna; b sao cho f n/D.anCb/2;8n2Z:

Lời giải NếuA D 0; f n/ D BnCC bình phương số nguyên, với n2 Z;khiB D0vàC Dc2; c 2Z:

XộtAÔ0:Khi ú ta phi cúA > 0;vỡ nun lớn thìf n/cùng dấu vớiA:Với mỗin2 Z; đặtAn2CBnCC DMn2;vớiMnlà số nguyên dương khinđủ lớn Hiển nhiên

lim n!C1 Mn n D p A: Ta có

.MnC1 Mn/ MnC1CMn/DMn2C1 Mn2

DA.nC1/2CB nC1/CC An2CBnCC D2AnCACB:

Suy

MnC1 MnD

2AnCACB MnC1CMn D

2AC ACnB

MnC1CMn

n

:

Thành thử ta lim

n!C1 MnC1 Mn/ D

p

A: Nhưng MnC1 Mn nhận giá trị

nguyên Suy rapADalà số nguyên dương vớinđủ lớn ta có MnC1 MnDa)MnC1 DMnCa)Mn2C1 DM

2

n C2aMnCa2

)2aMnCa2 DMn2C1 M n D2a

2

(85)

Như vậyB chia hết cho2a;tứcB D2ab;suy raMnDanCb;hay

An2CBnCC DMn2 D.anCb/

:

Tuy ta có hệ thức vớinđủ lớn, theo tính chất đa thức, ta thấy với số nguyênn:

Tiếp theo, ta đến với thí dụ sau tổng quát thí dụ

Ví dụ 11 Chou; v; a; b; c 2Nthỏa mãnu2 < vvà dãy số.x

n/xác định bởi

xn DaunCbvnCc:

Chứng minh dãy.xn/có vơ số số hạng khơng số phương.

Lời giải Giả sử phản chứng Tồn tạin02 Nsao cho với mọin>n0;tồn tạian2 Nthỏa mãn

a2

nDaunCbvnCc:Suy

a2n Da

u v

n

CbC c vn:

Chon! C1thì a2n

vn !b (vìu

2< v

) uv < 1), suy an

vn2 !

p

b:Ta có an2 bv

n

DaunCc; ,an

p

bvn2 a

nC

p bvn2

DaunCc;

)an

p bvn2

anC p

bvn2

un DaC

c un:

Vì v

u2 > 1cho nên

lim

n!C1 anC

p bvn2

un Dn!C1lim

vn2

an

vn2 C

p b

un Dn!C1lim

vn2

p b

un Dn!C1lim

v u2

n

2pb D C1

MàaCucn !a;khin! C1nên suy lim

n!C1

an

p bvn2

D0:Suy lim

n!C1

van

p bvnC22

D0:

Lại lim

n!C1

anC2

p bvnC22

D0:Từ lim

n!C1 anC2 van/ D0:Mặt khácan; v N

nên suy tồn số nguyên dươngn1sao choanC2 vanD0với mọin>n1:Ta có

an2C2 Dau nC2

CbvnC2Cc; màan2C2Dv2a2nDaunv2CbvnC2Ccv2:Suy

aunC2Cc aunv2 cv2 D0;

,aun u2 v2Dcv2 c )u2 v2D cv

2 c

aun ;8n>n1:

(86)

Ví dụ 12 Giả sửblà số nguyên lớn hơn5:Với số nguyên dươngn;xét số xn D11

„ ƒ‚ …

n

22 „ ƒ‚ …

n

5;

được viết dạng sốb:Chứng minh “tồn số nguyên dươngM sao cho với số

nguyênnlớn hơnM;sốxnlà số phương khibD10”.

Lời giải Giả sử rằngb >6có tính chất thỏa mãn tốn Ta có xnD5C2 bCb2C Cbn

C bnC1C Cb2n D5C2b 1CbC Cbn

CbnC1 1CbC Cbn D b

2n

CbnC1

C3b

b :

Xét dãy số yn D b 1/ xn: Từ định nghĩa xn ta có yn D b2n C bnC1 C3b 5: Khi

đó ynynC1 D b 1/2xnxnC1 số phương với n > M: Đặt zn D pxn: Từ

lim

n!C1

b2n

.b 1/xn D1;ta suy limn!C1

bn

zn D

p

b 1:Hơn từ

.bznCznC1/ bzn znC1/Db2xn xnC1 DbnC2C3b2 2b 5:

Ta có

lim

n!C1 bzn znC1/D

bpb : Từznlà số nguyên với mọin>M;ta cóbzn znC1 D b

p

b

2 vớinđủ lớn Do đób 1là số

chính phương nữabchia hết2znC1với mọinđủ lớn Điều dẫn đếnbj10:Do

có thểb D10:

Ví dụ 13 Ta nói số nguyênt là số tam giác nếut D n.n2C1/ với số nguyên dươngn:Tìm tất cả

cặp số nguyên.a; b/có tính chất với số ngunt thìt là số tam giác khiat Cb

là số tam giác.

Lời giải Đầu tiên ta có nhận xét “số nguyênt số tam giác khi8t C1là số phương lẻ,8tC1>9:”

Giả sử cặp số nguyên.a; b/thỏa mãnt số tam giác khiatCblà số tam giác Ta chứng minhaD1; b D0nghĩa cặp số nguyên thỏa mãn tính chất là.1; 0/ : Ta có8 at Cb/C1Da 8t C1/Cc;trong kí hiệuc D8bC1 a:Do tính chất tương đương với: Với u Z; u mod 8/; ulà số phương lẻ,u > 9khi khiauCc số phương lẻ

auCc >9: (4)

(87)

Xétv 2N; vlẻ,v >3:Giả sửt

v; tvC2 2N;đều số nguyên lẻ, lớn hơn2và thỏa mãn

(

av2Cc Dtv2

a.vC2/2Cc Dtv2C2 (5) Từ a > 0; ta có lim

v!C1tv D v!C1lim tvC2 D C1: Do limv!C1 tvC

p av

D C1: Nhưng c D tvC

p av

tv

p av

:Suy lim

v!C1 tv

p

avD0: (6)

Bởi lim

v!C1 tvC2 tv/Dv!C1lim

tvC2

p

a vC2/ tv

p

avC2paD2pa: (7) Chú ýtvC2 tv; v D3; 5; 7; : : :là dãy số nguyên nên từ (7) suy trừ số hữu hạn sốv

thìtvC2 tv D2

p

a:Hệ làpa2 N:Kết hợp với (6) nhận xét nếutv

p

av ¤0 thìˇ

ˇtv

p

avˇˇ> 1ta nhận đượcc D0:Tức làb D a 18 ;mà theo giả thiết thìb ngun nêna số phương lẻ Nhưng nếua>32 (4) sai khiu D1:VậyaD1; b D0:

Ví dụ 14 Cho a1; a2; : : : ; ak RC thỏa mãn có số chúng không nguyên.

Chứng minh tồn vô hạn số nguyên dươngn

gcd.n;ba1nc C ba2nc C C baknc/D1:

Lời giải Giả sử phản chứng Không tồn vô hạn số nguyên dươngnthỏa mãn đề Suy tồn số nguyên dươngn0mà với mọin>n0thì

gcd.n;ba1nc C ba2nc C C baknc/ > 1:

ChonDpi số nguyên tố tùy ý, ta có

ba1pic C ba2pic C C bakpic::: pi;

suy tồn số nguyênxi để

ba1pic C ba2pic C C bakpic Dxipi;

hay

pi.a1Ca2C Cak/Dxipi C fa1pig C C fakpig;

)a1Ca2C Cak xi D f

a1pig C fa2pig C C fakpig

pi

: Vì0 < ˚

ajpi < 1;8j D1; k;suy

0 < a1Ca2C Cak xi <

k pi

: Choi ! C1;ta có lim

i!C1xi Da1Ca2C Cak:Màxi 2N

(88)

và tồn tạin1 2Nsao cho

a1Ca2C Cak Dxi;8i >n1:

Do

fa1pig C fa2pig C C fakpig D0;8i >n1;

) fa1pig D fa2pig D D fakpig D0:

Suy raaj có dạng bj

pi;8i >n0; bj 2N

:Màp

i số nguyên tố nêna1; a2; : : : ; ak 2N:Điều

này mâu thuẫn với giả thiết

Ví dụ 15 Tìm tất đơn ánhf WN!Nthỏa mãn f f n// f n/Cn

2 ;8n2N :

Lời giải Cố địnha2 N:Đặt

f f f a///

„ ƒ‚ …

k

Dak:

Ở giả thiết, chonDuk;ta cóukC2 ukC12Cuk:ĐặtAk DmaxfukC1; ukg; Bk DminfukC1; ukg

thìukC2 ukC12Cuk Ak, suy maxfukC1; ukC2g Ak ) AkC1 Ak:MàAk > 0;8k

nên tồn lim

k!C1Ak DA:

Nếu tồn vơ sối chouiC1 DBiC1thì

uiC26

BiC1Cui

2

BiC1CAi

2 ;

hayAiC1

BiC1CAi

2 :

Do lim

k!C1Ak D A nên với " > 0;tồn N Nsao cho jAk Aj < ";8k > N: Cho i > N;ta có

A " < AiC1

BiC1CAi

2 <

BiC1CAC"

2 ;

suy

2A 2" < BiC1CAC";

)A 3" < BiC1 6AiC1< AC" < AC3";

) jBiC1_Aj< 3";8i > N:

Do lim

i!C1BiC1 D A: Mà Ak/ dãy số nguyên nên tồn T N thỏa mãn điều kiện Ak DBk;8k > T:Hay

ukC1Duk )f uk/Df uk 1/)uk Duk D Du1 )f a/Da:

Thử lại thỏa mãn

Nếu tồn hữu hạni chouiC1 DBiC1thì suy tồn vô sối chouiC2DBiC1:Tức

là tồn tạiT N chouiC2 D BiC1;8i > T ) uiC2 < uiC1;8i > T;điều vơ lý

(89)

Ví dụ 16 Giả sửa; b là số nguyên dương choajb2; b3ja4; a5jb6; b7ja8; : : :Chứng

minh rằngaDb:

Lời giải Từ giả thiếtajb2; b3ja4; a5jb6; b7ja8; : : :ta có a4nC1jb4nC2;8n2N;

và b4nC3ja4nC4;8n N:Gọivp.a/và vp.b/lần lượt số mũ cao số nguyên tố p

trong phân tícha; bthành thừa số nguyên tố tương ứng

Khi để chứng minh a D b ta cần chứng minh vp.a/ D vp.b/ : Từ giả thiết

a4nC1jb4nC2;8n2N, ta có vp.a/

4nC2

4nC1vp.b/ ;8n2N)vp.a/6n!C1lim

4nC2

4nC1vp.b/ Dvp.b/ : (8) Tương tự từ

b4nC3ja4nC4;8n2N)vp.b/6 lim n!C1

4nC4

4nC3vp.a/Dvp.a/ (9) Từ (8) (9) ta cóvp.a/Dvp.b/ :

Ví dụ 17 Choa; b; c; d Nđôi khác số nguyên tốp thỏa mãn apCbp Dcp Cdp:

Chứng minh rằngja cj C jb dj>p:

Lời giải Ta có

ap a0 mod p/; bp b mod p/; cp c mod p/; dp d mod p/: Suy

0D.ap cp/C.bp dp/a cCb d mod p/; suy raaCb cCd modp/ :

Ta xột cỏc trng hp sau:

aCb ÔcCd suy điều phải chứng minh

aCb DcCd:Giả sửa > c > d suy rab < d đóa > c > d > b:

Xét hàm sốf t /Dtp:Vìf t /có đạo hàm khoảng.c; a/ ; b; d /nên theo định lý Lagrange tồn tạit1 2.c; a/ ; t22 d; b/sao cho

f0.t1/D

f a/ f c/ a c ; f

0.t

2/D

f d / f b/ d b :

Do đóf0.t1/Df0.t2/ ;vơ lý vìp ngun tố vàt1; t2thuộc hai khoảng khác

(90)

3 Bài tập rèn luyện

Bài tập Cho a1; a2; : : :là dãy vô hạn số thực nhỏ 1thỏa mãnanC1.anC2/ D 3;

8n>1:Chứng minh a)

2 < an< 2:

b) anD 3; 8n:

Bài tập Cho dãy vô hạn.un/ ; nD0; 1; 2; : : :thỏa mãn hai điều kiện sau:

a) 06un62;8nD0; 1; 2; : : :

b) un 2unC1CunC2 >0; nD1; 2; : : :

Chứng minh rằng06n un unC1/62;8nD0; 1; 2; : : :

Bài tập Cho dãy số dươngx0 D1; x1Da > 0; xnC2 D x xn

nC1C1;8n>0:Chứng minh

tồn nhiều giá trịađểxnC1 6xn;8n:

Bài tập Cho dãy gồm vô hạn số thực dươnga0; a1; a2; : : :Chứng minh tồn

vô hạn giá trịnđể bất đẳng thức sau đúng1Can> an

n

p 2:

Bài tập Cho dãy số dương.cn/thỏa mãn n

P

kD1

ck < M;8n:Chứng minh có vơ số sốn

sao chocn>cnC1

n

p 2:

Bài tập Chứng minh không tồn cấp số cộng tăng thực độ dài vô hạn thỏa mãn :

Mỗi số hạng cấp số cộng có dạngab a; b 2ZC; a; b >2

Bài tập Cho dãy số thựca0; a1; a2; : : : ; an; : : :thỏa mãn

1Da0 6a16 6an (10)

Dãy sốb1; b2; : : : ; bn; : : :được xác định sau

bn D n

X

kD1

1 ak ak p ak :

Chứng minh

a) 06bn< 2;8n:

b) Với số C cho trước, C < 2;đều tồn dãy sốa0; a1; a2; : : : ; an; : : :thỏa

mãn điều kiện (10) chobn> C với vô số sốn:

(91)

a) glà hàm toàn ánh

b) 2f2.n/ Dn2Cg2.n/ ;8n2 ZC: c) jf n/ nj62016pn;8n2 ZC:

Chứng minh phương trìnhf x/Dxcó vơ số nghiệm

Bài tập Choa; blà số tự nhiên lớn hơn1thỏa mãn điều kiệnbn 1::: an 1;8n2N:

Chứng minh rằngblà lũy thừa củaa:

Bài tập 10 Dãy số nguyên dương.an/ ; nD1; 2; : : :thỏa mãn điều kiện

0 < anC1 an<pan;8n>1:

Lấy a; blà hai số nguyên dương tùy ý, với0 < a < b < 1:Chứng tỏ tồn vô số cặp số nguyên dương.p; q/sao < ap

aq < b:

Bài tập 11 Cho dãy an/tăng ngặt gồm số nguyên dương thỏa mãn dãy anC1 an/ bị

chặn Chứng minh tập ước nguyên tố dãy.an/là vô hạn

Bài tập 12 Với số tự nhiênn;gọiI n/là tập hợp số tự nhiênksao cho 50n< 7k < 50nC1:

a) Chứng tỏ với số tự nhiênn;ta cójI n/j D2hoặcjI n/j D 3: b) Chứng tỏ tồn vô số số tự nhiênnsao chojI n/j D3:

Bài tập 13 Tìm tất bộ3số dương.a; b; c/sao choŒna ŒnbD n2c

;8n2 N:

Bài tập 14 Chonlà số nguyên dương Tính phần nguyênh10C2nn p

n 4n

i :

Bài tập 15 Cho số nguyênnvới2000 n 2095:Đặta D

n

P

kD1995

k b D nC1

1995 Hãy tìm

phần nguyên sốb1a:

Bài tập 16 Chonvàklà số nguyên dương thỏa mãn điều kiệnn>7và26k < n:Chứng minh rằngkn > 2nk:

Bài tập 17 Choa; b số nguyên lớn 1:Chứng minh có bội củaachứa tất chữ số0; 1; : : : ; b 1khi viết hệ sốb:

Bài tập 18 Cho số thực dươnga; bthoả mãnb ŒanDa Œbn ;8n2ZC Chứng minh rằng aDb cảavàbđều nguyên

Bài tập 19 Chok N:Chứng minh tồn dãy a

0; a1; : : : ; an; : : : số nguyên

thỏa mãn điều kiệnan D an 1Cn

k

n ;8n>1thìk 2chia hết cho3:

Bài tập 20 Một số nguyên dương gọi “ số Kim cương2005” biểu diễn thập phân có2005số9đứng cạnh liên tiếp Dãy.an/ ; nD1; 2; 3; : : :là dãy tăng ngặt

các số nguyên dương thoả mãnan < nC C số thực dương đó) Chứng minh

(92)

Tài liệu

[1] Trần Nam Dũng (chủ biên),Các phương pháp giải toán qua kỳ thi Olympic [2] Titu Andreescu, Gabriel Dospinescu,Problems from the Book

(93)

TỔNG QUÁT HÓA ĐƯỜNG THẲNG DROZ FARNY

Trần Quang Hùng, THPT chuyên KHTN, Hà Nội

TĨM TẮT

Hai đường thẳng vng góc với trực tâm tam giác chắn ba cạnh tam giác ba đoạn thẳng mà trung điểm chúng thẳng hàng Đó nội dung định lý tiếng có tên Droz-Farny Bài viết đưa hướng tổng quát cho toán đường thẳng Droz-Farny với lời giải sử dụng phép nghịch đảo lời giải khác sử dụng tính chất chùm điều hòa

Định lý lần đề nghị Arnold Droz năm 1899 [1] Các lời giải sử dụng lượng giác, phương pháp tọa độ vector đưa [2,3,4,5,6] Trong [7] trình bày hướng tổng quát cho định lý sử dụng kiến thức tỷ số kép độ dài đại số Trong [8] định lý tổng quát khác đề cập với lời giải sử dụng tính chất điểm Miquel điểm đẳng giác Bài viết giới thiệu toán tổng quát giống [8]1 với lời giải sử

dụng phép nghịch đảo Đồng thời viết đề cập tới toán tổng quát nữa, với lời giải sử dụng túy hình học xạ ảnh

Bài tốn 1. Cho tam giácABCvà điểmP khơng thuộc đường thẳngBC, CA, AB Đường thẳng∆quaP Các điểmD, E, F thuộc đường thẳngBC, CA, ABsao cho P D, P E, P F đối xứng củaP A, P B, P C qua∆ Chứng minh D, E, F thẳng hàng

(94)

ǻ A

B

C P

F

E D

Hình

Sử dụng phép nghịch đảo cựcP phương tích ta chuyển tốn toán sau Bài toán 2. Cho tam giác ABC điểm P Gọi (K),(L),(N) đường

tròn ngoại tiếp tam giác P BC, P CA, P AB D, E, F thuộc (K),(L),(N) cho

P D, P E, P F đối xứng P D, P E, P F qua ∆ Chứng minh bốn điểm

P, D, E, F thuộc đường trịn Để giải tốn ta cần bổ đề sau

(95)

d

A

B C

D F

E

K

Hình

Chứng minh. Gọi đường thẳng quaB, Clần lượt song song vớiF D, DEcắt tạiK Ta có

(KB, KC) ≡ (F D, DE) ≡ (AB, AC)(modπ).Suy điểmK thuộc đường tròn ngoại tiếp tam giácABC Ta lại có(KA, EF)≡(KA, KB) + (KB, EF)≡(CA, CB) + (DF, EF)≡ (CA, CB) + (CB, CA)≡ 0(modπ).Do đóKAkEF Vậy đường thẳng quaA, B, C song song vớiEF, F D, DEđồng quy tạiK Ta có điều phải chứng minh

(96)

ǻ N

L

P

K A

B C

X

D Y

E Z

F K'

L'

N'

Hình

Lời giải tốn. Ta gọiX, Y, Z điểm đối xứng với A, B, C qua∆, theo giả thiết

dễ thấyX, Y, Z thuộc P D, P E, P F Gọi trung trực P X, P Y, P Z cắt tương ứng thành tam giácK0L0N0 Ta dễ thấy tam giácK0L0N0 đối xứng tam giácKLN qua∆

(97)

Nhận xét.Ta thấy kẻ đường thẳng∆0 ⊥∆thì chùm đường thằng(∆,∆0, OA, OD) chùm điều hịa OA, OD đối xứng qua ∆ Điều gợi mở cho chùng ta

hướng tổng qt tốn thơng qua khái niệm chùm điều hịa mà bỏ qua tính chất đối xứng Ta quy ước sử dụng ký hiệu

(XY, Z) = ZX

ZY tỷ số đơn ba điểm thẳng hàngX, Y, Z

(XY, ZT) = ZX

ZT : T X

T Y tỷ số kép bốn điểm thẳng hàng đồng viênX, Y, Z, T A(XY, ZT)chỉ tỷ số kép bốn tia(AX, AY, AZ, AT)

Với độ dài sử dụng độ dài đại số Ta xét toán tổng quát sau

Bài toán 3. Cho tam giácABC vàP, K, Llà điểm Giả sử có điểmD, E, F thuộc đường thẳng BC, CA, AB cho chùm P(KL, AD) = P(KL, BE) =

P(KL, CF) =−1 Chứng minh rằngD, E, F thẳng hàng

Ta thấy nếuP K ⊥ P L Ta thu toán ban đầu Bài tốn phát biểu dạng chùm điều hịa nên có lời giải túy xạ ảnh Ta cần bổ đề sau

Bổ đề 3.1. Cho điểmD, E, F, X, Y, Z, K, Lcùng thuộc đường thẳng thỏa mãn(KL, DX) = (KL, EY) = (KL, F Z) = −1 Chứng minh tích (EF, DX).(F D, EY).(DE, F Z) =

−1

Chứng minh. Bài toán thực chất biến đổi độ dài đại số trục, để đơn giản ta sử dụng tọa độ trục ChoD(d), E(e), F(f), X(x), Y(y), Z(z), K(k), L(l)

Từ (KL, DX) = −1 suy d−k

d−l :

x−k

x−l = −1 x =

dl−2kl+dk

2d−k−l Tương tự y = el−2kl+ek

2e−k−l , z =

f l−2kl+f k

2f −k−l (1)

Ta có(EF, DX).(F D, EY).(DE, F Z)

= (EF, D) (EF, X)

(F D, E) (F D, Y)

(DE, F) (DE, Z)

= −1

(EF, X).(F D, Y).(DE, Z)

=−x−f

x−e y−d y−f

z−e z−d (2)

Thay biểu thức từ (1) vào (2) dễ kiểm tra được(EF, DX).(F D, EY).(DE, F Z) =−1

(98)

d A B C P Y Z X K' L' K L Y' Z' X' F D E E' F' D' Hình

Lời giải toán. GọiP A, P B, P Clần lượt cắtBC, CA, AB tạiX, Y, Z Gọidlà đường thẳng GọiP A, P B, P C, P D, P E, P F, P K, P Llần lượt cắtdtạiD0, E0, F0, X0, Y0, Z0, K0, L0 Từ giả thiếtP(KL, AD) =P(KL, BE) =P(KL, CF) =−1, chiếu xuyên tâmP lêndta có

(K0L0, X0D0) = (K0L0, Y0E0) = (K0L0, C0F0) =−1

Áp dụng bổ đề ta suy ra(E0F0, D0X0).(F0D0, E0Y0).(D0E0, F0Z0) = −1 (1).

Sử dụng phép chiếu xuyên tâmP đường thẳngBC, CA, AB ta thu

P(E0F0, D0X0) = (BC, DX) = (BC, D)

(BC, X), P(F

0D0, E0Y0) = (CA, EY) = (CA, E)

(CA, Y), P(D

0E0, F0Z0) =

(AB, F Z) = (AB, F) (AB, Z) (2)

VìAX, BY, CZđồng quy tạiP nên theo định lý Ceva(BC, X).(CA, Y).(AB, Z) =−1 (3)

Từ (1),(2),(3) ta suy

−1 = (E0F0, D0X0).(F0D0, E0Y0).(D0E0, F0Z0)

=P(E0F0, D0X0).P(F0D0, E0Y0).P(D0E0, F0Z0)

= (BC, D) (BC, X)

(CA, E) (CA, Y)

(99)

=−(BC, D).(CA, E).(AB, F)

Áp dụng định lý Menelaus cho tam giácABC dễ suy raD, E, F thẳng hàng Ta có điều phải chứng minh

Cuối viết, tác giả xin nói lời cảm ơn tới TS Nguyễn Minh Hà người cho tác giả số ý tưởng lời giải nghịch đảo toán tổng quát

Tài liệu

[1] A Droz-Farny, Question 14111, Ed Times 71 (1899) 89-90 [2] J L Ayme, Forum Geom., Vol 4, (2004), pp 219-224

[3] F M van Lamoen, Hyacinthor messages 6140, 6144, December 11,2002 [4] D Grinberg, Hyacinthor messages 9854, July 23, 2003

[5] M Stevanovié, Hyacinthor messages 9130, January 25, 2004

[6] C Pohoata and S.H Ta, A Short Proof of Lamoen’s Generalization of the Droz-Farny Line Theorem, Mathematical reflections 2006

[7] N.M Ha and L.T Vinh, Purely synthetic proof of the Generalized Droz-Farny Theorem, Global journal of advanced research on classical and modern geometries

[8] T Andreescu and C Pohoata, Back to Euclidean: Droz-Farny Demystified, Mathematical reflections 2012

(100)

NOTE ON HERMITE - HADAMARD INEQUALITIES

Vandanjav Adiyasuren (National University of Mongolia) Enkhee Davaadulam, Bold Sanchir (Mongolian University of Life Science)

ABSTRACT

In this note we generalize some result of [1]

1 Introduction

Letf WŒa; b!Rbe a convex function, then the inequality

f

aCb b a Z b a

f x/ dx6 f a/Cf b/

2 ; 1:1/

is known as the Hermite-Hadamard inequality (see [3] for more information) Since then, some refinements of the Hermite-Hadamard inequality on convex functions have been extensively investigated by a number of authors (e.g.,[2,4])

2 Main Result

Theorem Assume thatf WI !Ris a convex function onI:Then for all2Œ0; 1;we have f

aCb

2

6ln./6 b a

Z b

a

f x/ dx6Ln./6 f a/Cf b/

2 ; 2:1/

where

ln./D

n n X

kD0

f

aC.b a/.2kC1/

2n

C.1 /f

bC.1 /aC.2kC1/.1 /.b a/

2n

;

and

Ln./D

2n n X

kD0

f

aC kC1/.b a/

n

Cf

aCk.b a/

n

C C.1 /

f

bC.1 /aCk.1 /.b a/

n

Cf

bC.1 /aC.kC1/.1 /.b a/

n

(101)

Proof First we denote

Z b

a

f x/ dxD

Z bC.1 /a

a

f x/ dxC

Z b

bC.1 /a

f x/ dx

D

n X

kD0

aC.kC1/.bn a/

Z

aCk.bna/

f x/ dxC

n X

kD0

Z bC.1 /aC.kC1/.1n/.b a/

bC.1 /aCk.1 /.bn a/

f x/ dx (2.2)

Applying.1:1/on the subintervalhaC k.b a/n ; aC kC1/.b a/n i,.k D0; : : : ; n 1/;we get n

X

kD0 f

aC b a/.2kC1/

2n n b a/ n X

kD0

Z aC.kC1/.bn a/

aCk.bna/

f x/ dx

6 n X

kD0

f

aC kC1/.b a/

n

Cf

aCk.b a/

n

: (2.3)

Applying.1:1/again onhbC.1 /aC k.1 /.b a/n ; bC.1 /aC

.kC1/.1 /.b a/ n

i , k D0; : : : ; n 1/we get

n X

kD0 f

bC.1 /aC.2kC1/.1 /.b a/

2n

6 n

.1 /.b a/ n X

kD0

Z bC.1 /aC.kC1/.1n/.b a/

bC.1 /aCk.1 /.bn a/

f x/ dx

61

n X

kD0

f

bC.1 /aCk.1 /.b a/

n

Cf

bC.1 /aC.kC1/.1 /.b a/

n

: (2.4) Multiplying.2:3/by; 2:4/by.1 /and adding the resulting inequalities, we get:

ln./6 b a

Z b

a

f x/ dx6Ln./; 2:5/

whereln./andLn./are defined as in Theorem 2.1 Using the fact thatf is a convex function, we obtain

ln./D

n n X

kD0

f

aC.b a/.2kC1/

2n

C.1 /f

bC.1 /aC.2kC1/.1 /.b a/

2n > n n X

kD0 f

aC.b a/.2kC1/

2n

C.1 /

bC.1 /aC.2kC1/.1 /.b a/

2n >f " n n X

kD0

aC.b a/.2kC1/

2n

C.1 /

bC.1 /aC.2kC1/.1 /.b a/

(102)

Other side, we can easily see that

f

aC.kC1/.b a//

n

6 n kC1/

n f a/C

.kC1/

n f b/; (2.7) f

aC k.b a//

n

6 n k

n f a/C k

n f b/; (2.8)

f

bC.1 /aCk.1 /.b a/

n

6 n k/.1 /

n f a/C

nCk.1 / n f b/;

(2.9) f

bC.1 /aC.kC1/.1 /.b a/

n

6 n k 1/.1 /

n f a/C

nC.kC1/.1 / n f b/:

(2.10)

Multiplying.2:5/and.2:6/by; 2:7/and.2:8/by.1 /and adding overkD0; n 1, we get:

Ln./6 f a/Cf b/

2 : 2:11/

Then by (2:5/,.2:6/and.2:11/we get.2:1/

References

[1] ABDALLAH EL FARISSI “Simple proof and Refinement of Hermite-Hadamard

Inequal-ity”Journal of Mathematical Inequalities Volume 4, Number (2010), 365-369

[2] DRAGOMIR, S S AND PEARCE, C E M Selected Topics on Hermite-Hadamard

Inequalities, (RGMIA Monographs http://rgmia.vu.edu.au/monographs/ hermitehadamard.html), Victoria University, 2000

[3] HADAMARD, J “Etude sur les proprietes des fonctions entires en particulier d’une

fonc-tion consideree par Riemann”) J Math Pures Appl 58,171-215 (1893)

[4] PECARIC, J E AND PROSCHAN, F AND TONG, Y C Convex Functions, Partial

(103)

ANDREI KOLMOGOROV

NGƯỜI MỞ ĐƯỜNG NGÀNH XÁC SUẤT HIỆN ĐẠI

Slava Gerovitch Người dịch Hoàng Mai

GIỚI THIỆU

Bài báo dịch từ viết The Man Who Invented Modern Probability Slava Gerovitch đăng trang Nautilus Phùng Hồ Hải hiệu đính

TheoTia Sáng

Nếu hai nhà thống kê lạc khu rừng vô hạn, trước tiên họ uống cho say. Khi đó, nói họ cách ngẫu nhiên việc mang lại hội tốt để họ gặp lại Tuy nhiên, nhà thống kê nên tỉnh táo họ muốn hái nấm Say rượu đi lung tung khơng mục đích thu hẹp phạm vi khám phá, khả cao họ quay trở lại vị trí cũ, nơi nấm bị hái hết rồi.

Kolmogorov (bìa phải) hai người bạn, nhà tốn học tiếng thời giờ, Lev Pontryagin (bìa trái) Pavel Alexandrov (ngồi giữa)

(104)

khi thân ông cẩn trọng ứng biến động đời sống trị hàn lâm nước Nga Xơ viết

Khi cịn trẻ, Kolmogorov ni dưỡng khơng khí tri thức sôi động Moskva hậu cách mạng, tràn ngập thử nghiệm văn chương, xu hướng tiên phong nghệ thuật, ý tưởng khoa học mẻ Ở năm đầu thập niên 1920;khi sinh viên lịch sử tuổi 17;ơng trình bày báo trước bạn học Đại học Moskva, đưa phân tích thống kê khác thường đời sống người Nga thời Trung cổ, cho thấy thuế khóa đánh làng thường số nguyên, thuế hộ dân lại biểu diễn phân số Bài báo kết luận – đầy tranh cãi vào thời điểm – thuế trước thu theo làng phân bổ đến hộ, thay thu theo hộ gộp tổng lại cho làng Thầy ông nhận xét gay gắt “cậu tìm thấy chứng mà thôi, vậy là không đủ với nhà sử học Cậu cần năm chứng.” Lúc đó, Kolmogorov định chuyển sang nghiên cứu toán học, nơi chứng minh đủ

Điều hợp lý cách kỳ lạ kiện ngẫu nhiên dẫn dắt Kolmogorov vào lãnh địa lý thuyết xác suất, nhánh nhỏ bị xem thường toán học Các xã hội tiền đại thường nhìn nhận kiện ngẫu nhiên biểu thị cho ý chí thần thánh, Ai Cập Hy Lạp Cổ đại, việc tung súc sắc nhìn nhận cơng cụ cho việc tiên tri hay bói tốn Cho đến đầu kỷ XIX, nhà toán học châu Âu phát triển kỹ thuật để tính tốn tỉ lệ cược, định nghĩa xác suất tỉ lệ số trường hợp muốn có số tất trường hợp đồng xác suất Nhưng cách tiếp cận lại vướng vào lập luận vòng quanh – xác suất định nghĩa theo số khả đồng xác suất – có hiệu lực với hệ có hữu hạn khả Nó khơng thích hợp với đại lượng vơ hạn đếm (như trị chơi với súc sắc có vơ hạn mặt) hay khơng đếm (như trị chơi với súc sắc hình cầu mà điểm mặt cầu khả năng) Những nỗ lực xử lý tình mang lại kết mâu thuẫn tạo hình ảnh xấu lý thuyết xác suất Uy tín danh phẩm chất Kolmogorov coi trọng Sau chuyển ngành học, ban đầu Kolmogorov gia nhập nhóm tốn Nikolai Luzin, giảng viên tiếng đầy sức hút Đại học Moskva Những học trị Luzin đặt tên cho nhóm “Luzitania”, cách chơi chữ theo tên giáo sư họ tàu Anh bị chìm Thế chiến thứ Họ thống “nhịp đập tim”, Kolmogorov mô tả, tập hợp lại sau học để bàn luận chuyên sâu phát kiến toán học Họ nhại partial differential equation (các phương trình đạo hàm riêng) thành partial irreverential equations (các phương trình bất kính riêng) finite difference (sai phân hữu hạn) thành fine night differences (những khác biệt đêm vui vẻ) Lý thuyết xác suất, thiếu sở lý thuyết chắn bị vướng vào nghịch lý, bị đùa cợt thành “lý thuyết không may

(105)

viện nghiên cứu người tiền nhiệm ông ủng hộ tự tơn giáo mà bị chế độ Stalin bỏ tù Bấy giờ, Kolmogorov tham gia phê bình quay lưng lại với Luzin Luzin trở thành đối tượng buổi xét xử Viện Hàn lâm Khoa học tất vị trí thức, khỏi bắt giam xử bắn quyền Nga cách ngạc nhiên Luzitania tan rã, bị đánh chìm thủy thủ đồn

Khơng bàn đến khía cạnh đạo đức định ông, Kolmogorov đặt cược thành công nhận lại tự để tiếp tục nghiên cứu Trái ngược với phục tùng trị, lý thuyết xác suất, Kolmogorov đưa sửa đổi cấp tiến thực tảng lĩnh vực Ông dựa vào lý thuyết độ đo, lý thuyết thời thượng, du nhập vào Nga từ Pháp Lý thuyết độ đo tổng quát hóa khái niệm “độ dài”, “diện tích” hay “thể tích”, cho phép đo đạc nhiều đối tượng tốn học rắc rối nằm khả phương pháp thơng thường Chẳng hạn, giúp tính diện tích hình vng với vơ hạn lỗ bên trong, chia thành vơ hạn mảnh nhỏ, phân tán mặt phẳng vô hạn Trong lý thuyết độ đo, người ta nói “diện tích” (độ đo) vật thể bị phân tán

Kolmogorov mô tả tương tự lý thuyết xác suất lý thuyết độ đo, thể năm tiên đề, ngày thường phát biểu thành sáu mệnh đề, đưa xác suất trở thành lĩnh vực tơn trọng giải tích toán học Khái niệm lý thuyết Kolmogorov “biến cố bản”, kết phép thử đơn lẻ, tung đồng xu Tất biến cố lập thành “không gian mẫu”, tập hợp tất kết Chẳng hạn với cú sét đánh Massachusetts, không gian mẫu bao gồm tất điểm bang mà sét đánh vào Một biến cố ngẫu nhiên định nghĩa “tập đo được” không gian mẫu, xác suất biến cố ngẫu nhiên “độ đo” tập Ví dụ xác suất sét đánh trúng Boston phụ thuộc vào diện tích (“độ đo”) thành phố Hai biến cố xảy đồng thời biểu diễn giao độ đo chúng, xác suất có điều kiện biểu diễn thương độ đo, xác suất mà hai biến cố không phụ thuộc vào xảy tính cách cộng độ đo (ví dụ như, xác suất Boston Cambridge bị sét đánh tính tổng diện tích chúng)

Nghịch lý Đường tròn lớn câu đố toán học quan trọng mà khái niệm xác suất Kol-mogorov cuối giải Giả sử người hành tinh hạ cánh ngẫu nhiên hành tinh hình cầu hồn hảo xác suất điểm hạ cánh phân bố Như có phải họ hạ cánh với xác suất nơi dọc theo đường tròn chia mặt cầu thành hai bán cầu nhau, hay cịn gọi “đường trịn lớn”? Hóa xác suất hạ cánh phân bố dọc theo đường xích đạo, phân bố khơng đường kinh tuyến, với xác suất tăng dần tới gần đường xích đạo giảm cực Nói cách khác, người ngồi hành tinh có xu hướng hạ cánh vùng có khí hậu nóng Có thể giải thích kết hình ảnh đường tròn vĩ tuyến lớn dần chúng tiến dần tới xích đạo – kết nghe thật vơ lý, quay đường trịn biến đường xích đạo thành đường kinh tuyến Kolmogorov đường trịn lớn có độ đo khơng, đoạn thẳng có diện tích không Điều lý giải mâu thuẫn hiển nhiên xác suất có điều kiện việc hạ cánh tồn khơng thể tính tốn cách nghiêm túc xác suất

(106)

phải quay với thực Trong Thế chiến thứ hai, Chính phủ Nga yêu cầu Kolmogorov phát triển phương pháp giúp tăng tính hiệu pháo binh Ơng thay cố gắng tối đa xác suất phát bắn trúng đích, số trường hợp cụ thể tốt bắn loạt đạn có độ lệch nhỏ so phát ngắm chuẩn xác, chiến thuật biết đến tên gọi “phân tán nhân tạo” Bộ môn Lý thuyết xác suất Đại học Moskva mà Kolmogorov tổ trưởng, tính tốn bảng đạn đạo cho pha ném bom tầm thấp, vận tốc nhỏ Vào năm1944và1945, phủ trao thưởng cho Kolmogorov hai Hn chương Lenin cho đóng góp ơng thời chiến sau chiến ông làm việc với tư cách cố vấn tốn học cho chương trình vũ khí nhiệt hạch

Nhưng mối quan tâm Kolmogorov hướng ơng tới hướng nghiên cứu có tính triết lý Tốn học dẫn ơng tới niềm tin giới dẫn dắt tính ngẫu nhiên đặt dựa định luật xác suất Ông thường vai trị tính khơng dự đốn mối quan hệ người Cuộc gặp gỡ tình cờ Kolmogorov với nhà toán học thời Pavel Alexandrov buổi chèo thuyền năm1929đã khởi đầu cho tình bạn thân thiết suốt đời Trong thư dài mà họ thẳng thắn trao đổi, Alexandrov phê phán Kolmogorov ý thích nói chuyện với người lạ tàu, ngụ ý gặp gỡ q hời hợt, khơng giúp nhận diện tính cách thực người Kolmogorov phản đối, ông đưa quan điểm xác suất cấp tiến tương tác xã hội người hành động mẫu thống kê đại diện cho nhóm lớn Ông viết hồi âm cho Alexandrov “một cá nhân có xu hướng hấp thu tinh thần xung quanh, thể với quanh mình, không với người bạn định, phong cách sống giới quan mà họ hấp thu được

Kolmogorov quan tâm sâu sắc tới âm nhạc, văn chương ơng tin phân tích chúng khía cạnh xác suất để thu hiểu biết sâu sắc cách tư bên trí óc người Ơng người tin vào tính thứ bậc văn học nghệ thuật Ở đỉnh tháp tác phẩm Goethe, Pushkin, Thomas Mann với sáng tác Bach, Vivaldi, Mozart Beethoven, cơng trình có giá trị trường tồn tương tự chân lý toán học vĩnh cửu Kolmogorov nhấn mạnh cơng trình nghệ thuật đích thực sáng tạo độc nhất, thứ khơng dự đốn được, nằm ngồi địa hạt chuẩn mực thống kê đơn giản “Liệu xếp cách hợp lý tác phẩm Chiến tranh Hòa bình Tolstoy vào chung tập hợp ‘tất tiểu thuyết sinh đời’, là thiết lập phân bố xác suất cho phần tử tập hợp hay không?”, ông hỏi đùa báo in năm1965

(107)

khác, Kolmogorov lập luận họ biến tấu vần luật để tạo “sắc thái tổng thể” cho thơ hay đoạn văn

Để đo giá trị nghệ thuật văn bản, Kolmogorov sử dụng phương pháp đốn chữ để đánh giá entropy ngơn ngữ tự nhiên Trong lý thuyết thông tin, entropy thước đo tính bất định tính khơng dự đốn được, tương ứng với nội dung thơng tin thơng điệp: Thơng điệp khơng thể dự đốn thơng tin mà hàm chứa nhiều Kolmogorov đưa entropy thành thước đo tính độc đáo nghệ thuật Nhóm ơng đặt chuỗi phép thử, tình nguyện viên xem trích đoạn văn xi thơ ca Nga, yêu cầu họ đoán chữ tiếp theo, nữa, tiếp tục Kolmogorov ngầm nhận xét rằng, từ góc nhìn lý thuyết thơng tin, tờ báo Xơ viết thường thơng tin thơ ca, diễn thuyết trị thường sử dụng nhiều cụm từ có tính khn sáo nội dung chúng dễ đoán trước Trái lại, thơ nhà thơ vĩ đại lại khó đốn nhiều, chúng phải tuân thủ quy phạm chặt chẽ theo thể thơ Theo Kolmogorov, biểu tính độc đáo Nghệ thuật đích thực khơng đốn trước được, phẩm chất lại đo lường lý thuyết xác suất có chất lượng cao

Kolmogorov khơng thể chấp nhận việc coi Chiến tranh Hịa bình phần tử nằm chung tập hợp tất tiểu thuyết – ơng biểu đạt tính khơng thể dự đốn cách tính tốn độ phức tạp Kolmogorov coi độ phức tạp đối tượng độ dài mơ tả ngắn nó, độ dài thuật toán tạo đối tượng Những đối tượng tất định đơn giản theo nghĩa chúng sinh từ thuật tốn ngắn chuỗi tuần hoàn số Những đối tượng thực ngẫu nhiên, dự đốn phức tạp, thuật tốn sinh chúng phải dài thân chúng Ví dụ, số vơ tỷ - số viết dạng phân số - dãy chữ số đằng sau dấu thập phân xuất ngẫu nhiên khơng có quy luật Bởi vậy, hầu hết số vô tỷ đối tượng phức tạp chúng ghi lại cách viết toàn dãy chữ số Cách hiểu độ phức tạp phù hợp với ý niệm trực quan khơng có phương pháp hay thuật tốn dự đốn đối tượng ngẫu nhiên Khái niệm ngày quan trọng vai trị thước đo tài ngun tính tốn cần có để biểu đạt đối tượng, đồng thời có nhiều ứng dụng định tuyến mạng đại, thuật toán xếp nén liệu

(108)(109)

MỞ RỘNG BỔ ĐỀ SAWAYAMA VÀ ĐỊNH LÝ

SAWAYAMA-THEBAULT

Đào Thanh Oai

TÓM TẮT

Trong viết này, tác giả đề xuất không chứng minh hai mở rộng bổ đề Sawayama, một mở rộng định lý Sawayama-Thebault Tác giả đưa nhận xét cịn có nhiều biến thể trường hợp đặc biệt mở rộng minh họa thông qua vấn đề thứ ba

1 Mở đầu

Bổ đề Sawayama định lý Sawayama-Thebault giới thiệu chi tiết viết tác giả người Pháp Jean-Louis Ayme [1] Một số ứng dụng bổ đề Sawayama định lý Sawayama-Thebault kỳ thi toán Olympic đề cập đến viết hai tác giả Trần Quang Hùng Dương Ánh Ngọc [2] Chính tác giả viết đưa mở rộng khác cho bổ đề Sawayama định lý Sawayama-Thebault [3]

Trong viết này, tác giả đề xuất khơng chứng minh hai vấn đề chính, vấn đề thứ mở rộng bổ đề Sawayama, vấn đề thứ hai mở rộng định lý Sawayama-Thebault Ngoài tác giả đưa nhận xét cịn có nhiều biến thể khác hai vấn đề này, ví dụ đưa vấn đề

Khơng giống với hầu hết định lý hình học cổ điển khác việc mở rộng bổ đề Sawayama định lý Sawayama-Thebault khó khăn, tác giả bắt đầu tìm cách mở rộng từ năm 2013 đến sau khoảng năm tìm kiếm mở rộng bổ đề Sawayama định lý Sawayama-Thebault thành công, tác giả đem giới thiệu đến với bạn đọc

Trước vào phần viết, tác giả nêu hai định nghĩa

- Đường tròn(O)được gọi tiếp xúc với hai cạnhAB,ACnếu(O)vàB phía với AC,(O)vàC phía vớiAB

(110)

2 Mở rộng thứ bổ đề Sawayama

Vấn đề 1.Cho tam giácABC vớiI là tâm đường tròn nội tiếp.(O)là đường tròn bất kỳ

quaB,C.(OA)là đường tròn tiếp xúc với cạnhAB,AC(O), cho điểm tiếp xúc của

hai đường tròn (OA),(O)Acùng thuộc nửa mặt phẳng chia bởi BC ChoP là điểm trong

mặt phẳng nằm ngồi đường trịn(OA), quaP kẻ đường thẳng`tiếp xúc với(OA) Gọi

(O1)là đường tròn tiếp xúc vớiBC,`(O1), cho:

1 Nếu(OA)tiếp xúc vớiAB,AC và tiếp xúc ngồi với(O)thì đường trịn (O1)(OA)

khác phía với`(Hình 1).

2 Nếu(OA)tiếp xúc ngồi vớiAB, AC và tiếp xúc với(O)thì đường trịn(O1)(OA)

cùng phía với`(Hình 2).

Tiếp điểm của(O1)BC,`lần lượt làD,E Chứng minh rằngD,E,I thẳng hàng.

Hình 1

3 Mở rộng định lý Sawayama-Thebault

Vấn đề 2.Cho tam giácABC vớiI là tâm đường tròn nội tiếp.(O)là đường tròn bất kỳ

quaB,C.(OA)là đường tròn tiếp xúc với cạnhAB,AC(O), cho điểm tiếp xúc của

hai đường tròn(OA), (O)và điểmA cùng thuộc nửa mặt phẳng chia bởiBC Cho P là điểm

trong mặt phẳng nằm ngoài (OA), quaP kẻ hai đường thẳng`1, `2 tiếp xúc với (OA).

Gọi(O1)là đường tròn tiếp xúc vớiBC,`1 (O), gọi(O2)là đường tròn tiếp xúc vớiBC,`2

(O)sao cho:

1 Nếu(OA)tiếp xúc vớiAB,AC và tiếp xúc ngồi với(O)thì(O1) (OA)khác phía

với`1,(O2)(OA)khác phía với`2.

2 Nếu(OA)tiếp xúc ngồi vớiAB,AC và tiếp xúc với(O)thì(O1) (OA)cùng phía

với`1,(O2)(OA)cùng phía với`2.

Chứng minh đường thẳngO1O2sẽ qua điểm cố định khiP di chuyển đường

(111)

Hình 2

4 Biến thể

Vấn đề 3. Cho tam giác ABC với I là tâm đường tròn nội tiếp, EA là tâm đường tròn bàng

tiếp gócA. (O)là đường trịn quaB, C.(OA)là đường tròn tiếp xúc với các

cạnhAB,AC và tiếp xúc (hoặc ngoài) với(O), cho tiếp điểm của(OA),(O)và điểm Akhông thuộc nửa mặt phẳng chia bởiBC ChoP là điểm mặt phẳng nằm

ngồi(OA), quaP kẻ đường thẳng`tiếp xúc với(OA).(O1)là đường trịn tiếp xúc vớiBC,`

(O)sao cho (O1) (OA)cùng phía với ` Tiếp điểm của(O1) BC, `lần lượt làD,E.

Chứng minh rằng

1.D,E,I thẳng hàng nếu(O),(OA)tiếp xúc (Hình 3)

2.D,E,EAthẳng hàng nếu(O),(OA)tiếp xúc ngồi (Hình 3)

(112)

5 Mở rộng thứ hai bổ đề Sawayama

P, Q hai điểm đẳng giác với tam giác ABC AP, AQ cắt đường tròn ngoại tiếp tam giác ABC tạiD,E Hai đường thẳng quaD,E cắt đường tròn ngoại tiếp hai điểmT,N cắtBC hai điểmG,H.P G,HQcắt đường trịn(GHN T)tạiK,F Khi đóK,F,Athẳng hàng

Hình 4

Tài liệu

[1] Jean-Louis Ayme, Sawayama and Thébault’s Theorem, Forum Geometricorum, (2003) 225–229

[2] Trần Quang Hùng, Dương Ánh Ngọc, Định lý Sawayama Thébault tốn hình học thi Olympic, Tạp chí Epsilon, Số 09, 06/2016

(113)

ĐƯỜNG THẲNG STEINER ĐIỂM ANTI-STEINER

Ngơ Quang Dương, ĐHKHTN-ĐHQGHN

TĨM TẮT

Trong Epsilon số 7, tác giả có viết đường thẳng Simson Như tiếp nối, xin đem tới bạn đọc viết đường thẳng Steiner điểm Anti-Steiner

1 Đường thẳng Steiner

1.1 Đường thẳng Steiner tam giác

Định lý 1(Steiner). P nằm đường trịn ngoại tiếp4ABC thì đối xứng củaP qua ba cạnh

tam giác trực tâm4ABC thẳng hàng.

Chứng minh. Pa,Pb,Pclà đối xứng củaP quaBC,CA,AB,Hlà trực tâm4ABC Lưu ý

(114)

Từ đó,Palần lượt nằm trên(HBC), tương tự,Pb thuộc(HCA)vàPcthuộc(HAB)

(HPb, HPc) = (HPb, HA) + (HA, HPc) (mod π)

= (BPb, BA) + (CA, CPc) (mod π)

= (BA, BP) + (CP, CA) (mod π)

= (mod π)

VậyPb,Pc,Hthẳng hàng Mà từ đường thẳng Simson,Pa,Pb,Pcthẳng hàng nênH,Pa,Pb,Pc thẳng hàng

Định lý 2. Parabol nhậnP làm tiêu điểm đường thẳng Steiner củaP làm đường chuẩn thì

tiếp xúc với ba cạnh tam giác.

Chứng minh. Thực ra, chất tính chất lại tính chất quang hình học parabol:

Cho trước parabola có tiêu điểmF, đường chuẩn`và điểmM trên parabol Tiếp tuyến của

parabol tạiM là phân giác ngồi góc tạo bởiM F và tiaM xvng góc với đường chuẩn,

khơng cắt đường chuẩn.

Ở xin đưa chứng minh không sử dụng tọa độ GọiHlà hình chiếu vng góc củaM lên đường chuẩn thìF vàH đối xứng qua phân giác ngồi củaF M x\ Giả sử đường phân giác ngoàiF M x\cịn điểm chungM0 nữa với parabol thìM0F = d(M0, `) MàM0F = M0H nên M0H = d(M0, `) - điều chứng tỏ M0 trùng M Vậy phân giác của F M x\ tiếp xúc parabol Ngoài ta phát biểu tính chất quang hình học theo cách khác:H nằm trên

đường chuẩn trung trựcF H tiếp xúc parabol.

(115)

1.2 Đường thẳng Steiner, đường thẳng Newton tứ giác toàn phần

Ta định nghĩa tứ giác toàn phần hình phẳng, bao gồm đường thẳng đơi cắt giao điểm chúng, đường số khơng đồng quy Tứ giác tồn phần kí hiệu chuẩn đường thẳng cấu thành nên Trong nhiều tài liệu khác lại kí hiệu tứ giác tồn phần đỉnh chúng - cách kí hiệu ngắn gọn, song dễ gây nhầm lẫn thứ tự đỉnh Trong mục này, ta xét tứ giác toàn phần tạo đường thẳnga,b,c, dvà ta kí hiệu tứ giác tồn phần là(a, b, c, d)

Định lý 3. Trực tâm4bcd,4cda,4cda,4abcthẳng hàng Trong đó4bcdlà tam giác tạo bởi

3 đường thẳngb,c,d.

Chứng minh. Theo định lý Miquel đường trịn đồng quy ta có đường tròn ngoại tiếp tam giác thành phần 4bcd, 4cda, 4dab, 4abc đồng quy điểm M(điểm Miquel tứ giác toàn phần) Từ lấy đối xứngM qua cạnh tứ giác toàn phần, theo định lý ta có điều phải chứng minh

Định lý 4 (Đường tròn Miquel). Điểm Miquel của (a, b, c, d), tâm ngoại tiếp tam giác

(116)

Dựa theo J.W.Clawson. Ở tác giả đưa cách chứng minh mẻ nghịch đảo - thấy mối liên hệ đường tròn với đường thẳng Steiner Chỉ đơn giản cộng góc, ta thu cặp tam giác sau đồng dạng thuận: 4M PdbPdc ∼ 4M PabPca, 4M PdcPbc ∼ 4M PdaPbc Điều dẫn tới hai hệ quan trọng:

M Pbc·M Pda =M Pca·M Pdb =M Pab·M Pdc=k

và góc∠(M Pbc, M Pda),∠(M Pca, M Pdb),∠(M Pab, M Pdc)có chung phân giác` Xét phép biến hìnhΨlà hợp phép nghịch đảo cựcM, phương tíchkvà phép đối xứng qua` Gọi M A0,M B0,M C0,M D0là đường kính đường trịn ngoại tiếp4bcd,4cda,4dab,4abc.

Ψ :Pbc, Pda, Pca, Pdb, Pab, Pdc7→Pda, Pbc, Pdb, Pca, Pdc, Pab

Ψ : (PdbPdcPbc)→Pca, Pab, Pad Do đó,Ψ(A0)là hình chiếu vng góc củaM lên đường thẳngP

ca, Pab, Pad, hay tương đương qua phép biến hìnhΨ, tâm đường tròn ngoại tiếp4bcdthành đối xứng củaM quaa Tương tự, Ψbiến tâm ngoại tiếp tam giác thành phần thành đối xứng điểm Miquel qua

cạnh tứ giác tồn phần Do tâm đường tròn ngoại tiếp điểm Miquel thuộc đường tròn

Chú ý. (1) Theo định lý 2, có parabol tiếp xúc với cạnh tứ giác tồn phần và parabol có tiêu điểm điểm Miquel, đường chuẩn đường thẳng Steiner.

(2) Phép biến hìnhΨđược giới thiệu lần đầu J.W.Clawson báo AMM vào

năm 1919[3] có nhiều ý nghĩa quan trọng việc tìm hiểu đối tượng tứ giác tồn phần việc giải nhiều toán.Ψbiếna,b,c,dlần lượt thành đường trịn ngoại

tiếp4bcd,4cda, 4dab,4abc Từ ta thu hệ đẹp tứ giác toàn phần ngoại tiếp

(117)

Định lý 5(Đường thẳng Newton đường thẳng Steiner). Đường thẳng Steiner vng góc với đường thẳng qua trung điểm củaPdaPbc,PdbPca,PdcPab.

Chứng minh. Ha, Hb, Hc, Hd trực tâm 4bcd, 4cda, 4dab, 4abc GọiAd, Bd, Cdlà hình chiếu vng góc củaHalên đường thẳngb,c,d Với trực tâm ta ln có

HaPbc·HaAd=HaPdc·HaBd =HaPdb·HaCd

Điều tương đương với việcHacó phương tích với đường trịn đường kínhPbcPda, PcaPdb,PabPdc Tương tự vớiHb,Hc,Hdlà có điều phải chứng minh

Chú ý. Nói thêm lịch sử Đường thẳng Steiner trục đẳng phương ba đường tròn đường kính PbcPda, PcaPdb, PabPdc chính nội dung định lý Gauss-Bodenmiller Trước Gauss,

Newton khám phá conic tiếp xúc với cạnh tứ giác tồn phần có tâm thuộc đường thẳng(các bạn học sinh biết tới sử dụng trường hợp đặc biệt conic là đường tròn tiếp xúc cạnh tứ giác tồn phần) Đường thẳng Newton gọi là đường thẳng Newton-Gauss hay đường thẳng Gauss.

Đường thẳng Steiner qua số điểm đặc biệt khác

Định lý 6. Một số điểm khác đường thẳng Steiner tứ giác toàn phần.

1 (A.Dixit, D.Grinberg) Cực trực giao củaa,b,c,dlần lượt với4bcd,4cda,4dab,4abc.

2 (Điểm Morley) Na, Nb, Nc, Nd là tâm đường trịn chín điểm của 4bcd, 4cda, 4dab, 4abc Đường thẳng quaNa,Nb,Nc,Nd và vng góc vớia,b,c, dđồng quy một

điểm đường thẳng Steiner của(a, b, c, d).

(118)

1 Dd cực trực giao củad với 4abc Khi đó, BdDd, CdDd vng góc với b, c Dễ thấy 4BdCdDdvà4PdcPdbHacó cạnh tương ứng song song nên có phép vị tự biến4BdCdDdthành4PdcPdbHa, tâm phép vị tự làT - giao điểm củaDdHavới d Theo định lý Thales

T Ha T Dd

= T Pdc

T Bd

= Tdb

T Cd

do đóT Pdc·T Cd =Tdb·T Bd nênT thuộc trục đẳng phương hai đường trịn đường kính PabPdc PcaPdb Điều có nghĩa Dd thuộc trục đẳng phương hai đường tròn này, tức làDdthuộc đường thẳng Steiner của(a, b, c, d)

(119)

−−−→

NaPbc+−−−→NaPdb+−−−→NaPdc = 3−−−→NaGa =−−−−→NaHa

Vì Na trọng tâm điểm Pbc, Pdb, Pdc, Ha Kí hiệu Pja phép chiếu theo phương vng góc vớiaxuống đường thẳng Steiner thì:

Pja:Pbc, Pdb, Pdc, Ha, Na 7→Hd, Hc, Hb, Ha, N

Do phép chiếu vector bảo toàn số tâm tỉ cự nên −−−→N Ha+−−→N Hb+N H−−→c+−−→N Hd = −→0 Do vậyN trọng tâm củaHa,Hb,Hc,Hd Chứng minh tương tự, đường thẳng quaNb, Nc,Nd, vng gócb,c,dđều quaN

Chú ý. Đối với tất bạn học sinh, định lý 6.2 biết tới IMO Shortlist 2009 nhưng thực tế phát đề cập Frank Morley(đây tác giả định lý tam giác từ việc chia ba góc tiếng) báo [2] vào năm 1903 -sớm tới kỉ.

2 Điểm Anti-Steiner

(120)

Định lý 7 (Định lý Collings). `là đường thẳng qua trực tâm4ABC, đối xứng của `qua

ba cạnh của4ABC đồng quy điểm trên(ABC).

Chứng minh. ` cắt BC, CA, AB D, E, F Ha, Hb, Hc đối xứng với H quaBC, CA, AB Do (HB, HC) = (AC, AB), (HC, HA) = (BA, BC), (HA, HB) = (CB, CA)

nênHa,Hb,Hcthuộc(ABC)

(EHb, F Hc) = (EHb, CA) + (AC, AB) + (AB, F Hc) (mod π)

= (CA, `) + (AC, AB) + (`, AB) (mod π) = 2(AC, AB) (mod π)

= (HaHb, HaHc) (mod π)

Từ điều dẫn đến giao điểm củaEHb,F Hcthuộc(ABC) Tương tự, giao điểm củaF Hcvà DHathuộc(ABC)nênDHa,EHb,F Hcđồng quy trên(ABC)

Chú ý. Điểm đồng quy gọi điểm Collings điểm Anti-Steiner đường thẳng`.

Có thể dễ dàng nhận thấy đối xứng điểm qua ba cạnh của4ABC đều thuộc`.

Ngoài ra, người ta gọi điểm Anti-Steiner đường thẳngHP là điểm Anti-Steiner củaP,

hay điểm Collings củaP.

Định lý 8. Cho điểmA, B, C, P. Pa, Pb, Pc đối xứng với P quaBC, CA, AB. P A, P B, P C cắt(ABC)tạiD, E,F Khi đó(APbPc),(BPcPa),(CPaPb),(P PaD),(P PbE),(P PcF)

(121)

Chứng minh. LấyHlà trực tâm4ABCvàHa,Hb,Hcđối xứng vớiHquaBC,CA,AB Theo định lý Collings,PaHa,PbHb,PcHcđồng quy điểmS Chỉ cần chứng minh(APbPc)và

(P PaD)đi quaSlà đủ Chỉ đơn giản cộng góc

(SPb, SPc) = (HbPb, HcPc) (mod π)

= (HbPb, AC) + (AC, AB) + (AB, HcPc) (mod π)

= (AC, HP) + (AC, AB) + (HP, AB) (mod π) = 2(AC, AB) (mod π)

= (APb, APc) (mod π)

(SD, SPa) = (SD, SHa) (mod π)

= (AD, AHa) (mod π)

= (P D, P Pa) (mod π)(AHasong songP Pa) Từ kết luận(APbPc),(P PaD)và tương tự, đường trịn lại quaS Điểm Anti-Steiner xuất chứng minh định lý Fontene

Định lý 9(Định lý Fontene). 4ABC có tâm ngoại tiếpO.P là điểm bất kì. D,E, F là hình

chiếu vng góc củaP lênBC,CA,AB.EF,F D,DElần lượt cắt đường trung bình ứng với A,B,Ccủa4ABC tạiX,Y,Z.

(122)

2 KhiP chạy đường thẳng quaOthì(DEF)ln qua điểm cố định.

Trong định lý 9, điểm đồng quy điểm Anti-Steiner củaP với tam giác có đỉnh trung điểmBC, CA, AB Định lý 9, với số cấu hình khác có xuất điểm Anti-Steiner, bạn đọc tham khảo viết tác giả [4]

Một điều đáng nói điểm Anti-Steiner cịn có mặt số tốn tiếp xúc Ở giới thiệu phát đẹp kĩ sư Đào Thanh Oai

Định lý 10. [5]Cho4ABC và điểmP bất kì.

Đường thẳng quaP và vng gócBC cắtAB,ACtạiAc,Ab.

Đường thẳng quaP và vng gócCAcắtBC,BAtạiBa,Bc.

Đường thẳng quaP và vng gócABcắtCA,CBtạiCb,Ca.

(P CbAb)cắt(P AcBc)tạiDkhácP.

(P AcBc)cắt(P BaCa)tạiEkhácP.

(P CbAb)cắt(P AcBc)tạiF khácP.

Khi đó(DEF)tiếp xúc(ABC)tại điểm Collings củaP.

Chứng minh. Trước hai đường trịn tiếp xúc, ta phải được(DEF)đi qua

điểm CollingsS Ở đây, để tạo liên kết hình học có liên quan tới điểmS, ta lấyPa,Pb,Pc điểm đối xứng vớiP quaBC,CA,AB Các điểm sau đồng viên

(123)

Có thể giải thích điều cách đơn giản Đầu tiên theo định lý 8, với việcP trực tâm của4ABcCb màPb,Pcđối xứng vớiP quaACb,ABcnênPb,Pcthuộc(ABcCb)

(DBc, DCb) = (DBc, DP) + (DP, DCb) (mod π)

= (AcBc, AcP) + (AbP, AbCb) (mod π)

= (AB, AC) (mod π)

= (ABc, ACb) (mod π)

Do vậyDthuộc(ABcCb)

Sử dụng điểm đồng viên trên, với định lý đủ để sử dụng góc định hướng Ta ra(DE, DF) = (SE, SF) (mod π)

(DE, DF) = (DE, DP) + (DP, DF) (mod π)

= (AcE, AcP) + (AbP, AbF) (mod π)

= (AcE, AbF) (mod π)

= (AcE, AcB) + (AcB, AbC) + (AbC, AbF) (mod π)

= (SE, SB) + (AB, AC) + (SC, SF) (mod π) = (SE, SB) + (SB, SC) + (SC, SF) (mod π) = (SE, SF) (mod π)

Như vậyS,D,E,F đồng viên.Stlà tiếp tuyến tạiScủa(DEF), cần chứng minhStcũng tiếp tuyến của(ABC)là đủ

(St, SA) = (St, SD) + (SD, SA) (mod π) = (ES, DE) + (CbD, CbA) (mod π)

= (ES, EAc) + (EAc, ED) + (CbD, CbAb) (mod π)

= (BS, BAc) + (P Ac, P D) + (P D, P Ab) (mod π)

= (BS, BA) (mod π)

Do ta kết luậnStlà tiếp tuyến của(ABC), thế(ABC)và(DEF)tiếp xúc tạiS

Kết thúc viết tác giả đưa số toán đề nghị

Bài 1. Cho4ABC và điểmP không nằm trên(ABC). P A,P B, P C cắt(ABC)tạiD, E, F khácA,B, C Chứng minh tam giác tạo trung trực P D,P E, P F thấu xạ với 4ABC tại điểmT trên(ABC)P T đi qua điểm Anti-Steiner củaP với4ABC. Bài 2. Cũng với kí hiệu 1, chứng minh trục thấu xạ của4ABC và tam giác tạo

bởi trung trực củaP D,P E,P F là trung trực củaSP.

Tài liệu

(124)

[2] F Morley, Orthocentric properties of the plane n-Line, Trans Amer Math Soc, (1903) 1-12

http://www.ams.org/journals/tran/1903-004-01/

S0002-9947-1903-1500618-2/S0002-9947-1903-1500618-2.pdf

[3] J.W.Clawson, The complete quadrilateral, American Mathematics Monthly, volume 20, 1919, pages 232-262

http://www.jstor.org/stable/1967118

[4] Quang Duong’s blog,Nine-point circle, pedal circle and cevian circle

https://blogcuaquangduong.blogspot.com/2015/08/ nine-point-circle-pedal-circle-cevian.html

[5] Dao Thanh Oai, Advanced Plane Geometry

(125)

VỀ BÀI TOÁN TAM GIÁC 80-80-20 (TIẾP THEO)

Lê Phúc Lữ

(Thành phố Hồ Chí Minh)

GIỚI THIỆU

Tiếp theo Epsilon số 9, viết này, xem xét tiếp lời giải cho toán tam giác80−80−20 Qua 10 lời giải cho toán thú vị này, ta thấy hầu hết cách tiếp cận tìm cách dựng tam giác khai thác tính chất đặc biệt mơ hình Bên cạnh đó, đến việc mở rộng số giả thiết tương tự khác

Cách 6.(của Alexander Kornienko)

GọiB0 điểm đối xứng vớiBquaACvàC0 điểm đối xứng vớiCquaAB.Khi đó, dễ thấy rằngAB0 =AB=AC =AC0nên tam giácAB0C0cân.

Ngồi ra,6 B0AC0 = 20◦+ 20◦+ 20◦ = 60◦ nên tam giácAB0C0 Vì6 AB0D=6 ABD=

30◦nênB0Dlà phân giác của6 AB0C0hayB0Dlà trung trực củaAC0.

(126)

Do đó,E ∈DB0 hayE, D, B0, M thẳng hàng vớiM là trung điểm củaAC0. Vậy ta có

6 CED = 180◦−(6 C0EM +6 C0EB+6 BEC) = 180◦−(70◦+ 40◦+ 40◦) = 30◦.

Cách 7.(của Sergey Saprikin)

GọiT giao điểm phân giác6 ACBvớiAB Ta có:

6 BT C = 180◦−(6 T BC+6 T CB) = 180◦−(80◦+ 40◦) = 60◦.

Ngoài ra, dễ thấy∆BT C = ∆DT C(c.g.c)nên6 DT C = 60◦

Do đó,T Elà phân giác ngồi góc6 DT C.Xét tam giácDT C cóE giao điểm phân giác của6 DT C phân giác của6 DCT nênElà tâm đường tròn bàng tiếp gócC Do đó6 EDT =6 EDA

Cũng tam giácDT C, ta có6 DT C = 60◦,6 DCT = 40◦nên6 CDT = 80◦ Do

6 EDT =6 EDA= 180

◦−80◦

2 = 50

(127)

nên

6 EDC = 50◦+ 80◦ = 130◦.

Vậy ta có

6 CED = 180◦−(130◦+ 20◦) = 30◦.

Cách 8.(bởi Luke Rapley)

Đường trịn tâmC,bán kínhCEcắtCA, CB tạiM, N

Tam giácCEN cân tạiC,có6 ECB = 60◦nên tam giác Suy raEN =EC.Hơn nữa, tam giácEAC cân tạiE nênEA=EC =EN

Ngoài ra,6 BEN = 60◦−40◦ = 20◦ =6 EAM

Xét hai tam giácBEN vàM AE có:6 BEN =6 EAM,EN =AE BE =AB−AE =AC−EC=AC−CM =AM

Do đó, hai tam giác nhau, suy BN = EM Mặt khác M D = CM −CD =

CN −CB =BN nênM E =M D hay tam giácM ED cân tạiM Chú ý

(128)

Ta tính được6 M ED =6 M DE = 180◦−80◦

2 = 50◦ Do đó6 CED =6 CEM −6 DEM = 30◦.

Cách 9.(bởi Mariano Perez)

TrênABlấy điểmF choCF =CB Khi đó, ta cóCD =CB = CF nên tam giácCDF cân Hơn nữa,6 F CB = 180◦−2·80◦ = 20◦ nên6 F CD = 80◦−20◦ = 60◦.Suy tam giác CDF

XétK ∈AC cho6 CBK = 60◦và đặtP giao điểm củaCE, BK.

Khi đó, ta cóCBP nên tồn phép quay tâmC biếnCBP thànhCF D.Do đó, gọi M giao điểm củaDF, BP thìCM phân giác của6 ACB.

Xét phép đối xứng qua trụcCM, ta có:P biến thànhF,Dbiến thànhB Giả sửK biến thành K0thìK0 ∈BC

Ta có:

6 F CK0 =6 P CK= 20◦,6 CF K0 =6 CP K = 120◦.

(129)

Chú ý tam giác CP K vàEK = EP, DK =DP nênEDlà trung trực củaP K Vì nênEDcũng phân giác của6 P EK

Vậy ta có

6 CED =6 P ED = 60◦

2 = 30

◦.

Cách 10.Ta giải toán tổng quát Đặt6 A= 2a,6 DBC =b,6 ECB =c,6 CED =x.

Theo định lý sin, ta có

CD

sinx =

CE

sin(90◦+a+c−x),

CE

sin(90◦−a) =

CB

sin(90◦−a−c),

CD

sinb =

BC

sin(90◦+a−b)

Từ đó, ta suy đẳng thức

cos(a+c−x)

sinx =

cos(a−b) cosa

(130)

Mặt kháccos(a+c−x) = cos(a+c) cosx+ sin(a+c) sinxnên

cos(a+c) cotx+ sin(a+c) = cos(a−b) cosa

cos(a−c) sinb

⇔cos(a+c) cotx+ sin(a+c) = cos(a−b) cosa

cos(a−c) sinb

⇔cotx= cos(a−b) cosa

cos(a+c) cos(a−c) sinb −tan(a+c) Vớia= 10◦, b= 50◦, c = 60◦, ta có

cotx= cos 40

◦cos 10◦

cos 70◦cos 50◦sin 50◦ −tan 70◦ =

cos 10◦

cos 70◦cos 50◦ −tan 70◦

= sin 100◦−sin 70◦cos 50◦

cos 70◦cos 50◦ =

2 sin 50◦−sin 70◦

cos 70◦ =

2 sin 130◦−sin 70◦

cos 70◦

= sin 60

◦cos 70◦+ cos 60◦sin 70◦−sin 70◦

cos 70◦ =

3 cos 70◦

cos 70◦ =

3

Do đóx= 30◦.

Nhận xét.Từ đẳng thức trên, ta xây dựng số trường hợp cho gócxđẹp cho tam giác80−80−20này:

STT Gócb Gócc Gócx

1 20 50 10

2 40 50 30

3 30 60 10

4 50 60 30

5 25 65

6 60 65 40

7 50 70 10

8 60 70 20

Chúng giới thiệu lời giải cho số trường hợp vào số Các thông tin tham khảo từ trang web:

http://www.qbyte.org/puzzles/p022s.htmlvà

(131)

GIỚI THIỆU VỀ KỲ THI HỌC BỔNG DU HỌC NGA

Lê Phúc Lữ

(Thành phố Hồ Chí Minh)

GIỚI THIỆU

Chuỗi thi Olympic "Đã đến lúc du học Nga" dự án nhóm trường Đại học Liên bang Nga (LB Nga), quan hợp tác LB Nga Bộ giáo dục&Khoa học LB Nga tổ chức từ năm 2013 đến

Mục đích thi Olympic nhằm tăng cường thu hút người quan tâm lựa chọn LB Nga để du học Ban tổ chức chọn thí sinh xuất sắc tốt nghiệp THPT ĐH để cấp học bổng du học trường đại học LB Nga

Dành chiến thắng thi Olympic, thí sinh cấp học bổng Chính phủ LB Nga theo học ngành kỹ thuật theo trường đề cập mục

Các thi tuyển sinh viên cho năm học 2016-2017 diễn nước: Abkhazia, Angola, Armenia, Việt Nam, Ấn Độ, Kazakhstan, Trung Quốc, Moldova, Ethiopia, Zam-bia, Kenya, Namibia

Tại Việt Nam, năm 2015 có tổ chức cho 600 thí sinh địa điểm Hà Nội, Đà Nẵng TP Hồ Chí Minh, tuyển chọn 50 thí sinh Năm 2016 tổ chức địa điểm Hà Nội, Đà Nẵng Hịa Bình

1 Giới thiệu đề thi mẫu đề tham khảo tương tự

(132)(133)

1.2 Đề tham khảo số 1

Bài 1 Tính(1 +√5)3+ (1−√5)3.

Bài 2 Tìm tổng ước số chung lớn bội số chung nhỏ số8436.

Bài 3 Một cạnh hình chữ nhật có chiều dài là4, đường chéo có chiều dài là√41 Tính chu

vi hình chữ nhật.

Bài 4 Tìm số nghiệm phân biệt phương trìnhx6 −5x4+ 4x2 = 0.

Bài 5 Tìm số tự nhiênN biết rằng:

1 SốN chia cho74

2 SốN chia hết cho11

Bài 6 Tính tổng + 42+ 43+ 44+· · ·+ 450

450−1 .

Bài 7 Một tam giác có diện tích là16√3 Tính diện tích hình trịn nội tiếp tam giác.

Bài 8 Biết rằng

2 < α <2πsinα=

5, tìmsin 2α.

Bài 9 Tìm số nguyên lớn thỏa mãn bất đẳng thức5√x <25.

Bài 10 Một hình thang vng có góc nhọn bằng arctan 5, độ dài hai đáy là8 6 Tìm diện

tích hình thang.

Bài 11 Tìm giao điểm(x0, y0)các đường thẳngx−3y=−24x+ 5y = 9.

Bài 12 Tìm diện tích đa giác mặt phẳng tọa độOxy thỏa mãn bất đẳng thức|x+ 6|+

|y−3| ≤2.

Bài 13 Tìm nghiệm phương trìnhlog3x+ log3x2+ log

3x3+· · ·+ log3x15= 60

(134)

Bài 15 Tìm giá trịx0 của biếnxcủa hàm sốf(x) =x2−4x+ cos

πx

4 có giá trị nhỏ nhất.

Bài 16 Bán kính đáy hình trụ gấp đơi chiều cao Tìm diện tích xung quanh của hình trụ, biết thể tích hình trụ bằng32π

Bài 17 Tìm tất giá trị tham sốa,sao cho hàm sốf(x) = log7−2axđồng biến.

Bài 18 Tìm nghiệm nguyên nhỏ phương trình2·4x−5·2x+ = 0.

Bài 19 Tìm tất giá trị tham sốa, cho phương trìnhtan2x·cot2(x−3π) = +a

có nghiệm.

Bài 20 Tìm chu kỳ nhỏ hàm sốf(x) = sin 3x−2 cosx 1.3 Đề tham khảo số 2

Bài 1 Tính 1−2√33+ 2√3 + 13.

Bài 2 Tìm tổng ước số chung lớn bội số chung nhỏ số9628.

Bài 3 Một cạnh hình chữ nhật có chiều dài là7, đường chéo có chiều dài là√65 Tính chu

vi hình chữ nhật.

Bài 4 Tìm số nghiệm phân biệt phương trìnhx6 −6x4+ 11x2−6 = 0.

Bài 5 Tìm số tự nhiênN nhỏ biết rằng:

1 SốN chia cho104

2 SốN chia hết cho13.

Bài 6 Tính tổng 3−32+ 33 −34+· · · −3100

3100−1 .

Bài 7 Một tam giác có diện tích là√3 Tính diện tích hình trịn nội tiếp tam giác.

Bài 8 Biết rằng0< α < πcosα=

(135)

Bài 9 Tìm số nguyên lớn thỏa mãn bất đẳng thức5x <

25.

Bài 10 Một hình thang vng có góc nhọn bằng arctan(1/3), độ dài hai đáy là10 4 Tìm

diện tích hình thang.

Bài 11 Tìm giao điểm(x0, y0)các đường thẳng4x−y= 4x+ 5y= 1.

Bài 12 Tìm diện tích đa giác mặt phẳng tọa độOxythỏa mãn bất đẳng thức|3x+ 9|+

|2y| ≤5.

Bài 13 Tìm nghiệm phương trìnhlog5x2+ log5x4 + log5x6+· · ·+ log5x20= 440.

Bài 14 Tìm số nguyênxnhỏ thỏa mãn bất đẳng thứclog22(3x−8) + log23x−82≥2.

Bài 15 Tìm giá trịx0 của biếnxcủa hàm sốf(x) =x3−3x+ sin

πx

2 có giá trị nhỏ nhất.

Bài 16 Bán kính đáy hình trụ bằng 1/8chiều cao Tìm diện tích xung quanh

của hình trụ, biết thể tích hình trụ bằngπ

Bài 17 Tìm tất giá trị tham sốa,sao cho hàm sốf(x) = log7−3a2xđồng biến.

Bài 18 Tìm nghiệm nguyên nhỏ phương trình4x−5·6x+ 4·9x = 0.

Bài 19 Tìm tất giá trị tham sốa, cho phương trìnhtan2(x−π)·cot2(x+ 3π) =

5−a2 có nghiệm.

Bài 20 Tìm chu kỳ nhỏ hàm sốf(x) = sin 4x−2cos2x.

1.4 Đề tham khảo số 3

Bài 1 Tính

1·2 +

1

2·3+

1

3·4 +· · ·+ 99·100.

Bài 2 Biết hai số tự nhiên có ước chung lớn là7và bội chung nhỏ là56.GọiS

là tổng hai số Tìm giá trị nhỏ có củaS

Bài 3 Một cạnh hình chữ nhật có chu vi là 34và diện tích là60 Tính độ dài đường chéo

(136)

Bài 4 Tính tổng bình phương nghiệm phương trình3x4−5x2+ = 0.

Bài 5 Tìm số tự nhiênN biết rằng:

1 SốN không chia hết cho30, không chia hết cho45và không chia hết cho75

2 SốN chia hết cho15.

Bài 6 Tính tổng + + 11 + 15 +· · ·+ 2015

4 + + 10 + 13 +· · ·+ 2017.

Bài 7 Một tam giác có chu vi đường trịn nội tiếp là Tính diện tích hình trịn

ngoại tiếp tam giác.

Bài 8 Biết rằng0< α < π

2 tanα= 3, tìmsin 2α+ cos 2α.

Bài 9 Tìm số nguyên lớn thỏa mãn bất đẳng thức5x+ 2x<133.

Bài 10 Một hình thang vng có góc tù bằngarccos(−0.6), độ dài hai đáy là137 Tìm chu

vi hình thang.

Bài 11 Tìm giao điểm(x0, y0)các đồ thịy = 2x2 y =x3+x.

Bài 12 Tìm diện tích đa giác mặt phẳngOxy thỏa mãny ≥ |x|y≤3−2|x|. Bài 13 Tìm nghiệm phương trình2x+ 4x+ 8x+ 16x+ 32x = 62

2x−1. Bài 14 Tìm số nguyênxnhỏ thỏa mãn bất đẳng thứclog22(3x−8)≤25.

Bài 15 Tìm giá trịx0 củaxđểf(x) =

1 4x

4−x3+ 4x+ sinπx

2 +

1

3cos 6πxcó GTNN.

Bài 16 Bán kính đáy hình nón bằng1/2độ dài đường sinh Tìm diện tích xung

quanh hình nón, biết thể tích bằngπ9 √

3

2 .

Bài 17 Tìm tất giá trị tham sốa,sao cho hàm sốf(x) = log2a2−axnghịch biến. Bài 18 Tìm nghiệm nguyên nhỏ củalog23(2x−3)−4log3(2x−3) + = 0.

Bài 19 Tìm tất giá trị tham sốa, cho phương trìnhtan2(x−π)+cot2(x) =a2+a

có nghiệm.

(137)

1.5 Đề tham khảo số 4

Bài 1 Tính

1·4 +

1

2·5+

1

3·7 +· · ·+ 97·100.

Bài 2 Biết hai số tự nhiên có ước chung lớn là6và bội chung nhỏ là96 GọiS

là tổng hai số Tìm giá trị lớn có củaS

Bài 3 Một cạnh hình chữ nhật có đường chéo là41và chu vi là98 Tính diện tích hình

chữ nhật này.

Bài 4 Tính tổng bình phương nghiệm phương trình2x4−3x2−1 = 0.

Bài 5 Tìm số tự nhiênN biết rằng:

1 SốN không chia hết cho26và không chia hết cho39.

2 SốN lớn hơn100và chia hết cho13.

Bài 6 Tính tổng + + + 11 +· · ·+ 2018

3 + + + +· · ·+ 2017 .

Bài 7 Một tam giác có chu vi là Tính tổng diện tích hình trịn ngoại tiếp nội tiếp

của tam giác.

Bài 8 Cho gócαthỏa mãncotα= 2, tìmsin2α−4cos2α.

Bài 9 Tìm số nguyên nhỏ thỏa mãn bất đẳng thức4x−3x >100.

Bài 10 Một hình diều có độ dài cạnh là 5,10,10,5 Biết đường chéo dài bằng 5√5 Tính diện tích hình này.

Bài 11 Tìm giao điểm(x0, y0)các đồ thịy =x4+x2y =x3+x.

Bài 12 Tìm diện tích đa giác mặt phẳngOxy thỏa mãny ≤1 +|x|y ≥3− |x|. Bài 13 Tìm nghiệm phương trình3x−9x+ 27x−81x = −240

3x+ 1.

(138)

Bài 15 Tìm giá trịx0 củaxđểf(x) =

x+

x2+ 3 + sin

πx

6 + cos 6πxcó GTNN.

Bài 16 Bán kính đáy hình nón chiều cao Tìm thể tích hình nón, biết rằng diện tích xung quanh làπ√2.

Bài 17 Tìm tất giá trị tham sốa,sao cho hàm sốf(x) = (a2−1)xnghịch biến.

Bài 18 Tìm nghiệm nguyên nhỏ phương trình4x+ 6x = 2·5x.

Bài 19 Tìm tất giá trị tham sốa, cho phương trìnhtan2(x−π) + 4cot2(x) =

5a−a2 có nghiệm.

Bài 20 Tìm chu kỳ nhỏ hàm sốf(x) = sin 2x+ cos 5x 1.6 Đề tham khảo số 5

Bài 1 Tính14(1 +√2)3+ (3−2√2)3.

Bài 2 Tìm tổng ước số chung lớn bội số chung nhỏ số10084.

Bài 3 Một cạnh hình chữ nhật có chiều dài là6, đường chéo có chiều dài là√40 Tính chu

vi hình chữ nhật.

Bài 4 Tìm số nghiệm khác phương trìnhx6−x2 = 0.

Bài 5 Tìm số tự nhiênN nhỏ biết rằng:

1 SốN chia cho171

2 SốN chia hết cho10

Bài 6 Tính tổng + 33+ 35+· · ·+ 341

342−1 .

Bài 7 Một tam giác có diện tích là√300 Tính diện tích hình trịn nội tiếp tam giác.

Bài 8 Biết rằng2< α <3sinα =

(139)

Bài 9 Tìm số nguyên lớn thỏa mãn bất đẳng thức5x x <3125.

Bài 10 Một hình thang vng có góc nhọn bằngarctan 3, độ dài hai đáy là1816 Tìm diện

tích hình thang.

Bài 11 Tìm giao điểm(x0, y0)các đường thẳngx+y= 103x−2y = 5.

Bài 12 Tìm diện tích đa giác mặt phẳng tọa độOxythỏa mãn bất đẳng thức|2x+ 5|+

|2y−5| ≤2.

Bài 13 Tìm nghiệm phương trìnhlog5x5+ log5x6 +· · ·+ log5x20 = 100.

Bài 14 Tìm số nguyênxnhỏ thỏa mãn bất đẳng thứclg2(x−1) +log2

x−110≥2.

Bài 15 Tìm giá trịx0 của biếnxcủa hàm sốf(x) =x2−2x+ sin

πx

2 có giá trị nhỏ nhất.

Bài 16 Bán kính đáy hình trụ nhỏ lần chiều cao Tìm diện tích xung quanh hình trụ thể tích hình trụ bằng80π

Bài 17 Tìm tất giá trị tham sốa,sao cho hàm sốf(x) = log4−3axlà hàm số tăng.

Bài 18 Tìm nghiệm nguyên nhỏ phương trình8·9x−10·6x−3·4x = 0.

Bài 19 Tìm tất giá trị tham sốa, cho phương trình

tan2x+π

2

·cot2

x− 3π

2

= +a

có nghiệm.

(140)

1.7 Đề tham khảo số 6

Bài 1 Tính13 √3−13− 2√3−33.

Bài 2 Tìm tổng ước số chung lớn bội số chung nhỏ số80128.

Bài 3 Một cạnh hình chữ nhật có chiều dài là60, đường chéo có chiều dài là61 Tính chu

vi hình chữ nhật.

Bài 4 Tìm số nghiệm khác phương trìnhx6−6x4 + 9x2 = 0.

Bài 5 Tìm số tự nhiênN nhỏ biết rằng:

1 SốN chia cho2120

2 SốN chia hết cho25

Bài 6 Tính tổng 53−55+ 57−59· · ·+ 599−5101

5103−1 .

Bài 7 Một tam giác có diện tích là√243 Tính diện tích hình trịn nội tiếp tam giác.

Bài 8 Biết rằng−3< α <0cosα = √2

13, tìmsin 2α.

Bài 9 Tìm số nguyên lớn thỏa mãn bất đẳng thức2x+2√x <2048.

Bài 10 Một hình thang vng có góc nhọn bằng arctan(0.1), độ dài hai đáy là30 20 Tìm

diện tích hình thang.

Bài 11 Tìm giao điểm(x0, y0)các đường thẳngx+ 3y= 03x−y= 10.

Bài 12 Tìm diện tích đa giác mặt phẳng tọa độOxythỏa mãn bất đẳng thức|x−4|+

|4x−6| ≤3.

Bài 13 Tìm nghiệm phương trình

log4

x2 + log4

1

x + log4x+ log4x

2+· · ·+ log

4x22= 125

Bài 14 Tìm số nguyênxnhỏ thỏa mãn bất đẳng thứclg2(3x−5) +log2

(141)

Bài 15 Tìm giá trịx0 của biếnxđể hàm sốf(x) = 3x2−x+ sin 9πcó giá trị nhỏ nhất.

Bài 16 Bán kính đáy hình trụ nhỏ hơn √3lần chiều cao Tìm diện tích xung

quanh hình trụ thể tích hình trụ bằng72π

Bài 17 Tìm tất giá trị tham sốa,sao cho hàm sốf(x) = log1+5axlà hàm số tăng.

Bài 18 Tìm nghiệm nguyên lớn phương trình3·2x−5(√6)x+ 2·3x = 0.

Bài 19 Tìm tất giá trị tham sốa, cho phương trình

tan2

2x− 3π

2

·cot2

2x+ 5π

2

= 2a−1

có nghiệm.

Bài 20 Tìm chu kỳ nhỏ hàm sốf(x) = sin 4x−2cos32x.

1.8 Đề tham khảo số 7

Bài 1 Rút gọn

1 + +

1

1 + + +

1

1 + + + +· · ·+

1

1 + + +· · ·+ 100.

Bài 2 Hai số có ước số chung lớn làavà bội số chung nhỏ làb.Biết rằngb−a= 20.

Tìm giá trị nhỏ có tổng hai số ban đầu.

Bài 3 Một hình hộp chữ nhật có đường chéo bằng √38và diện tích tồn phần bằng 62 Tính

thể tích hình hộp này.

Bài 4 Tính tổng nghịch đảo nghiệm phương trìnhx3−3x−1 = 0.

Bài 5 Tìm số tự nhiênN nhỏ biết rằng:

1 SốN chia dư 1, chia dư 2, chia dư 1.

2 SốN có tận 1.

Bài 6 Tính tổng 1·2 + 3·4 + 5·6 + 7·8 +· · ·+ 10002000

1·3 + 3·6 + 6·9 + 9·12 +· · ·+ 1000·3000.

(142)

Bài 8 Cho gócαthỏa mãn10< α <13sinα+√3 cosα= 2.

Bài 9 Tìm số nguyênxnhỏ thỏa mãn bất đẳng thức6x >(1 + 2x+ 3x+ 4x+ 5x)2.

Bài 10 Một hình diều có độ dài cạnh là 5,10,10,5 Biết đường chéo dài bằng 5√5 Tính diện tích hình này.

Bài 11 Có giao điểm NGUYÊN(x0, y0)các đồ thịy=x2−6x=y2−6?

Bài 12 Tìm diện tích đa giác mặt phẳngOxy bị giới hạn bởix= 1, x= 3, y = 2, y = 5.

Bài 13 Tìm nghiệm phương trìnhlog22x+ log24x+ log28x+ log216x= 14.

Bài 14 Tìm số nguyên âmxlớn thỏa mãn bất đẳng thứcsin2πx

2

<1.

Bài 15 Tìm giá trịx0 để hàm sốf(x) =

2x2+ 3x+ +√2x2+ 5x+ 5có GTNN.

Bài 16 Đường kính đáy hình trụ lần chiều cao Tìm thể tích hình trụ biết diện tích tồn phần là48π

Bài 17 Tìm giá trịanhỏ nhất, cho hàm sốf(x) =π

a x

nghịch biến.

Bài 18 Tìm nghiệm thực nhỏ phương trình4x =x+ 1.

Bài 19 Tìm tất giá trị tham sốa, cho phương trìnhsin2x+ 3cos2x=a2+a+ 1

có nghiệm.

(143)

1.9 Đề tham khảo số 8

Bài 1 Rút gọn

1−2+

1

1 + 2−3+

1

1 + + 3−4 +· · ·+

1

1 + + +· · ·+ 39−40.

Bài 2 Hai số có ước số chung lớn làavà bội số chung nhỏ làb.Biết rằngb−a= 39.

Tìm giá trị nhỏ có tổng hai số ban đầu.

Bài 3 Một hình hộp chữ nhật tổng độ dài tất cạnh là44, diện tích tồn phần là576

Tính độ dài đường chéo dài hình hộp.

Bài 4 Tính tổng bình phương nghiệm phương trìnhx3−7x2+ 11x−3 = 0

Bài 5 Tìm số tự nhiênN nhỏ biết rằng:

1 SốN chia dư 1, chia dư 2, chia dư 1.

2 SốN có tận 1.

Bài 6 Tính tổng 1·2·3 + 2·4·6 +· · ·+ 1000·2000·3000

1·3·5 + 2·6·10 +· · ·+ 1000·3000·5000.

Bài 7 Hỏi số đường chéo đa giác có 15 đỉnh đa giác khác có 10 đỉnh bao nhiêu?

Bài 8 Tìm gócαthỏa mãn4< α <7√3 sinα−cosα=−2.

Bài 9 Tìm số nguyênxlớn thỏa mãn bất đẳng thức(√7)x >3x+ 4x.

Bài 10 Một hình diều có độ dài cạnh là10,17,17,10 Biết đường chéo ngắn bằng 16.Tính diện tích hình này.

Bài 11 Có giao điểm NGUN(x0, y0)các đồ thịy= 2x2−3x= 2y2−3?

Bài 12 Tìm diện tích đa giác mặt phẳng Oxy bị giới hạn đồ thị hàm

sốy=x, y =−x+ 2, y =x+ 2, y =−x−2.

Bài 13 Tìm nghiệm phương trìnhlog120x

2 + log120

x

3 + log120

x

4 + log120

x

5 = 4.

Bài 14 Tìm số nguyên dươngxlớn thỏa mãn bất đẳng thứctan2x

10

(144)

Bài 15 Tìm giá trịx0để hàm số

f(x) = √3x−1−√x+

có GTNN.

Bài 16 Một hộp sữa hình trụ tích bằng12πvà diện tích xung quanh bằng12π Tính

chiều cao hộp sữa.

Bài 17 Tìm giá trịanhỏ nhất, cho hàm sốf(x) =a

e2

x

đồng biến.

Bài 18 Tìm nghiệm thực nhỏ phương trìnhln(x2+x+ 1) +x4+x= 0.

Bài 19 Tìm tất giá trị tham sốa, cho phương trình6sin2x−cos2x=a2+a

nghiệm.

Bài 20 Tìm chu kỳ nhỏ hàm sốf(x) = tan 10x+ cot 4x 1.10 Đề tham khảo số 9

Bài 21 Rút gọn

1·2·3 +

1

2·3·4+

1

3·4·5 +· · ·+

1 98·99·100.

Bài 22 Trong phịng thí nghiệm, có ống nghiệm có dung tích là 42ml 77ml Hỏi với hai

ống này, người ta đong dung tích nước xác nhỏ bao nhiêu?

Bài 23 Trên mảnh đất hình chữ nhật20m×30m, người ta làm đường xung quanh có bề

rộng là2m(đường xây vào phần đất bên trong) Tính diện tích đường đi.

Bài 24 Tính tổng giá trị tuyệt đối nghiệm phương trìnhx3+ =x2+ 11.

Bài 25 Tìm số tự nhiênN nhỏ biết rằng:

1 SốN là số nguyên tố.

2 SốN chia132

(145)

Bài 27 Một hình đa giác có số đo góc bằng140◦ Hỏi đa giác có tất bao

nhiêu cạnh?

Bài 28 Tìm gócαthỏa mãntanα= 3, tính giá trị củatan 2α.

Bài 29 Hỏi có số nguyênxthỏa mãn điều kiện1050<3x <1060?

Bài 30 Một hình thang cân có độ dài cạnh là3,2,3,10 Tính diện tích hình này.

Bài 31 Có giao điểm(x0, y0)các đồ thịx2+y2 = 6y=x2?

Bài 32 Hỏi diện tích đa giác mặt phẳngOxy bị giới hạn điều kiện sau bao

nhiêu:y≥0, y ≤x, y ≤4−x?

Bài 33 Tìm nghiệm phương trìnhlog2x·log4x·log8x= 36.

Bài 34 Tìm số nguyên dươngxlớn thỏa mãn bất đẳng thức2x <10x+ 20.

Bài 35 Tìm cực trịx0nhỏ củaf(x) =

x+ 13+ √x+ 93 − √2x+ 43.

Bài 36 Một bóng hình cầu tích là27π Hỏi diện tích bề mặt bóng bao

nhiêu?

Bài 37 Tìm giá trị nguyênanhỏ nhất, cho hàm sốf(x) = (π+e+a)xđồng biến.

Bài 38 Hỏi có giá trịxthỏa mãn3x+ 5x <4x?

Bài 39 Tìm tất giá trị tham sốa, cho phương trìnhtan2x+cos2x=acó nghiệm.

(146)

2 Một số thông tin tuyển sinh trường Liên bang Nga 2016.

Các thông tin tham khảo từ tài liệu Trung tâm Khoa học Văn hóa Nga Hà Nội đợt tuyển sinh 2016

Thành phố Moskva:

1 Đại học Năng lượng Moskva (MPEI)

Địa chỉ: 111250 Matxcơva, E-250, phố Rasnakazarmennaia, số 14 Email: study@mpei.ru

Website:mpei.ru

2 Đại học Tổng hợp Công nghệ Nghiên cứu quốc gia (MISIS) Địa chỉ: LB Nga, 119049, TP Moskva, Leninskiy prospect, nhà số

Website:http://en.misis.ru/vn/- tiếng Việt vàwww.misis.ru

3 Đại học Hữu nghị dân tộc (RUDN)

Địa chỉ: 117198, TP Moskva, phố Miklukho - Maklaia, số nhà Website:www.rudn.ru

4 Đại học Xây dựng quốc gia Moskva (MGSU)

Địa chỉ: 129337, LB Nga, Moskva, Jaroslavskoye Soxxe, số nhà 26 Website:www.mgsu.ruhoặcwww.vk.com/mgsu.hoặc

www.facebook.com/misí921mgsu

5 Đại học Cơng nghệ Moskva (MIREA)

Địa chỉ: đại số Vernadski, số nhà 78, Moskva, 119454, LB Nga Website:www.mirea.ru

6 Đại học kỹ thuật vật lý Moskva (MIPT)

Địa chỉ: 141700, LB Nga, tỉnh Moskva, TP Dolgoprudni, Ngõ Instituski, số nhà Website:www.mipt.ru

(147)

Địa chỉ: 105005, LB Nga, Moskva, Baumanxkaya thứ 2, số nhà 5, tòa Website:http://www.bmstu.ruhoặchttp://www.bmstu.ru/en/

8 Đại học giao thông đường sắt Moskva (MIIT) Địa chỉ: phố Obrazxova, nhà 9, tòa 9, Moskva, 127994 MIIT, tòa nhà 1, phòng 1301, Phòng hợp tác đào tạo quốc tế Website:www.miit.ru

9 Đại học công nghệ Moskva (WTU)

Địa chỉ: 119334 LB Nga, Moskva, Leninskiy Prospect, số nhà 38A Website:www.mti.edu.ru

Thành phố Zelenograd.

10 Đại học Nghiên cứu Quốc gia (MIET)

Địa chỉ: 124498, Moskva, Zelenograd, đường 4806, số Website:eng.miet.ru

Thành phố Saint Petersburg

11 Đại học Bách khoa Saint – Perterburg

Địa chỉ: LB Nga, thành phố Saint Petersburg, đường Polychekhnhitreskaia, số 29 Website:www.spbstu.ru

12 Đại học Kiến trúc – Xây dựng Saint – Pertersburg

Địa chỉ: phòng hợp tác đào tạo quốc tế, 190005, St.Petersburg, phố Krasnoarmeiskaia số 2, số nhà

Website:www.spbgasu.ru

Thành phố Novisibirsk

13 Đại học Y Quốc gia Novisibirsk (NGMU)

(148)

14 Đại học Quốc gia kiến trúc – Xây dựng Quốc gia Novosibirsk

Địa chỉ: LB Nga, 630008, thành phố Novosibirsk, phố Leningradskaia, số nhà 113 Website:www.sibstrin.ru

Thành phố Tomsk

15 Trường Đại học nghiên cứu quốc gia – Đại học Bách khoa Tomsk (TPU) Địa chỉ: Văn phòng tuyển sinh SV quốc tế, LB Nga, phố Usova 4A, văn phòng 420 Website:www.iie.tpu.ru

Thành phố Novocherhask

16 Đại học Bách khoa miền Nam liên bang Nga (NPI)

Địa chỉ: phố Prosvesenia, số nhà 132, thành phố Novocherkass, tỉnh Rostov, LB Nga Website:www.npi-tu.ru

Thành phố Krasnoyarsk

17 Đại học liên bang Sibiri (SFU)

Địa chỉ: đại lộ Svobodni, số nhà 82A, phòng 448, TP Krasnoyarsk, 660041, LB Nga Website:www.sfu-kras.ru

Các bạn có nhu cầu liên hệ để nắm thêm thơng tin theo địa chỉ: số 501, đường Kim Mã, quận Ba Đình, Hà Nội

Điện thoại: (04) 37719937

(149)

NHỮNG CÂU ĐỐ MÁT - XCƠ - VA

Nguyễn Quốc Khánh (Hà Nội)

(150)

Thời u thích việc học tốn Và thời giờ, ngày nữa, không đứa trẻ lại không bắt đầu hành trình vạn dặm với tốn học toán đố, với hàng đồng hồ loay hoay với que tính, với kéo giấy, với vòng chun, với viên bi, với thước thẳng ê kê, chí với dụng cụ học quái đản để vẽ hình thù đẹp đẽ Tơi khơng thể nhớ xác biết tới câu đố logic với ba vị thần Thần nói thật ln nói thật, Thần nói dối ln nói dối, Thần hay đùa lúc nói thật lúc nói dối từ bao giờ, tơi biết chẳng có đứa trẻ thời lại khơng biết tới câu đố vậy, bây giờ, dường với rất nhiều người, toán logic tức môn câu đố ba vị thần Thật rung động cưỡng lại câu đố vui

Boris Kordemsky giải câu đố Ảnh:ageofpuzzles.com

Nói tốn đố, “Ơng tổ” trị chơi Sam Loyd người Mỹ, tới “Nhà sáng chế” Henry Dudeney người Anh, tới “Người khổng lô” Andrey Kolmogorov và “Thầy giáo làng

Boris Kordemsky người Nga, tới “Nhà ảo thuật” Martin Gardner người Mỹ, tốn học giải trí (recreational mathematics) trở nên giàu có trù phú hết Nếu người ta thấy câu đố Sam Loyd trêu đùa trí óc ngơn ngữ huyền ảo đêm Ả-rập, thấy câu đố Dudeney tình nghe qua dễ dàng mà thực tinh xảo xác cao độ khiến cho tất phải đau đầu cách thích thú, cho dù người ta thấy lại câu đố hai tác giả Kordemsky, “Những câu đố Mát-xcơ-va” mà bạn cầm tay dịch tiếng Việt sau50năm

gốc đời, thảo nguyên rộng khắp chứa đựng ý tưởng đẹp đẽ đến bất ngờ Và hai kho tàng khổng lồ, có khác biệt khơng nhỏ phong cách tốn đố Kordemsky Gardner Tơi nghĩ điều đến từ gọi “Tâm hồn Nga”, mà cảm thụ hòa hợp tuyệt đẹp người, thiên nhiên đẹp đẽ, ý niệm hiểu biết sâu sắc giới, với nhân sinh quan rộng mở, điều 1Nhà toán học vĩ đại người Nga Kolmogorov thuở nhỏ bắt đầu sáng tạo câu đố để đố bạn bè, một

(151)

mà dễ nhận không dễ định nghĩa cho thật rõ ràng, đặc biệt đặt khác biệt so với tư xác hóa mức phương Tây Anh Mỹ, vẻ đẹp lại trở nên rõ ràng

Người ta nói tốn đố trị tiêu khiển để giết thời gian luyện trí não, có lẽ khơng thế, có nữa, trị tiêu khiển chắn chưa trở nên nhàm chán, có giá trị mà người từ chối Một số tín đồ cuồng nhiệt tốn đố có lẽ khơng thể khơng nhắc tới nhà tốn học John Conway, cha đẻ “Trò chơi sống” tiếng, mơ Conway vận động tế bào thông qua mô hình tốn đố đơn giản Conway Giáo sư tốn học Đại học Princeton, nơi mà ban đầu ông có văn phịng làm việc riêng, ơng khơng ngồi đó, ngược lại, suốt ngày ơng ngồi sảnh chung, “tóm cổ” tất lơ đễnh đứng sảnh chung mà khơng nói chuyện với ai, để ơng đố họ câu đố ban đầu hấp dẫn, sau người ta thấy buồn cười tính trẻ háo hức ơng Dần dần người trốn câu đố Conway ban đầu nghe dễ mà giải q khó, họ sợ bẽ mặt, cịn trường Princeton cuối đành phải thu hồi văn phòng Conway để dành cho người khác Về sau Conway xuất 4tập

sách dày tới nghìn trang để diễn giải trị chơi mà ơng tưởng tượng ra, câu đố “hại não”, cách lại có ứng dụng đáng ngạc nhiên tự nhiên sống

Không phải ngẫu nhiên mà bị câu đố hút, đặc biệt nhà tốn học nói riêng nhà khoa học nói chung Cứ sểnh họ đố nhau, đố ăn trưa, lúc dạo, uống cà phê, tiệc trà, bên hành lang hội nghị kiện Dần dần, hiểu điều là, câu đố thứ tạo nên cảm hứng cho trí tưởng tượng tị mò tất người, đặc biệt cho trẻ em lứa tuổi mà em bắt đầu phát triển trí lực Nhưng chuyện khơng hiển nhiên tơi vừa nói Tơi cịn nhớ thuở bé bực với câu hỏi ngớ ngẩn khủng khiếp việc mèo nằm chuột hỏi ta chén chuột trước, câu hỏi việc bốn người đạp xe mạng đường hình bơng hoa đích trước, thật ngớ ngẩn, thật buồn cười, thật phi logic, thật thú vị, ngơn từ hình ảnh sống động đến Và tất nhiên chẳng muốn biết câu trả lời, cịn muốn tự tìm câu trả lời khám phá, phát tự thân đầy sung sướng, ngớ ngẩn cách đặt đầu sao, hiển trước mặt, khơng loại bỏ được, mà để lại bứt rứt khơng n? Hóa ra, ngớ ngẩn kia, lại “tình cờ” điều giúp ni dưỡng trí tưởng tượng trẻ thơ, mà trí tưởng tượng tài sản lớn đứa trẻ Albert Einstein nói tính logic dắt ta đoạn đường đủ dài đáp ứng hầu hết mong muốn bản, trí tưởng tượng lại đưa ta đến đâu ta mơ ước tới, giống việc Peter Pan bay tưởng tượng (và tất nhiên phải có niềm tin vào) việc bay Tiếc thay, dường lại điều mà nhịp sống hối sống đại khiến cho chúng khơng cịn đất sống

Một người bạn mà tơi tình cờ biết tới anh Sonny Xn Vũ, người mà vòng chưa tới

(152)

anh câu đố số24trong sách này, câu đố việc vẽ đường gấp khúc4đoạn liền

nét cắt qua9điểm, học lớn từ thưở nhỏ không quên Tôi nghĩ,

ví dụ khơng có đáng ngạc nhiên, nói ra, ta hiểu rõ số điều hiển nhiên Chẳng hạn ta suy nghĩ cho thấu đáo giá trị toán đố với phát triển trí tưởng tượng tị mị, đặc biệt đặt bối cảnh mà “toán bổ củi” ngày trở nên “lạm phát”, có lẽ khơng thể khơng nhắc lại với tốn đố ngồi việc cơng cụ, cách thức tốt để khiến học sinh u thích tốn hơn, nghĩ tốn lúc nơi, cịn cách để bố mẹ, anh chị em học toán

Cảm ơn Nhà sách Nhã Nam nhóm biên dịch đem trở lại giá trị truyền thống toán học giáo dục toán học, đặc biệt bạn đọc thủy chung lại lần có hội tận hưởng lại gió thảo ngun Xơ Viết điệu dân ca Nga dung dị mà vô sâu sắc chứa đựng câu chữ, hình ảnh gần gũi, mộc mạc, mà có hồn, ngày Hội sách tháng mùa thu vàng Hà Nội, Mùa tựu

(153)

BÀI TOÁN HAY LỜI GIẢI ĐẸP

Ban biên tập

GIỚI THIỆU

Chuyên mục lấy cảm hứng từ viết thầy Nguyễn Duy Liên số báo trước toán số6trong kỳ thi IMO2001với 5cách giải khác Mục để dành viết toán hay, lời giải đẹp câu chuyện thú vị xung quanh tốn lời giải

Tên chuyên mục mượn từ tên nhóm người u tốn Facebook anh Nguyễn Văn Lợi sáng lập “Bài toán hay – Lời giải đẹp – Đam mê toán học” Chuyên mục ghi nhận đề cử bạn đọc chọn đăng kỳ1; 2bài tốn Số chúng tơi giới thiệu với bạn đọc toán số3trong đề thi toán quốc tế năm 2005cùng với lời giải độc đáo

Bài toán Cho ba số thực dươngx; y; zthỏa mãn điều kiệnxyz >1:Chứng minh rằng x5 x2

x5Cy2Cz2 C

y5 y2 y5Cz2Cx2 C

z5 z2

z5Cx2Cy2 >0:

Lời giải (Phạm Kim Hùng1) Sử dụng giả thiếtxyz >1và bất đẳng thức AM-GM, ta có x5 x2

x5Cy2Cz2 >

x5 x2xyz

x5C.y2Cz2/xyz D

x4 x2yz x4Cyz.y2Cz2/ >

2x4 x2.y2Cz2/ 2x4C.y2Cz2/2 :

ĐặtaDx2; b Dy2; c Dz2ta cần chứng minh X2c2 a.bCc/

2a2C.bCc/2 >0;

hay

X

.a b/

a

2a2C.bCc/2

b

2b2C.cCa/2

>0;

hoặc

X

.a b/2 c

2C2c.aCb/Ca2 abCb2

Œ2a2C.bCc/2 Œ2b2C.cCa/2 >0:

Hiển nhiên đúng, đẳng thức xảy khixDy Dz D1:

(154)

Lời giải (Đáp án BTC) Chú ý x5xCy2xCz2 D

x Cy Cz

x5Cy2Cz2;nên bất đẳng thức

tương đương với

Xx2Cy2Cz2

x5Cy2Cz2 63:

Sử dụng giả thiếtxyz >1và bất đẳng thức Cauchy-Schwarz, ta có

.x2Cy2Cz2/2 6px5yzCy2

Cz2

2

6.x5Cy2Cz2/.yzCy2Cz2/;

do

x2Cy2Cz2

x5Cy2Cz2

yzCy2Cz2

x2Cy2Cz2:

Suy

Xx2Cy2Cz2

x5Cy2Cz2 62C

xyCyzCzx x2Cy2Cz2 63:

Bài toán chứng minh

Lời giải (Iurie Boreico2) Ta có

x5 x2 x5Cy2Cz2

x5 x2

x5Cx3.y2Cz2/ D

.x3 1/2.y2Cz2/

x.x2Cy2Cz2/.x5Cy2Cz2/ >0;

kết hợp với giả thiếtyz > x1;ta x5 x2

x5Cy2Cz2 >

x5 x2

x5Cx3.y2Cz2/ D

x2 x1 x2Cy2Cz2 >

x2 yz x2Cy2Cz2

Từ suy

x5 x2 x5Cy2Cz2 C

y5 y2 y5Cz2Cx2 C

z5 z2

z5Cx2Cy2 >1

xyCyzCzx x2Cy2Cz2 >0:

Vậy toán chứng minh

Bình luận Bài tốn có3điểm khó, là:

1 Bất đẳng thức không bậc cao Bất đẳng thức dạng phân thức

3 Điều kiện dạng bất đẳng thức

Thông thường với bất đẳng thức khơng có điều kiện đẳng thức, ta dùng điều kiện để thực bước chuẩn hóa, điều kiện bất đẳng thức khó ta phải biết thay chỗ (để thay hợp lệ không dẫn đến bất đẳng thức sai)

2Iurie Boreico thành viên đội tuyển IMO Moldova5năm liên tiếp từ2003đến2007:Anh giành HCV

(155)

Ba lời giải trình bày xử lý khéo léo3khó khăn mà tốn đặt theo trình tự

khác

Lời giải Phạm Kim Hùng xử lý vấn đề trước (thay1bằngxyz dưới)

Bước tinh tế khơng hiển nhiên nhìn (giảm tử số tăng mẫu số lên) tử số âm, phải nhân chéo chứng minh đánh giá Tuy nhiên bước đánh giá tự nhiên Tiếp theo bước đặt biến phụ để giảm bậc cuối bước biến đổi dạng SOS (một sáng tạo Phạm Kim Hùng)

Trong cách giải đáp án, bước sử dụng mối liên hệ tử số mẫu số phân số để tạo ra3phân số có chung tử số Tiếp theo áp dụng nghệ thuật

bất đẳng thức Cauchy - Schwarz, đồng thời sử dụngxyz >1để nâng bậc x2Cy2Cz2 6x52.yz/

1

2 Cy2Cz2;

từ tạo bất đẳng thức đồng bậc hóa (và lại có mẫu số chung, giải khó khăn thứ2

x2Cy2Cz2 x5Cy2Cz2

yzCy2Cz2 x2Cy2Cz2:

Chú ý ý thứ cách giải chứng minh cần chứng minh bất đẳng thức đề cho trường hợpxyz D1:

Trong cách giải Iurie Boreico, việc hóa chia thành2bước Đầu tiên

nhất hóa mẫu số Bước tự nhiên nếux > 1thì tử số dương phân số nhỏ

ta tăng mẫu số lên (thayy2Cz2bằngx3.y2Cz2/) cịn nếux < 1thì tử số âm phân số

nhỏ ta giảm mẫu số xuống (thayy2Cz2bằngx3.y2Cz2/) Cách viết lời giải

là tường minh điều cơng thức Sau hóa mẫu số chuyển sang hóa tử số cách thay

x bằngyz:

Với lời giải mình, Iurie Boreico, nhận giải thưởng đặc biệt IMO dành cho lời giải độc đáo Đáng ý giải đặc biệt suốt 21năm: Từ năm 1996 đến 2016:Trước Iuri Boreico có Nikolay Nikolov giải đặc biệt IMO1995:

Có điều thú vị Nikolay Nikolov, học sinh đạt giải đặc biệt năm1995;lấy cảm hứng

từ toán IMO2005 lời giải Iurie Boreico chứng minh mở rộng bất

đẳng thức này, cụ thể

Chox1; x2; : : : ; xn là số thực dương thỏa mãnx1x2 xn> 1˛ >1:Khi ta có bất

đẳng thức

n X

iD1

xi˛ xi

SCxi˛ xi >

0;

trong đóS D

n X

iD1

(156)

CÁC VẤN ĐỀ CỔ ĐIỂN VÀ HIỆN ĐẠI Ban biên tập

GIỚI THIỆU

Chuyên mục dành cho vấn đề cổ điển đại trình bày dạng tốn xâu chuỗi Đó chuỗi để giải toán đẳng chu, chứng minh đẳng thức Euler kỳ diệu1C212C

1

32 C D

6 ;một chuỗi tốn vận trù Cách trình bày xuất phát từ vấn đề đơn giản, dễ hiểu, khái niệm định nghĩa để đọc tương đối độc lập Và chuỗi nêu vấn đề định, giải tốn kinh điển hay nêu giả thuyết mới, vấn đề Lời giải thảo luận toán đăng sốNC3:

BÀI DỰ THI VÒNG CHUNG KẾT “ENTROPY – KHAI PHÁ DỮ LIỆU”

(tiếp theo kỳ trước) Bài dự thi Lê Tạ Đăng Khoa (Bảng A)

Phần I Phân tích liệu phi cấu trúc

A Giới thiệu vấn đề

Tóm tắt:Cho28:000bài báo thiếu chủ đề, nhóm báo có nội dung tương tự lại, rồi

đặt tên chủ đề phù hợp cho nhóm.

Phát biểu lại Tốn:

(157)

1 Biểu diễn file text thành features (feature extraction)

2 Sử dụng thuật Tốn để nhóm file text theo nội dung (clustering) Tối ưu hóa hai bước (optimization)

Yêu cầu2:Sau có nhóm, đặt tên chủ đề phù hợp cho nhóm Sau xuất file.csv gồm tên báo chủ đề tương ứng

B Giải vấn đề

Ý tưởng: Ta dựa từ sử dụng báo để định lượng liên quan mặt nội dung chúng

Yêu cầu1:1: Feature extraction

Bước 1:Ta clean file text để đảm bảo nội dung từ túy (lower-cased, no punctuation)

Lý do: Vì “Tơi” “tơi”, “học” “học!” hay “ngồi” “ngồi,” Việc có nhiều biến thể từ sinh nhiều features cần thiết, làm chương trình chạy chậm Kết khơng xác

Bước2:Sử dụng mơ hìnhTf - Idfđể biểu diễn file text

Lý do: Mơ hìnhTf - Idf định lượng cách hiệu độ liên quan từ đến nội dung báo chứa Biểu diễn kết hợp Term Frequency ( số lần xuất bài) Document Frequency (số viết chứa từ đó)

1 Term Frequency: Xuất nhiều độ liên quan cao

2 Document Frequency: Xuất nhiều báo từ thơng dụng, nên độ liên quan đến nội dung thấp

Inverse, tức nghịch đảo lại, ta số tỉ lệ thuận với độ liên quan

Kết quả: GọiS tập hợp tất cảntừ dùng trong28:000bài báo Một báoAsẽ biểu diễn dạng.i1; i2; : : : ; in/ ;trong ik số Tf - Idf thể độ liên quan từ vựng thứktrongS đến nội dung củaA:

(158)

Code để áp dụng biểu diễn Tf-Idf cho file text

Chú thích1:Tham số sublinear_tfDTrue

Một từ xuất 10lần, so với xuất hiện1lần, khơng có nghĩa độ liên quan nội dung mạnh hơn10lần, vấn đề Term Frequency

Ở đây, ta áp dụng “Sublinear Tf Scaling” để giải Chú thích2:Tham số max_dfD0:5:

Sau bước2;ta có lượng features lớn, chương trình chắn chậm

Mặt khác, ta cần quan tâm đến từ thật liên quan đến nội dung, mà từ phổ biến thường không Bằng cách loại bỏ từ đó, số lượng features giảm đáng kể, chương trình chạy nhanh

Ở đây, từ xuất trên50% số báo (14:000) phổ biến để xem xét Yêu cầu1:2: Clustering

Ta sử dụng thuật Toán K-Means Clustering để nhóm báo thànhK nhóm Lý do: Thuật Tốn nhanh

Tóm tắt hướng đi: Sau yêu cầu1:1;một báoAsẽ biểu diễn vector thể liên quan từ vựng đến nội dung củaA: Có thể khẳng định, vector gần nội dung báo tương ứng liên quan đến

Thuật Tốn K-Means Clustering nhóm28:000vector thànhK nhóm theo khoảng cách Khi đó, vector gần nhau, tức báo có nội dung liên quan đến nhau, nằm nhóm

Bước1:Xác địnhK (số lượng nhóm)

Bằng lấy mẫu ngẫu nhiên, ta thấy báo nhiều khả thuộc trang VNExpress (18 chuyên mục) Ngoài ra, trang báo tiếng khác có số chuyên mục dao động từ 12 đến20chuyên mục Để chắn, ta dự đốnK nhận giá trị từ10đến24: Phương pháp: Ta sử dụng phương pháp elbow

Với giá trị củaK, áp dụngK - Means Clustering ghi lại số lỗi (inertia) Vẽ đồ thị xác định vị trí “thoải” nó, tức giá trị củaK mà tăngK lên inertia giảm chậm trước Đây số nhóm thật sự, inertia giảm chậm đồng nghĩa với vector đủ gần để tạo thành1nhóm

(159)

Về lý thuyết xác suất mà mẫu mang tính đại diện cao, nên giá trị tìm choK chấp nhận

Kết quả: Ta có đồ thị sau

Vị trí “thoải” đồ thị là:KD18:Ta dự đoán số nhóm thật của28:000bài báo

Bước2:Áp dụng thuật TốnK Means Clustering vớiK D18:

Chú thích Tham số max_iterD300và n_initD5:

(160)

Do việc lấy ngẫu nhiên mà q trình khơng dừng lại, dừng lại khơng vị trí, mà ta dùng2tham số

Ở đây, chọn vị trí ngẫu nhiên5lần Ở lần, ta khơng cập nhật vị trí tâm q 300lần Sau lấy kết lần có inertia tốt

Yêu cầu1:3: Optimization

Trong Toán này, tốc độ chạy chương trình phụ thuộc vào số lượng features ta dùng để biểu diễn báo Và ta giải vấn đề yêu cầu 1:1bằng cách chỉnh lại tham số max_df

Ta thử xem xét2phương pháp thường dùng khác để giảm số features Text Analysis: Stemming: Trong tiếng Anh, ta thường dùng phương pháp để giảm số biến thể

cùng từ, ví dụ “reserve” “reservation” xem có root Tuy nhiên, theo hiểu biết tơi tiếng Việt khơng có kiểu biến thể Bỏ stopwords: Stopwords từ thường dùng khơng liên quan đến nội dung,

ví dụ “sẽ”, “đã”, “à”, v.v Chúng ta dùng thư viện JVnTextPro [1] để liệt kê loại bỏ từ từ đầu

Tuy nhiên, stopwords từ thường dùng, nên phần lớn bị ta loại bỏ từ việc tùy chỉnh tham số max_df

Yêu cầu2: Đặt tên nhóm

Sau yêu cầu 1, ta có dictionary bao gồm label (tên nhóm) text_id (thứ tự báo) nhóm Ta thực việc đặt tên nhóm sau:

1 Lấy ngẫu nhiên10bài báo nhóm

2 Google tiêu đề báo để xác định chủ đề Chọn chủ đề lặp lại nhiều

Về lý thuyết việc lấy ngẫu nhiên nhỏ mang tính đại diện chúng thuộc1nhóm nên nội dung giống

Output đoạn code tham khảo file tmp.txt nằm link source code phần kết luận báo cáo ([2])

(161)

C Kết luận

Trong báo cáo này, phân đặt tên nhóm cho28:000bài báo sau: Clean báo để đưa từ túy

2 Áp dụng mơ hình Tf-Idf để biểu diễn báo Tối ưu hóa chương trình qua việc giảm số features

4 Kết hợp thực tế Toán với phương pháp elbow để chọn giá trị K phù hợp cho thuật ToánK Means Clustering

5 Áp dụng thuật Toán K-Means Clustering với tham số phù hợp để phân nhóm28:000bài báo

6 Lấy mẫu ngẫu nhiên nhóm để đặt tên cho Báo cáo sử dụng:

1 Ngơn ngữ lập trình Python Thư viện scikit-learn Thư viện matplotlib

Phần II Phân tích liệu có cấu trúc

A Giới thiệu vấn đề

Tóm tắt: Dựa liệu doanh số chi phí thịt trộn vịng24q cơng ty QK, hãy: Xây dựng mơ hình hồi quy tuyến tính để dự đốn doanh số

Quyết định nên đầu tư vào quảng cáo hay khuyến

Xác định thị trường thịt trộn có tính chất phản chu kỳ hay khơng Xác định doanh số có tính chất mùa vụ hay khơng

Phát biểu lại Tốn:

u cầu 1.Xác định biến độc lập có liên quan (explanatory variables) để đề xuất mơ hình hồi quy tuyến tính có khả dự đốn doanh số

Yêu cầu 2. Định lượng tác động ngắn hạn dài hạn đến doanh số trường hợp $1000 quảng cáo, trường hợp $1000khuyến

(162)

B Giải vấn đề

Báo cáo xây dựng mơ hình hồi quy tuyến tính để giải cả4yêu cầu Ta thực hiện3bước sau:

1 Xác định biến độc lập có liên quan (explanatory variables) Xác định hệ số cho biến ước lượng tính xác

3 Dựa hệ số ước lượng tính xác, ta định lượng mối quan hệ biến doanh số, từ trả lời câu hỏi yêu cầu

Do đề u cầu mơ hình hồi quy tuyến tính, ta giả định quan hệ cần khảo sát tuyến tính (ta vẽ đồ thị để khẳng định điều này)

Bước Xác định biến liên quan

Quảng cáo Khuyến mãi:Để khẳng định quảng cáo khuyến có tác động dài hạn đến doanh số hay không, định lượng quan hệ này, ta:

Sử dụng4biến:

1 adv: Chi phí quảng cáo quý

2 pre_adv: Chi phí quảng cáo quý trước prom: Chi phí khuyến quý

4 pre_prom: Chi phí khuyến quý trước Cần giải quyết2vấn đề sau:

1 Hệ số biến pre_adv pre_prom có significance hay khơng?

2 Để tránh lỗi multi-collinearity, ta cần tính correlation (adv, pre_adv), (prom, pre_prom)

Chỉ số Kinh tế:Để khẳng định thị trường thịt trộn có tính phản chu kỳ hay không, ta:

Sử dụng biến diff_index hiệu số kinh tế quý này, số kinh tế quý trước

Lý do: Chỉ số kinh tế tự thân khơng nói lên điều “IndexD100” có ý nghĩa ta đem so sánh với index giai đoạn khác Vì vậy, muốn xác định tình hình kinh tế quý tốt hay tệ đi, ta cần so sánh với index quý trước

Cần trả lời2câu sau:

(163)

Tính Mùa vụ:Để khẳng định doanh số có tính mùa vụ hay khơng, ta:

Sử dụng biến cold_quarter bằng1nếu mùa lạnh (quý1và quý4), bằng0nếu mùa nóng (quý2và quý3)

Cần trả lời: Hệ số cold_quarter có significance hay khơng? Bước Xây dựng mơ hình hồi quy

Dựa trên6biến xác định, ta có bảng liệu sau:

(164)

Bước Trả lời câu hỏi

Dựa số liệu bước2, ta có:

Nhận xét Mơ hình tốt có sốR2cao Các biến có quan hệ đến doanh số p-value bé hơn5% (tức hệ số có significance)

u cầu Phương trình hồi quy tuyến tính là:

salesD255:32C6:21prom 3:72pre_promC2:53advC2:88pre_adv 12:62diff_index C51:72cold_quarter:

Yêu cầu 2:Nên đầu tư vào quảng cáo hay khuyến mãi?

Correlation (adv, pre_adv) 0:25, không đáng kể Correlation (prom, pre_prom) 0:46, không nhỏ không đáng kể so với mức giảm sốR2khi chọn1trong 2biến Do đó, ta bỏ qua lỗi multi-collinearity

Ảnh hưởng quảng cáo lên doanh thu:2:53C2:88D5:41: Ảnh hưởng khuyến lên doanh thu:6:21 3:72D2:49:

Suy đầu tư vào quảng cáo cho kết dài hạn tốt đầu tư vào khuyến Tuy nhiên, đầu tư vào khuyến cho kết tốt ngắn hạn (hệ số dương lớn), tệ tương lai (hệ số âm)

u cầu 3:Thị trường thịt trộn có tính chất phản chu kỳ hay khơng?

Có Do hệ số diff_index âm ( 12:62), significance Yêu cầu 4:Doanh số có tính chất mùa vụ hay khơng?

(165)

C Kết luận

Trong báo cáo này, chọn ra6biến để xây dựng mơ hình hồi quy:

1 adv: Chi phí quảng cáo quý

2 pre_adv: Chi phí quảng cáo quý trước prom: Chi phí khuyến quý

4 pre_prom: Chi phí khuyến quý trước

5 diff_index hiệu số kinh tế quý này, số kinh tế quý trước

6 cold_quarter bằng1nếu mùa lạnh (quý1và quý4), bằng0nếu mùa nóng (q q3)

Mơ hình hồi quy là:

salesD255:32C6:21prom 3:72pre_promC2:53advC2:88pre_adv 12:62diff_index C51:72cold_quarter:

Dựa mơ hình này, ta đến kết luận:

1 Nên đầu tư vào quảng cáo tăng doanh số nhiều dài hạn Thị trường thịt trộn có tính phản chu kỳ

3 Doanh số có tính chất mùa vụ, cụ thể tăng vào mùa lạnh (quý1và quý4)

Báo cáo sử dụng phần mềm Microsoft Excel để xây dựng mơ hình hồi quy ước lượng độ xác hệ số

Tài liệu

[1] Thư viện JVnTextPro:http://jvntextpro.sourceforge.net/

(166)

Bài dự thi Lê Vũ Hoàng (Bảng B)

Phần I Phân tích liệu phi cấu trúc

Câu hỏi Với số lượng viết lớn (hơn28:000bài viết), bạn tìm cách để

nhóm viết theo chủ đề khác Bạn đề xuất phương pháp để có thể đặt tên cho chủ đề cách hợp lý nhất.

Bước1WTách từ tiếng Việt

Vì viết dạng tiếng Việt, yêu cầu phải nhóm từ tiếng Việt lại với Điều giúp phân tích chủ đề ngữ nghĩa tiếng Việt

Phương pháp tách từ: Conditional Random Fields mơ hình Markov (Lafferty et al.,2001) với xác suất state space

p.sjo/D

1 Z.o/exp

" T X

tD1

F s; o; t /

#

;

F s; o; t /DX i

ifi.st 1; st/C

X

j

jgj.o; st/:

Ta sử dụng Viterbi algorithm để huấn luyện tập Train gồm câu Tiếng Việt Maximum Likelihood

LD N

X

jD1

loghp.s.j /jo.j //

i X

k

2 k

22: Ví dụ File data gốc:00014E4D9B4AD4F 48B770F 1AB5285494: Nội dung:

Nên mua Pentax K50 hay Pentax K S2?

Mình định dấn thân vào đường “hao tiền tốn của” với máy “entry level” để tập chụp Do thích “hàng độc” so vo với tụi bạn nên định mua máy Pentax Mình tỉnh lẻ nên khơng có điều kiện thử máy trực tiếp Bạn chụp qua cho nhận xét hai máy Mình mua máy Pentax nên không quan tâm nhiều đến Nikon, Canon, phân khúc giá

Tin Khác Đời sống số

Vịng đeo tay thơng minh từ thương hiệu tiếng thường có giá cao nên người Việt chuyển sang dùng hàng Trung Quốc với nhiều mẫu mã, tính giá bán thấp

(167)

vẫn chào bán eBay với giá cao gấp vài lần giá

Lợi độ mở giúp di động Samsung chụp tối tốt khả xóa phơng mạnh với chụp cận cảnh

Trang chủ

Kết word segmentation

Mình định dấn_thân vào con_đường \" hao tiền tốn \"

với máy \" entry level \" để tập chụp Do_thích \" hàng_độc \" so vo với tụi bạn nên quyết_định mua máy Pentax Mình tỉnh_lẻ nên khơng có điều_kiện thử_máy trực tiếp Bạn chụp qua cho nhận_xét hai máy Mình mua máy Pentax thơi nên khơng quan_tâm nhiều đến Nikon , Canon phân_khúc giá Tin Khác Đời_sống số Vịng đeo tay thơng_minh từ thương_hiệu nổi_tiếng thường có giá cao

nên_người Việt chuyển sang dùng hàng Trung_Quốc với nhiều mẫu_mã , tính_năng giá_bán thấp Những máy_nghe nhạc Apple dù ngừng sản_xuất chào_bán eBay với giá cao gấp

Từ ghép: sản xuất!sản_xuất chào bán!chào_bán Bước 2: Lọc đoạn vô nghĩa

Chỉ lấy nội dung viết, viết khơng có nội dung bỏ, viết mang tính câu hỏi liệt vào topic “Khách hỏi”

Bước 3: Topic modeling – chia article giống vào chủ đề

(168)

LDA phương pháp mở rộng LSA pLSA với phân phối Dirichlet dự đoán Bayes LDA chọn tính flexible thường cho kết tốt LSA

Để tính số topic, tính harmonic mean hàm likelihood

Như vậy, số topic cần là36:

(169)

Phần câu hỏi

1.Đề xuất mơ hình hồi quy tuyến tính (linear regression) để dự đoán doanh số bán thịt trộn cho QK.

Khi xây dựng mơ hình tuyến tính, ta có số R-squared để có hiệu số fit mơ hình data Tuy nhiên R-square khơng nói lên mơ hình có thích hợp với data hay khơng, nên chọn lựa theo R-squared bị nguy overfit (điều xảy với data này)

Để tránh trường hợp trên, ta chia data24quý thành2tập training test để chọn lựa mơ hình tốt dựa thang đo MSE (Mean square error – out of sample test) Rsquared (in -sample test)

Mơ hình1WLấy tất biến cho (training all =24quý)

Mô hình có p-value <0:05cho thấy mơ hình tuyến tính tốt cho việc dự đoán doanh số sales Tuy nhiên để định hệ số mơ hình quan trọng Việc cải thiện mơ hình cần lấy từ gợi ý kinh doanh

Mơ hình2WThêm chi phí khuyến mùa trước

Gợi ý: “Một số người khác lại cảm thấy việc khuyến có tác động làm giảm doanh số bán hàng tương lai Nghĩa là, họ cảm thấy đại lý quản lý cửa hàng mua rất nhiều thời gian khuyến sau khơng đặt hàng giai đoạn tiếp theo cho đến họ cần.

(170)

Ta có Adjusted R - squared tăng (0:68!0:84/:Test out - of - sample cho thấy mơ hình3ln có MSE thấp hơn1&2:

Suy rangược với nhận định, quảng cáo, khuyến kinh tế có ảnh hưởng đến doanh số.

Ta chọn mơ hình để dự đốn doanh số

Doanh số = 1801:2820C5:739 Chi phí khuyến quý –3:506 Chi phí khuyến quý trước C2:424 Chi phí quảng cáo quý C2:6951 Chi phí quảng cáo quý trước –1508:4553Index kinh tế quý này=Index kinh tế quý trước

2: Nếu bạn có$1:000để dành cho hai việc quảng cáo khuyến mãi, bạn nên

chọn sao? Có tác động đến việc sử dụng $1:000 trong mỗi

việc quảng cáo khuyến mãi?

Dựa mô hình tuyến tính ta thấy dành $1:000cho khuyến làm doanh số quý tăng $5:739nhưng giảm doanh số quý tới $3:506suy tổng tăng = $2:233:

Ngược lại, $1:000đầu tư cho quảng cáo làm tăng doanh số quý $2:424và quý tới $2:695 suy tổng tăngD$5:119:

Vậy ta nên chọn quảng cáo

3: Bạn có đồng ý với ý kiến chun viên phịng tài thị trường thịt trộn có tính

chất “phản chu kỳ” (counter-cyclical) so với số kinh tế? Tại sao?

Đồng ý thị trường thịt trộn có tính phản chu kìvì số eco DIndex kinh tế quý này=Index kinh tế quý trước có hệ số 1057:4553 < 0suy Index tăng so với kì trước doanh thu giảm ngược lại

4:Bạn có nghĩ có tính chất mùa vụ doanh số bán hàng hay không? Tại sao? Test1WKiểm định Independent test với biến Sales Season

H0 WSales Season độc lập

H1 WSales Season không độc lập Contingency Table

Mùa lạnh Mùa nóng

6400 5

401!500

> 500

Test statistic: Chi - square

2 DX i;j

.fij eij/2

eij

: X - squaredD0:31111; df D2;p - valueD0:8559:

(171)

Test2WCorrelation2biến Sales SeasonD0:1018suy gần khơng có tương quan tuyến tính

Ngày đăng: 07/02/2021, 21:49

Tài liệu cùng người dùng

Tài liệu liên quan